2f Property Case Digests

July 11, 2017 | Author: Jennylyn C Ong | Category: Property, Taxes, Estoppel, Judgment (Law), Res Judicata
Share Embed Donate


Short Description

Download 2f Property Case Digests...

Description

Ladera v. Hodges G.R. No. 8027-R, September 23, 1952, Vol. 48, No. 12, Official Gazette 5374 Reyes, J.B.L., J. FACTS: Paz G. Ladera entered into a contract with C.N. Hodges. Hodges promised to sell a lot with an area of 278 square meters to Ladera, subject to certain terms and conditions. The agreement called for a down payment of P 800.00 and monthly installments of P 5.00 each with interest of 1% per month, until P 2,085 is paid in full. In case of failure of the purchaser to make any monthly payment within 60 days after it fell due, the contract may be considered as rescinded or annulled. Ladera built a house on the lot. Later on, she defaulted in the payment of the agreed monthly installment. Hodges filed an action for the ejectment of Ladera. The court issued an alias writ of execution and pursuant thereto, the city sheriff levied upon all rights, interests, and participation over the house of Ladera. At the auction sale, Ladera’s house was sold to Avelino A. Magno. Manuel P. Villa, later on, purchased the house from Magno. Ladera filed an action against Hodges and the judgment sale purchasers. Judgment was rendered in favor of Ladera, setting aside the sale for non-compliance with Rule 39, Rules of Court regarding judicial sales of real property. On appeal, Hodges contends that the house, being built on a lot owned by another, should be regarded as movable or personal property. ISSUE: Whether or not Ladera’s house is an immovable property. HELD: YES. The old Civil Code numerates among the things declared by it as immovable property the following: lands, buildings, roads and constructions of all kind adhered to the soil. The law does not make any distinction whether or not the owner of the lot is the one who built. Also, since the principles of accession regard buildings and constructions as mere accessories to the land on which it is built, it is logical that said accessories should partake the nature of the principal thing.

Mindanao Bus Company v. The City Assessor and Treasurer G.R. No. L-17870, September 29, 1962, 6 SCRA 197 Labrador, J. FACTS: Petitioner Mindanao Bus Company is a public utility solely engaged in transporting passengers and cargoes by motor trucks, over its authorized lines in the Island of Mindanao, collecting rates approved by the Public Service Commission. Respondent sought to assess the following real properties of the petitioner; (a) Hobart Electric Welder Machine, (b) Storm Boring Machine; (c) Lathe machine with motor; (d) Black and Decker Grinder; (e) PEMCO Hydraulic Press; (f) Battery charger (Tungar 1

charge machine) and (g) D-Engine Waukesha-M-Fuel. It was alleged that these machineries are sitting on cement or wooden platforms, and that petitioner is the owner of the land where it maintains and operates a garage for its TPU motor trucks, a repair shop, blacksmith and carpentry shops, and with these machineries, which are placed therein. Respondent City Assessor of Cagayan de Oro City assessed at P4, 400 petitioner's above-mentioned equipment. Petitioner appealed the assessment to the respondent Board of Tax Appeals on the ground that the same are not realty. Respondents contend that said equipments, though movable, are immobilized by destination, in accordance with paragraph 5 of Article 415 of the New Civil Code. ISSUE: Whether the equipments in question are immovable or movable properties. HELD: The equipments in question are movable. So that movable equipments to be immobilized in contemplation of the law, it must first be "essential and principal elements" of an industry or works without which such industry or works would be "unable to function or carry on the industrial purpose for which it was established." Thus, the Court distinguished those movable which become immobilized by destination because they are essential and principal elements in the industry from those which may not be so considered immobilized because they are merely incidental, not essential and principal. The tools and equipments in question in this instant case are, by their nature, not essential and principle municipal elements of petitioner's business of transporting passengers and cargoes by motor trucks. They are merely incidentals—acquired as movables and used only for expediency to facilitate and/or improve its service. Even without such tools and equipments, its business may be carried on, as petitioner has carried on, without such equipments, before the war. The transportation business could be carried on without the repair or service shop if its rolling equipment is repaired or serviced in another shop belonging to another.

Makati Leasing and Finance Corporation v. Wearever Textile Mills, Inc. G.R. No. L-58469, May 16, 1983, 122 SCRA 29 De Castro, J. FACTS: To obtain financial accommodations from the Makati Leasing and Finance Corporation, the Wearever Textile discounted and assigned several receivables with them under a “receivable purchase agreement.” To secure the collection of receivables assigned, Wearever Textile executed a chattel mortgage over certain raw materials inventory, as well as machinery described as an aero dryer stentering range. Upon default of Wearever Textile, the Makati Leasing petitioned for extrajudicial foreclosure of the properties mortgaged to it. When the sheriff failed to enter Wearever Textile’s premises to seize the machinery, Makati Leasing applied for a replevin. Wearever Textile contended that it cannot be a subject of replevin or a chattel mortgage because

2

it is a real property as it is attached to the ground by means of bolts and that the only way to remove it is to destroy the concrete floor. ISSUE: Whether or not the machinery is real or personal property. HELD: The machinery is a personal property. The Supreme Court explained that if a house of strong materials may be considered as personal property for purposes of executing a chattel mortgage, there is absolutely no reason why a machinery, which is movable in its nature and becomes immobilized only by destination or purpose, may not be likewise treated as such.

Santos Evangelista v. Alto Surety and Insurance Co., Inc. G.R. No. L-11139, April 23, 1958, 103 Phil. 401 Concepcion, J. FACTS: On June 4, 1949, Santos Evangelista instituted a civil case for a sum of money. On the same date, he obtained a writ of attachment, which was levied upon a house, built by Rivera on a land situated in Manila and leased to him. In due course, judgment was rendered in favor of Evangelista, who bought the house at public auction held in compliance with the writ of execution issued in said case. When Evangelista sought to take possession of the house, Rivera refused to surrender it, upon the ground that he had leased the property from the Alto Surety & Insurance Co., Inc. and that the latter is now the true owner of said property. It appears that on May 10, 1952, a definite deed of sale of the same house had been issued to Alto Surety, as the highest bidder at an auction sale held. Hence, Evangelista instituted an action against Alto Surety and Ricardo Rivera, for the purpose of establishing his title over said house, and securing possession thereof, apart from recovering damages. After due trial, the CFI Manila rendered judgment for Evangelista, sentencing Rivera and Alto Surety to deliver the house in question to Evangelista and to pay him, jointly and severally, P40.00 a month from October, 1952, until said delivery, plus costs. ISSUE: Whether or not a house constructed by the lessee of the land on which it is built, should be dealt with, for purposes of attachment, as immovable property or as personal property. HELD: The house is not personal property, much less a debt, credit or other personal property not capable of manual delivery, but immovable property. As explicitly held, in Ladera vs. Hodges (48 OG 5374), "a true building (not merely superimposed on the soil) is immovable or real property, whether it is erected by the owner of the land or by a usufructuary or lessee. The opinion that the house of Rivera should have been attached in accordance with subsection (c) of said section 7, as "personal property capable of manual delivery, by taking and safely keeping in his custody", for it declared that "Evangelista could not have validly purchased Ricardo Rivera's house from the sheriff

3

as the latter was not in possession thereof at the time he sold it at a public auction” is untenable.

Tsai v. Court of Appeals G.R. No. 120098, October 2, 2001, 366 SCRA 324 Quisumbing, J. FACTS: On November 26, 1975, respondent Ever Textile Mills, Inc. (EVERTEX) obtained a three million peso (P3,000,000.00) loan from petitioner Philippine Bank of Communications (PBCom). As security for the loan, EVERTEX executed in favor of PBCom, a deed of Real and Chattel Mortgage over the lot where its factory stands, and the chattels located therein. On April 23, 1979, PBCom granted a second loan to EVERTEX. The loan was secured by a chattel mortgage over personal properties enumerated in a list attached thereto. After April 23, 1979, the date of the execution of the second mortgage mentioned above, EVERTEX purchased various machines and equipments. Upon EVERTEX's failure to meet its obligation to PBCom, the latter commenced extrajudicial foreclosure proceedings against EVERTEX. On December 15, 1982, the first public auction was held where petitioner PBCom emerged as the highest bidder and a Certificate of Sale was issued in its favor on the same date. On March 7, 1984, PBCom consolidated its ownership over the lot and all the properties in it. In November 1986, it leased the entire factory premises to petitioner Ruby L. Tsai. On May 3, 1988, PBCom sold the factory, lock, stock, and barrel to Tsai, including the contested machineries. On March 16, 1989, EVERTEX filed a complaint for annulment of sale, reconveyance, and damages with the Regional Trial Court against PBCom. EVERTEX claimed that no rights having been transmitted to PBCom over the assets of insolvent EVERTEX, therefore Tsai acquired no rights over such assets sold to her, and should reconvey the assets. ISSUE: Whether or not the inclusion of the questioned properties in the foreclosed properties is proper. HELD: Yes. While it is true that the questioned properties appear to be immobile, a perusal of the contract of Real and Chattel Mortgage executed by the parties gives a contrary indication. In the case at bar, the true intention of PBCOM and the owner, EVERTEX, is to treat machinery and equipment as chattels. Assuming that the properties in question are immovable by nature, nothing detracts the parties from treating it as chattels to secure an obligation under the principle of estoppel. It has been held that an immovable may be considered a personal property if there is a stipulation as when it is used as security in the payment of an obligation where a chattel mortgage is executed over it, as in the case at bar. 4

Serg’s Products, Inc. v. PCI Leasing and Finance, Inc. G.R. No. 137705, August 22, 2000, 338 SCRA 499 Panganiban, J. FACTS: Respondent PCI Leasing and Finance Inc. filed with the RTC of Quezon City a complaint for sum of money, with an application for a writ of replevin. A writ of replevin was issued, directing the sheriff to seize and deliver the machineries and equipment to PCI Leasing after five days and upon payment of the necessary expenses. The sheriff proceeded to petitioner's factory and seized one machinery. Petitioner filed a motion for special protective order invoking the power of the court to control the conduct of its officers and amend and control its processes, praying for a directive for the sheriff to defer enforcement of the writ of replevin. The motion was opposed by PCI on the ground that the properties were personal and therefore still subject to seizure and writ of replevin. In their reply, petitioners asserted that the properties were immovable as defined in Article 415 of the Civil Code, the parties' agreement to the contrary notwithstanding. Petitioners went to the Court of Appeals via an original action for certiorari. The Court of Appeals ruled that the subject machines were personal property as provided by the agreement of the parties. ISSUE: Whether or not the subject machines were personal, not real, property, which may be a proper subject of a writ of replevin. HELD: The contracting parties may validly stipulate that a real property be considered as personal. After agreeing to such stipulation, they are consequently estopped from claiming otherwise. Under the principle of estoppel, a party to a contract is ordinarily precluded from denying the truth of any material fact found therein. In the present case, the lease agreement clearly provides that the machines in question are to be considered as personal properties. Clearly then, petitioners were estopped from denying the characterization of the subject machines as personal property. Under the circumstances, they are proper subject of the writ of seizure. Accordingly, the petition was denied and the assailed decision of the Court of Appeals was affirmed.

Burgos v. Chief of Staff, AFP G.R. No. 64261, December 26, 1984, 133 SCRA 800 Escolin, J. FACTS: On December 7, 1982, two search warrants where issued and the premises at 19, Road 3, Project 6, Quezon City, and 784 Units C & D, RMS Building, Quezon Avenue, Quezon City, business addresses of the "Metropolitan Mail" and "We Forum" newspapers were searched. Office and printing machines, equipment, paraphernalia, motor vehicles and other articles used in the printing, publication and distribution of the 5

said newspapers, as well as numerous papers, documents, books and other written literature alleged to be in the possession and control of Jose Burgos, Jr. publisher-editor of the "We Forum" newspaper, were seized. ISSUE: Whether or not real properties were seized under the disputed warrants. HELD: No. Under Article 415 (5) of the Civil Code, "machinery, receptacles, instruments or implements intended by the owner of the tenement for an industry or works which may be carried on in a building or on a piece of land and which tend directly to meet the needs of the said industry or works" are considered immovable property. In Davao Sawmill Co. v. Castillo, it was said that machinery which is movable by nature becomes immobilized when placed by the owner of the tenement, property or plant, but not so when placed by a tenant, usufructuary, or any other person having only a temporary right, unless such person acted as the agent of the owner. In the present case, petitioners do not claim to be the owners of the land and/or building on which the machineries were placed. The machineries, while in fact bolted to the ground, remain movable property susceptible to seizure under a search warrant.

Lopez v. Orosa, Jr., and Plaza Theatre, Inc. G.R. No. L-10817-18, February 28, 1958, 103 Phil. 98 Felix, J. FACTS: Lopez was engaged in business under the name Lopez-Castelo Sawmill. Orosa approached Lopez and invited the latter to make an investment in the theatre business he was forming, the Plaza Theatre. Lopez expressed his unwillingness to invest. Nonetheless, Lopez agreed to supply the lumber for the construction of the theatre. Lopez further agreed that that the payment therefore would be on demand and not cash on delivery basis. Lopex delivered the lumber which was used for the construction of the Plaza Theatre. However, of the total cost of materials amounting to P62, 255.85, Lopez was paid only P 20, 848.50, thus leaving a balance of P 41, 771.35. Due to Lopez’ demands, Orosa issued a deed of assignment over his shares of stock of the Plaza Theatre, Inc. As there was still an unpaid balance, Lopez filed a case against Orosa and Plaza Theatre. He asked that Orosa and Plaza theatre be held liable solidarily for the unpaid balance, and in case defendants failed to pay, the land and building should be sold in public auction with the proceeds to be applied to the balance, or that the shares of stock be sold in public auction. ISSUE: Whether or not the lien for the value of the materials used in the construction of the building attaches to said structure alone and does not extend to the land on which the building is adhered to. HELD: No. While it is true that generally, real estate connotes the land and the building constructed thereon, it is obvious that the inclusion of the building, separate and distinct 6

from the land, in the enumeration of what may constitute real properties could only mean one thing—that a building is by itself an immovable property. In view of the absence of any specific provision to the contrary, a building is an immovable property irrespective of whether or not said structure and the land on which it is adhered to belong to the same owner. The lien so created attaches merely to the immovable property for the construction or repair of which the obligation was incurred. Therefore, the lien in favor of appellant for the unpaid value of the lumber used in the construction of the building attaches only to said structure and to no other property of the obligors.

Yap v. Tañada G.R. No. L-32917, July 18, 1988, 163 SCRA 464 Narvasa, J. FACTS: Goulds Pumps International (Phil.), Inc. filed a complaint against Yap and his wife seeking recovery of P1,459.30 representing the balance of the price and installation cost of a water pump in the latter's premises. Goulds presented evidence ex parte and judgment by default was rendered by Judge Tañada requiring Yap to pay to Goulds the unpaid balance of the pump purchased by him and interest of 12% per annum. Thereafter, the water pump in question was levied by the sheriff and by notice dated November 4, 1969, scheduled the execution sale thereof. But in view of the pendency of Yap's motion for reconsideration, suspension of the sale was directed. It appears however that a copy of the order suspending the sale was not transmitted to the sheriff Hence, the Deputy Provincial Sheriff went ahead with the scheduled auction sale and sold the property levied on to Goulds as the highest bidder. Yap argues that "the sale was made without the notice required by Sec. 18, Rule 39, of the New Rules of Court," i.e., notice by publication in case of execution sale of real property, the pump and its accessories being immovable because attached to the ground with character of permanency (Art. 415, Civil Code). ISSUE: Whether or not the water pump in question is an immovable property. HELD: No. Yap's argument is untenable. The Civil Code considers as immovable property, among others, anything "attached to an immovable in a fixed manner, in such a way that it cannot be separated therefrom without breaking the material or deterioration of the object." The pump does not fit this description. It could be, and was in fact separated from Yap's premises without being broken or suffering deterioration. Obviously, the separation or removal of the pump involved nothing more complicated than the loosening of bolts or dismantling of other fasteners.

7

Machinery and Engineering Supplies, Inc. v. Court of Appeals G.R. No. L-7057, October 29, 1954, 96 Phil. 70 Concepcion, J. FACTS: Petitioner Machinery and Engineering Supplies filed a complaint for replevin for the recovery of the machinery and equipment sold and delivered to Ipo Limestone Co. An order was issued to seize and take immediate possession of the properties specified in the order. Upon carrying out the court’s order, Roco, the company’s President, along with a crew of technical men and labourers, proceeded to the factory. The manager of Ipo Limestone Co. and Torres protested against the seizure of the properties on the ground that they are not personal properties. However, since the sheriff contended that his duty is purely ministerial, they all went to the factory and dismantled the equipment despite the fact that the equipment could not be dismantled without causing damage or injuries to the wooden frames attached to them. Consequently, they had to cut some of the supports of the equipment which rendered its use impracticable. ISSUE: Whether or not the machinery and equipment in question could be the subject of replevin. HELD: No. Replevin is applicable only to personal property. The machinery and equipment in question appeared to be attached to the land, particularly to the concrete foundation of said premises, in a fixed manner, in such a way that the former could not be separated from the latter without breaking the material or deterioration of the object. Hence, in order to remove the said outfit, it became necessary not only to unbolt the same, but also to cut some of its wooden supports. Moreover, said machinery and equipment were intended by the owner of the tenement for an industry carried on said immovable. For these reasons, they were already immovable pursuant to paragraphs 3 and 5 of Article 415 of the Civil Code.

FELS Energy, Inc. v. The Province of Batangas G.R. No. 168557, February 16, 2007 Callejo, Sr., J. FACTS: On January 18, 1993, NPC entered into a lease contract with Polar Energy, Inc. over diesel engine power barges moored at Balayan Bay in Calaca, Batangas. The contract staes that NPC shall be responsible for the payment of all taxes other levies imposed government to which POLAR may be or become subject to in respect of the Power Barges. Subsequently, Polar Energy, Inc. assigned its rights under the agreement to FELS Energy Inc. On August 7, 1995, FELS received an assessment of real property taxes on the power barges from Provincial Assessor of Batangas City. The assessed tax amounted to P56,184,088.40 per annum. FELS referred the matter to NPC, reminding it of its obligation under the agreement to pay all real estate taxes. NPC sought reconsideration 8

of the Provincial Assessor’s decision to assess real property taxes on the power barges, alleging that barges are non-taxable items. In its answer, the Provincial Assessor averred that the barges were real property for purposes of taxation under Section 199(c) of Republic Act (R.A.) No. 7160. ISSUE: Whether power barges, which are floating and movable, are personal properties and therefore, not subject to real property tax. HELD: NO. The power barges are real property and are thus subject to real property tax. Tax assessments by tax examiners are presumed correct and made in good faith, with the taxpayer having the burden of proving otherwise. Besides, factual findings of administrative bodies, which have acquired expertise in their field, are generally binding and conclusive upon the Court.

9

Laurel v. Garcia G.R. No. 92013, July 25, 1990, 187 SCRA 797 Gutierrez, J. FACTS: In view of the Reparations Agreement between the Philippines and Japan, four properties located in Japan were given to the Philippines. One of these properties is the Roppongi property. The said property was formerly the location of the Chancery of the Philippine Embassy until it was transferred to Nampeidai on July 22, 1976. The Roppongi property has remained abandoned from the time of the transfer due to lack of funds to develop the said property. Consequently, Administrative orders were issued by the President authorizing the study of the condition of the properties of the Philippines in Japan. Subsequently, Executive Order 296 was issued by President Aquino allowing non-Filipinos to buy or lease some of the properties of the Philippines located in Japan, including Roppongi. Petitioners now contend that the Roppongi property cannot be alienated as it is classified as public dominion and not of private ownership because it is a property intended for public service under paragraph 2, article 420 of the Civil Code. On the other hand, respondents aver that it has already become part of the patrimonial property of the State which can be alienated because it has not been used for public service for over 13 years. They further contend that EO 296 converted the subject property to patrimonial property. ISSUE: Whether or not the Roppongi property still forms part of the public dominion hence cannot be disposed nor alienated. HELD: Yes. The respondents failed to convincingly show that the property has already become patrimonial. The fact that the Roppongi site has not been used for a long time for actual Embassy service does not automatically convert it to patrimonial property. Under Art. 422 of the Civil Code, there must be a definite and a formal declaration on the part of the government to withdraw it from being public. Abandonment must be a certain and a positive act based on correct legal premises. The mere transfer of the embassy to Nampeidai is not a relinquishment of the property’s original purpose. The Administrative orders authorizing the study of the conditions of government properties in Japan were merely directives for investigation but did not in any way signify a clear intention to dispose of the properties. Likewise, EO 296 did not declare that the properties lost their public character; it merely made them available to foreigners in case of sale, lease or other disposition. Thus, since there is no law authorizing its conveyance, the Roppongi property still remains part of the inalienable properties of the State.

Rabuco v. Villegas G.R. No. L-24916, February 28, 1974, 55 SCRA 658 10

Teehankee, J. FACTS: The issue in this case involves the constitutionality of Republic Act No. 3120 whereby the Congress converted the lots in question together with another lot in San Andres, Malate that are reserved as communal property into disposable or alienable lands of the State. Such lands are to be placed under the administration and disposal of the Land Tenure Administration for subdivision into small lots not exceeding 120 square meters per lot for sale on instalment basis to the tenants or bona fide occupants thereof and expressly prohibited ejectment and demolition of petitioners' homes under Section 2 of the Act. Respondent contends that the Act is invalid and unconstitutional for it constitutes deprivation of property without due process of law and without just compensation. ISSUE: Whether or not Republic Act No. 3120 is constitutional. HELD: Yes. The lots in question are manifestly owned by the city in its public and governmental capacity and are therefore public property over which Congress had absolute control as distinguished from patrimonial property owned by it in its private or proprietary capacity of which it could not be deprived without due process and without just compensation. It is established doctrine that the act of classifying State property calls for the exercise of wide discretionary legislative power, which will not be interfered with by the courts. The Acts in question were intended to implement the social justice policy of the Constitution and the government program of land for the landless and that they were not intended to expropriate the property involved but merely to confirm its character as communal land of the State and to make it available for disposition by the National Government. The subdivision of the land and conveyance of the resulting subdivision lots to the occupants by Congressional authorization does not operate as an exercise of the power of eminent domain without just compensation in violation of Section 1, subsection (2), Article III of the Constitution, but simply as a manifestation of its right and power to deal with state property.

Macasiano v. Diokno G.R. No. 97764, August 10, 1992, 212 SCRA 464 Medialdea, J. FACTS: The Municipality of Paranque passed an ordinance that authorized the closure of J. Gabriel, G.G. Cruz, Bayanihan, Lt. Garcia Extension and Opena Streets located at Baclaran, Paranaque Metro Manila and the establishment of a flea market thereon. Thereafter, the municipal council of Paranaque issued a resolution authorizing Paranaque Mayor Walfrido N. Ferrer to enter into a contract with any service cooperative for the establishment, operation, maintenance and management of flea markets and/or vending areas. By virtue of this, respondent municipality and respondent Palanyag, a service cooperative, entered into an agreement whereby the latter shall operate, maintain and manage the flea market in the aforementioned streets with the 11

obligation to remit dues to the treasury of the municipal government of Paranaque. Consequently, market stalls were put up by Palanyag on the said streets. Petitioner Macasiano, PNP Superintendent of the Metropolitan Traffic Command, then ordered the destruction and confiscation of the stalls along the abovementioned streets. Hence, respondents filed with the trial court a joint petition for prohibition and mandamus with damages and prayer for preliminary injunction, to which the petitioner filed his opposition to the issuance of the writ of preliminary injunction. The trial court upheld the validity of the ordinance in question. ISSUE: Whether or not an ordinance or resolution which authorizes the lease and use of public streets or thoroughfares as sites for flea markets is valid. HELD: No. The aforementioned streets are local roads used for public service and are therefore considered public properties of respondent municipality. Article 424 of the Civil Code provides that properties of public dominion devoted for public use and made available to the public in general are outside the commerce of man and cannot be disposed of or leased by the local government unit to private persons. Properties of the local government which are devoted to public service are deemed public and are under the absolute control of Congress. Hence, LGUs have no authority whatsoever to control or regulate the use of public properties unless specific authority is vested upon them by Congress.

Republic of the Philippines v. Court of Appeals G.R. No. 100709, November 14, 1997, 281 SCRA 639 Panganiban, J. FACTS: Morato filed for a patent on a parcel of land located in Calauag, Quezon, which was approved, provided that the land shall not be encumbered or alienated within a period of five years from the date of the issuance of the patent. Later on, the land was established to be a portion of Calauag Bay, which was five to six feet deep during high tides and three feet deep on low tides. The water level rose because of the ebb and flow of tides from the bay and the storms that frequently passed through the area. Furthermore, it was observed by the Director of Lands from his investigation, that the land of Morato was leased to Advincula for P100 per month and it was also mortgaged to Co for P10,000. The Director of Lands filed a suit with the contention that Morato violated the 5-year prohibitory period and thus the patent should be cancelled and the land should revert back to the State. ISSUE: Whether or not there is a violation of the prohibition of the patent, and thus, the subject land should revert back to the ownership of the State. HELD: Yes. The lease was an encumbrance included in the prohibitions of the patent because it impairs the use of the land by Morato herself. As for the mortgage, it is a 12

legal limit on the title and if there will be foreclosure because Morato was not able to pay her debts, the property will be auctioned. It is also a limitation on Morato's right to enjoy and possess the land for herself. Encumbrance, as defined, is an impairment on the use or transfer of property, or a claim or lien on the property where there is a burden on the title. Thus, Morato clearly violated the terms of the patent on these points. Moreover, the property became a foreshore land because it turned into a portion of land which was covered most of the time with water, whether it was low or high tide. Foreshore is defined as land between high and low waters which is dry depending on the reflux or ebb of the tides. In accordance with this land reclassification, the land can no longer be subject to a pending patent application and must be returned to the State.

Province of Zamboanga del Norte v. City of Zamboanga G.R. No. L-24440, March 28, 1968, 22 SCRA 1334 Bengzon, J.P., J. FACTS: On June 6, 1952, Republic Act 711 was approved dividing the province of Zamboanga into two (2): Zamboanga del Norte and Zamboanga del Sur. Republic Act 3039 was approved providing that “all buildings, properties and assets belonging to the former province of Zamboanga and located within the City of Zamboanga are hereby transferred, free of charge, in favor of the said City of Zamboanga.” Plaintiff-appellee Zamboanga del Norte filed a complaint in the Court of First Instance of Zamboanga del Norte against defendants-appellants Zamboanga City, the Secretary of Finance and the Commissioner of Internal Revenue. It was prayed that Republic Act 3039 be declared unconstitutional for depriving plaintiff province of property without due process and just compensation. Included in the properties were the capital site and capitol building, certain school sites, hospital and leprosarium sites, and high school playground. ISSUE: Whether or not the properties mentioned are properties for public use or patrimonial. HELD: The subject properties are properties for public use. The validity of the law ultimately depends on the nature of the lots and buildings in question. The principle itself is simple: If the property is owned by the municipality (meaning municipal corporation) in its public and governmental capacity, the property is public and Congress has absolute control over it. But if the property is owned in its private or proprietary capacity, then it is patrimonial and Congress has no absolute control. The municipality cannot be deprived of it without due process and payment of just compensation. Applying the norm obtaining under the principles constituting the law of Municipal Corporations, all those of the 50 properties in question which are devoted to public service are deemed public; the rest remain patrimonial. Under this norm, to be 13

considered public, it is enough that the property be held and, devoted for governmental purposes like local administration, public education, public health, etc. Regarding the several buildings existing on the lots above-mentioned, the records do not disclose whether they were constructed at the expense of the former Province of Zamboanga. Considering however the fact that said buildings must have been erected even before 1936 when Commonwealth Act 39 was enacted and the further fact that provinces then had no power to authorize construction of buildings such as those in the case at bar at their own expense, it can be assumed that said buildings were erected by the National Government, using national funds. Hence, Congress could very well dispose of said buildings in the same manner that it did with the lots in question.

Chavez v. Public Estates Authority G.R. No. 133250, July 9, 2002 Carpio, J. FACTS: In 1973, the Government through the Commissioner of Public Highways and the Construction and Development Corporation of the Philippines (CDCP) signed a contract to reclaim certain foreshore and offshore areas of Manila Bay. PD 1084 was issued, creating Public Estates Authority (PEA), and PD 1085, transferring the reclaimed lands under the MCCRRP to PEA. In 1995, PEA entered into a Joint Venture Agreement (JVA) with AMARI, a private corporation to develop the Freedom Islands, and the JVA was approved by President Ramos. However, PEA and AMARI entered into the JVA through negotiation without public bidding. A Legal Task Force was created to look into the issue. The said task force upheld the legality of the JVA. In 1998, Frank I. Chavez, as a taxpayer, filed a petition to compel PEA to disclose all facts on its negotiations with AMARI, invoking the constitutional right of the people to information on matters of public concern. He assails the sale to AMARI of lands of the public domain as a blatant violation of the constitutional prohibiting in the sale of alienable lands of the public domain to private corporations. Despite the ongoing court petitions, PEA and AMARI signed an Amended Joint Venture Agreement (Amended JVA) in 1999, and such was approved by President Estrada. The Amended JVA seeks to convey to AMARI the ownership of 77.34 hectares of the Freedom Islands. ISSUE: Whether AMARI has the capacity to acquire the lands held by PEA. HELD: No. Under the 1987 Constitution, private corporations such as AMARI cannot acquire alienable land of the public domain. Reclaimed lands comprising the Freedom Islands, which are covered by certificates of title in the name of PEA, are alienable 14

lands of the public domain. PEA may lease these lands to private corporations but may not sell or transfer ownership of these lands to private corporations. PEA may only sell these lands to Philippine citizens, subject to the ownership limitations in the 1987 Constitution and existing laws. Thus, the Amended Joint Venture Agreement between AMARI and PEA was null and void.

Chavez v. National Housing Authority G.R. No. 164527, August 15, 2007 Velasco, Jr., J. FACTS: President Corazon Aquino issued Memorandum Order No. 161 approving and directing the implementation of the Comprehensive and Integrated Metropolitan Manila Waste Management Plan. Respondent National Housing Authority was ordered to “conduct feasibility studies and develop low–cost housing projects at the dumpsite and absorb scavengers in NHA resettlement/low–cost housing projects”, particularly in the Smokey Mountain. It produced the “Smokey Mountain Development Plan and Reclamation of the Area Across R-10” or the Smoke Mountain Development and Reclamation Project. The Project aimed to covert Smokey mountain dumpsite into a habitable housing project, inclusive of the reclamation of the area. President Aquino approved the said Project through MO 415. After President Aquino’s term, President Fidel Ramos, through Proclamation No. 39, authorized the NHA to enter into a Joint Venture Agreement with R-II Builders, Inc. (RBI) for the implementation of the project. Afterwards, President Ramos issued Proclamation No. 465 increasing the proposed area for reclamation across R-10 from 40 hectares to 79 hectares. The petitioner Francisco Chavez contended that the respondent NHA or respondent RBI has no authority to reclaim foreshore and submerged land. ISSUE: Whether or not respondent NHA has the authority to reclaim foreshore and submerged land. HELD: Yes. The National Housing Authority (NHA) is a government agency not tasked to dispose of public lands under its charter – it is an “end-user agency” authorized by law to administer and dispose of reclaimed lands. The moment titles over reclaimed lands based on the special patents are transferred to the National Housing Authority (NHA) by the Register of Deeds, they are automatically converted to patrimonial properties of the State which can be sold to Filipino citizens and private corporations, 60% of which are owned by Filipinos. The combined and collective effect of Proclamations Nos. 39 and 465 with Special Patents Nos. 3592 and 3598 is tantamount to and can be considered to be an official declaration that the reclaimed lots are alienable or disposable lands of the public domain. Even if it is conceded that there was no explicit declaration that the lands are no longer needed for public use or public service, there was however an implicit executive declaration that the reclaimed areas are not necessary anymore for public use or public service when President Aquino through MO 415 conveyed the same to the National Housing Authority (NHA) partly for 15

housing project and related commercial/industrial development intended for disposition to and enjoyment of certain beneficiaries and not the public in general and partly as enabling component to finance the project.

Manila International Airport Authority v. Court of Appeals G.R. No. 155650, July 20, 2006 Carpio, J. FACTS: MIAA received Final Notices of Real Estate Tax Delinquency from the City of Parañaque for the taxable years 1992 to 2001. MIAA’s real estate tax delinquency was estimated at P624 million. Thus, the City of Parañaque, through its City Treasurer, issued notices of levy and warrants of levy on the Airport Lands and Buildings. The Mayor of the City of Parañaque threatened to sell at public auction the Airport Lands and Buildings should MIAA fail to pay the real estate tax delinquency. City of Parañaque contends that Section 193 of the Local Government Code expressly withdrew the tax exemption privileges of “government-owned and-controlled corporations” upon the effectivity of the Local Government Code. However, MIAA avers that airport lands and buildings are owned by the State, and thus, exempt from tax. ISSUE: Whether or not airport lands and buildings of MIAA are exempt from real estate tax. HELD: Yes. MIAA is a government instrumentality vested with corporate powers to perform efficiently its governmental functions. MIAA is like any other government instrumentality, the only difference is that MIAA is vested with corporate powers. Unless the government instrumentality is organized as a stock or non-stock corporation, it remains a government instrumentality exercising not only governmental but also corporate powers. Thus, MIAA exercises the governmental powers of eminent domain, police authority and the levying of fees and charges. The airport lands and buildings of MIAA are property of public dominion and therefore owned by the State or the Republic of the Philippines. Hence, the subject properties are not subject to tax.

16

Javier v. Veridiano II G.R. No. L-48050, October 10, 1994, 237 SCRA 565 Bellosillo, J. FACTS: Javier filed a Miscellaneous Sales Application for lot 1641. She later instituted a complaint for forcible entry against Babol, alleging that she was forcibly dispossessed of a portion of said land. The case for forcibly entry was however dismissed as it was found by the court that the occupied portion was outside Lot 1641. The same was dismissed on appeal. Javier was eventually granted a Miscellaneous Sales Patent and issued an OCT for lot 1641. Babol, however had sold the property he was occupying, including a portion of 200 square meters to Rosete. Javier demanded the surrender of the same area from Rosete who repeatedly refused to comply. After 4 years, Javier instituted a complaint for quieting of title and recovery of possession with damages against Babol and Rosete. Rosete moved to dismiss the complaint on the ground of res judicata. The CFI sustained the argument of Rosete and granted his motion to dismiss. Javier contends that res judicata cannot apply in the instant case since there is no identity of parties and causes of action between her complaint for forcible entry, which had long become final and executory, and her subsequent petition for quieting of title. Javier maintains that there is no identity of causes of action since the first case was for forcible entry, which is merely concerned with the possession of the property, whereas the subsequent case was for quieting of title, which looks into the ownership of the disputed land. ISSUE: Whether or not there are really different causes of action between the forcible entry case and the later quieting of title case. HELD: Yes. For res judicata to bar the institution of a subsequent action the following requisites must concur: (1) There must be a final judgment or order; (2) The court rendering the judgment must have jurisdiction over the subject matter; (3) The former judgment is a judgment on the merits; and, (4) There is between the first and second actions identity of (4a) parties, (4b) of subject matter and (4c) of causes of action. Javier's argument that there is no identity of parties between the two actions is without merit. We have repeatedly ruled that for res judicata to apply, what is required is not absolute but only substantial identity of parties. But, there is merit in Javier's argument that there is no identity of causes of action. "The only issue in an action for forcible entry is the physical or material possession of real property, that is, possession de facto and not possession de jure. The philosophy underlying this remedy is that irrespective of the actual condition of the title to the property, the party in peaceable quiet possession shall not be turned out by strong hand, violence or terror." A judgment rendered in a case for recovery of possession is conclusive only on the question of possession and not on the ownership. It does not in any way bind the title or affects the ownership of the land or building. On the other hand, Civil Case No. 2203-0 is in reality an action to recover a parcel of land or an accion reivindicatoria under Art. 434 of the Civil Code, and should be 17

distinguished from Civil Case No. 926, which is an accion interdictal. Accion interdictal, which is the summary action for forcible entry (detentacion) where the defendant's possession of the property is illegal ab initio, or the summary action for unlawful detainer (desahuico) where the defendant's possession was originally lawful but ceased to be so by the expiration of his right to possess, both of which must be brought within one year from the date of actual entry on the land, in case of forcible entry, and from the date of last demand, in case of unlawful detainer, in the proper municipal trial court or metropolitan trial court; accion publiciana which is a plenary action for recovery of the right to possess and which should be brought in the proper regional trial court when the dispossession has lasted for more than one year; and, accion reivindicatoria or accion de reivindicacion which seeks the recovery of ownership and includes the jus utendi and the jus fruendi brought in the proper regional trial court. Accion reivindicatoria or accion de reivindicacion is thus an action whereby plaintiff alleges ownership over a parcel of land and seeks recovery of its full possession. It is different from accion interdictal or accion publiciana where plaintiff merely alleges proof of a better right to possess without claim of title. In Civil Case No. 926 Javier merely claimed a better right or prior possession over the disputed area without asserting title thereto. It should be distinguished from Civil Case No. 2203-0 where she expressly alleged ownership.

Bustos v. Court of Appeals G.R. No. 120784-85, January 24, 2001, 350 SCRA 155 Pardo, J. FACTS: Paulino Fajardo died intestate on April 2, 1957. He had four (4) children, namely: Manuela, Trinidad, Beatriz and Marcial, all surnamed Fajardo. On September 30, 1964, the heirs executed an extra-judicial partition of the estate of Paulino Fajardo. On the same date, Manuela sold her share to Moses G. Mendoza, husband of Beatriz by deed of absolute sale. At the time of the sale, there was no cadastral survey in Masantol, Pampanga. Later, the cadastre was conducted and the property involved in the partition case was specified as Lots 280, 283, 284, 1000-A and 1000-B. The share of Manuela, which was sold to Moses, includes Lot 284 of the Masantol Cadastre and Lot 284 was subdivided into Lots 284-A and 284-B. Trinidad was in physical possession of the land. She refused to surrender the land to her brother-in-law Moses G. Mendoza, despite several demands. On September 3, 1971, Moses filed with the Court of First Instance, Pampanga a complaint for partition claiming the one fourth (1/4) share of Manuela which was sold to him. During the pendency of the case for partition, Trinidad Fajardo died. On December 15, 1984, the heirs executed an extra-judicial partition of the estate of Trinidad Fajardo. On February 16, 1987, Lucio Fajardo Ignacio, son of Trinidad sold Lot 284-B to spouses Venancio Viray and Cecilia Nunga-Viray. 18

On February 8, 1989, the Regional Trial Court, Pampanga, Macabebe, Branch 55 rendered a decision in favor of Moses G. Mendoza.In the meantime, on November 6, 1989, spouses Venancio Viray and Cecilia Nunga-Viray, buyers of Lucio Ignacio's share of the property, filed with the Municipal Circuit Trial Court, Macabebe-Masantol, Pampanga an action for unlawful detainer against spouses Bustos, the buyers of Moses G. Mendoza, who were in actual possession as lessees of the husband of Trinidad, Francisco Ignacio, of the subject land. The municipal circuit trial court decided the case in favor of spouses Viray. Subsequently, the trial court issued writs of execution and demolition, but stayed when spouses Bustos filed with the regional Trial Court, Pampanga, Macabebe, Branch 55, a petition for certiorari, prohibition and injunction. On December 18, 1992, the regional trial court rendered a decision dismissing the case. On September 9, 1994, petitioners filed a motion for reconsideration; however, on June 21, 1995, the Court of Appeals denied the motion. ISSUE: Whether or not petitioners could be ejected from what is now their own land. HELD: In this case, the issue of possession is intertwined with the issue of ownership. In the unlawful detainer case, the Court of Appeals affirmed the decision of the trial court as to possession on the ground that the decision has become final and executory. This means that the petitioners may be evicted. In the accion reinvindicatoria, the Court of Appeals affirmed the ownership of petitioners over the subject land. Hence, the court declared petitioners as the lawful owners of the land. In the present case, the stay of execution is warranted by the fact that petitioners are now legal owners of the land in question and are occupants thereof. To execute the judgment by ejecting petitioners from the land that they owned would certainly result in grave injustice. Besides, the issue of possession was rendered moot when the court adjudicated ownership to the spouses Bustos by virtue of a valid deed of sale. Placing petitioners in possession of the land in question is the necessary and logical consequence of the decision declaring them as the rightful owners is possession. It follows that as owners of the subject property, petitioners are entitled to possession of the same. "An owner who cannot exercise the seven (7) "juses" or attributes of ownership-the right to possess, to use and enjoy, to abuse or consume, to accessories, to dispose or alienate, to recover or vindicate and to the fruits is a crippled owner.

Heirs of Roman Soriano v. Court of Appeals G.R. No. 128177, August 15, 2001, 363 SCRA 87 Ynares – Santiago, J. FACTS: The land in dispute in this case is originally owned by Adriano Soriano who died sometime in 1947. Adriano Soriano has 7 heirs whom leased the subject parcel of land to David de Vera and Consuelo Villasista for a term of 15 years starting July 1, 1967. The lease contract states that Roman Soriano will serve as the caretaker of the said property during the period of lease. During the effectivity of the lease contract, the 19

heirs of Adriano Soriano entered into extrajudicial settlement of his estate. As a result of the settlement, the property was divided into two property, Lot No. 60052 which was assigned to Lourdes and Candido, heirs of Adriano and the heirs of Dionisia another heir of Adriano. The other property, Lot No. 8459 was assigned to Francisco, Librada, Elcociado and Roman all heirs of Adriano. The owners of Lot No. 60052 sold the lot to spouses Braulio and Aquiliana Abalos, and the owners of Lot No. 8459, except Roman also sold their shares to spouses Briones. On March 14, 1968, the de Vera spouses ousted Roman as caretaker and appointed Isidro Versoza and Vidal Versoza as his substitute. Roman filed a case for reinstatement and reliquidation against the de Vera spouses in CAR Case No. 1724-P68. On September 30, 1969, the Agrarian Court rendered a decision authorizing the ejectment of Roman. On appeal, the decision was reversed by the Court of Appeals. The deicion became final and executor. However, before it was executed, the parties entered into a post-decisional agreement wherein the de Vera spouses allowed Roman Soriano to sub-lease the property until the termination of the original lease on June 30, 1982. This agreement was approved by the CAR court in an order dated December 22, 1972. On August 16, 1976, the Abalos spouses applied for the registration of the disputed parcel of land. Roman Soriano and the Director of Lands acted as oppositors. On June 27, 1983, the Land Registration Court granted the application for registration. On April 13, 1983, after the expiration of the original lease and sub-lease in favor of Roman Soriano, the Abalos spouses filed a case for unlawful detainer against Roman Soriano, later, this case was dismissed on motion of the Abalos spouses. On July 14, 1983, Elcociado, Librada, Roman, Francisco, Lourdes, Candido and the heirs of Dionisia filed a complaint to annul the deeds of sale they executed in favor of the Abalos spouses or should the deeds be not annulled, to allow Roman, Elcociado and Librada to redeem their shares in the disputed land and to uphold Roman Soriano’s possession of the fishpond portion of the property as a tenant-caretaker. After the dismissal of the case for unlawful detainer, the Abalos spouses filed on August 22, 1984, a motion for execution of the post-decisional order embodying the agreement of Roman Soriano and the de Vera spouses allowing the former to sublease the property. On October 25, 1984, Roman filed a motion to suspend hearing on the rental demanded by the Abalos spouses until after the other issues raised in his opposition to the motion for execution are resolved. The motion to suspend hearing on the issue of the rentals was denied and the trial court authorized the substitution of the de Vera spouses by the Abalos spouses. Roman Soriano's motion for reconsideration was denied on March 16, 1985. Roman filed petition for certiorari and prohibition in the Court of Appeals but the latter denied the petition, pending the denial of this petition, Roman Soriano died. Not satisfied with the decision of the Court of Appeals, the heirs of Roman Soriano brought this case in the Supreme Court.

20

ISSUE: Whether or not a winning party (ABALOS) in a land registration case can effectively eject the possessor (SORIANO) thereof, whose security of tenure rights is still pending determination before the DARAB. HELD: No. The Court held that a judgment in a land registration case cannot effectively used to oust the possessor of the land, whose security of tenure rights are still pending determination before the DARAB. There is no dispute that Abalos spouses' title over the land under litigation has been confirmed with finality. However, the declaration pertains only to ownership and does not automatically include possession, especially soin the instant case where there is a third party occupying the said parcel of land, allegedly in the concept of an agricultural tenant. Agricultural lessees are entitled to security of tenure and they have the right to work on their respective landholdings once the leasehold relationship is established. Security of tenure is a legal concession to agricultural lessees which they value as life itself ad deprivation of their landholdings is tantamount to deprivation of their only means of livelihood. The exercise of the right of ownership, then, yields to the exercise of the rights of an agricultural tenant. The Supreme Court decided to refrain from ruling whether petitioners may be dispossessed of the subject property while petitioner's status as tenant has not yet been declared by the DARAB.

Garcia v. Court of Appeals G.R. No. 133140, August 10, 1999, 312 SCRA 180 Puno, J. FACTS: Petitioner Atty. Pedro Garcia, with the consent of his wife Remedios Garcia, sold a parcel of land situated at Bel Air II Village, Makati to his daughter Maria Luisa Magpayo and her husband Luisito Magpayo. The Magpayos mortgaged the land to the Philippine Bank of Communications (PBCom) to secure a loan. The Magpayos failed to pay their loan upon its maturity, hence, the mortgage was extrajudicially foreclosed and at the public auction sale in which PBCom bought the land. The redemption period of the foreclosed mortgage expired without the Magpayos redeeming the same, hence, title over the land was consolidated in favor of PBCom. PBCom subsequently filed a petition for the issuance of a writ of possession over the land with the Regional Trial Court (RTC) of Makati. The RTC granted the petition. Upon service of the writ of possession, Maria Luisa Magpayo’s brother, Jose Ma. T. Garcia, who was in possession of the land, refused to honor it. Jose Garcia thereupon filed against PBCom, the Magpayos, and the RTC Sheriff the instant suit for recovery of realty and damages wherein he contended, inter alia, that at the time of the alleged sale to the Magpayo spouses, he was in possession of the property; that, when his mother Remedios Tablan Garcia died, sometime in October, 1980, he became, by operation of law, a co-owner of the property; and that, Atty. Pedro V. Garcia, at the time of the execution of the instrument in favor of the Magpayo spouses was not in possession of the subject property. 21

ISSUE: Whether or not Jose Magpayo was a co-owner of the parcel of the land in dispute. HELD: No. Possession and ownership are distinct legal concepts. Ownership exists when a thing pertaining to one person is completely subjected to his will in a manner not prohibited by law and consistent with the rights of others. Ownership confers certain rights to the owner, one of which is the right to dispose of the thing by way of sale. Atty. Pedro Garcia and his wife Remedios exercised their right to dispose of what they owned when they sold the subject property to the Magpayo spouses. On the other hand, possession is defined as the holding of a thing or the enjoyment of a right. Literally, to possess means to actually and physically occupy a thing with or without right. Possession may be had in one of two ways: possession in the concept of an owner and possession of a holder. “A possessor in the concept of an owner may be the owner himself or one who claims to be so.” On the other hand, “one who possesses as a mere holder acknowledges in another a superior right which he believes to be ownership, whether his belief be right or wrong.” The records show that petitioner Jose Garcia occupied the property not in the concept of an owner for his stay was merely tolerated by his parents. An owner’s act of allowing another to occupy his house, rent-free does not create a permanent and indefeasible right of possession in the latter’s favor. Consequently, it is of no moment that petitioner was in possession of the property at the time of the sale to the Magpayo spouses. It was not a hindrance to a valid transfer of ownership. All said, the Magpayo spouses were already the owners when they mortgaged the property to PBCom.

Rodil Enterprises, Inc. v. Court of Appeals G.R. No. 129609, November 29, 2001, 371 SCRA 79 Bellosillo, J. FACTS: Rodil Enterprises Inc. (RODIL) is the lessee of the Ides O'Racca Building (O'RACCA) since 1959 which is a property owned by the Republic of the Philippines. In 1980, Rodil entered into a sublease contract with respondents Carmen Bondoc, Teresita Bondoc-Esto, Divisoria Footwear and Chua Huay Soon, members of the Ides O’Racca Building Tenants Association, Inc. On 12 September 1982 BP 233 was enacted. It authorized the sale of "former alien properties" classified as commercial and industrial, and the O'RACCA building was classified as commercial property. RODIL and Ides O’Racca Building Tenants Association, Inc., offered to purchase the subject property. Pending action on the offer of RODIL to purchase the property, Director Factora of the Building Services and Real Property Management Office granted RODIL's request for another renewal of the lease contract on 23 September 1987 for another five (5) years from 1 September 1987. The renewal contract was forwarded to then Secretary Jose de Jesus of Department of General Services and Real Estate Property Management (DGSREPM) for approval. Upon recommendation of DGSREPM Rufino Banas, De Jesus disapproved the renewal contract in favour of Rodil and 22

recalled all papers signed by him regarding the subject. Secretary De Jesus likewise directed RODIL to pay its realty tax delinquency and ordered the issuance of a temporary occupancy permit to the ASSOCIATION. On 6 October 1987 RODIL filed an action for specific performance, damages and injunction with prayer for temporary restraining order before the Regional Trial Court of Manila against the REPUBLIC, De Jesus, Banas, Factora and the ASSOCIATION. De Jesus, Banas and Factora were later substituted by Secretary Fulgencio Factoran of the Department of Environment and Natural Resources (DENR) in the action for specific performance. On 31 May 1988 Factora issued Order No. 1 designating the Land Management Bureau represented by Director Abelardo Palad, Jr. as custodian of all "former alien properties" owned by the REPUBLIC. Pending the action for specific performance, RODIL signed a renewal contract with Director Palad which was approved by Secretary Factora. The renewal contract would extend the lease for ten (10) years from 1 September 1987. A supplement to the renewal contract was subsequently entered into on 25 May 1992 where rentals on the previous lease contract were increased. As a result, the action was dismissed in favour of Rodil. Rodil then filed an action for unlawful detainer against Divisoria Footwear, Bondoc, Bondoc-Esto and Chua Huay Soon. Upon appeal, the Court of Appeals declared the contracts null and void and dismissed the actions for unlawful detainer. ISSUE: Whether or not Rodil has the right to occupy the building by virtue of its lease contract with the Republic. HELD: Yes. The owner has the right to enjoy and dispose of a thing, without other limitations than those established by law. Every owner has the freedom of disposition over his property. It is an attribute of ownership, and this rule has no exception. The REPUBLIC being the owner of the disputed property enjoys the prerogative to enter into a lease contract with RODIL in the exercise of its jus disponendi. Hence, as lessor, the REPUBLIC has the right to eject usurpers of the leased property where the factual elements required for relief in an action for unlawful detainer are present. Private respondents claim that the agreements of 23 September 1987, 18 May 1992 and 25 May 1992 did not give rise to valid contracts.This is true only of the Contract of Lease entered into on 23 September 1987 which the REPUBLIC did not approve. RODIL neither alleged nor proved that such approval was made known to it. The socalled approval of the lease contract was merely stated in an internal memorandum of Secretary De Jesus addressed to Director Factora. This is evident from the fact that Secretary De Jesus, in his letter, asked Factora to duly execute a lease contract and forward it to his office for approval. The consequences of this fact are clear. The Civil Code provides that no contract shall arise unless acceptance of the contract is communicated to the offeror. Until that moment, there is no real meeting of the minds, no concurrence of offer and acceptance, hence, no contract. However, the same is not true of the contracts of 18 May 1992 and 25 May 1992. As argued by RODIL, these contracts are not proscribed by law; neither is there a law 23

prohibiting the execution of a contract with provisions that are retroactive. Where there is nothing in a contract that is contrary to law, morals, good customs, public policy or public order, the validity of the contract must be sustained. The Court of Appeals invalidated the contracts because they were supposedly executed in violation of a temporary restraining order issued by the Regional Trial Court. The appellate court however failed to note that the order restrains the REPUBLIC from awarding the lease contract only as regards respondent ASSOCIATION but not petitioner RODIL. While a temporary restraining order was indeed issued against RODIL, it was issued only on 25 May 1992 or after the assailed contracts were entered into. As correctly stated by petitioner, one cannot enjoin an act already fait accompli. Private respondents argue that the "renewal contract" cannot "renew" a void contract. However, they could cite no legal basis for this assertion. It would seem that respondents consider the renewal contract to be a novation of the earlier lease contract of 23 September 1987. However, novation is never presumed. Also, the title of a contract does not determine its nature. On the contrary, it is the specific provisions of the contract which dictate its nature. Furthermore, where a contract is susceptible of two (2) interpretations, one that would make it valid and another that would make it invalid, the latter interpretation is to be adopted. The assailed agreement of 18 May 1992, "Renewal of Contract of Lease," merely states that the term of the contract would be for ten (10) years starting 1 September 1987. This is hardly conclusive of the existence of an intention by the parties to novate the contract of 23 September 1987. Nor can it be argued that there is an implied novation for the requisite incompatibility between the original contract and the subsequent one is not present. Based on this factual milieu, the presumption of validity of contract cannot be said to have been overturned. Respondent ASSOCIATION claims that the Decision of the Office of the President declaring null and void the lease contracts of 18 May 1992 and 25 May 1992 should be counted in its favor. We do not agree. The contention does not hold water. It is wellsettled that a court's judgment in a case shall not adversely affect persons who were not parties thereto.

Isaguirre v. De Lara G.R. No. 138053, May 31, 2000, 332 SCRA 803 Gonzaga – Reyes, J. FACTS: Alejandro de Lara was the original applicant-claimant for a Miscellaneous Sales Application over a parcel of land with an area of 2,342 square meters. Upon his death, Alejandro de Lara was succeeded by his wife-respondent Felicitas de Lara as claimant. On this lot stands a two-story residential-commercial apartment declared for taxation purposes in the name of respondent’s sons, Apolonio and Rodolfo de Lara. When Felicitas encountered financial difficulties, she approached petitioner Cornelio M. Isaguirre. On February 10, 1960, a document denominated as “Deed of Sale and Special Cession of Rights and Interests” was executed by Felicitas and Isaguirre, 24

whereby the former sold a 250 square meter portion of the subject lot, together with the two-story commercial and residential structure standing thereon. Sometime in May 1969, Apolonio and Rodolfo de Lara filed a complaint against petitioner for recovery of ownership and possession of the two-story building. However, petitioner filed a sales application over the subject property and was issued an OCT. Due to overlapping of title, petitioner filed an action for quieting of title. Judgment was rendered in favor of the respondents. When respondent filed a motion for execution, petitioner opposed, and alleged that he had a right of retention over the property until payment of the value of the improvements he had introduced on the property. ISSUE: Whether or not petitioner can be considered a builder in good faith with respect to the improvements he made on the property. HELD: No. The petitioner is a possessor in bad faith. Based on the factual findings from this case, it is evident that petitioner knew from the very beginning that there was really no sale and that he held respondent’s property as mere security for the payment of the loan obligation. Therefore, petitioner may claim reimbursement only for necessary expenses; however, he is not entitled to reimbursement for any useful expenses which he may have incurred.

25

German Management & Services, Inc. v. Court of Appeals G.R. No. 76216 and 76217, September 14, 1989, 177 SCRA 495 Fernan, J. FACTS: Spouses Cynthia Cuyegkeng Jose and Manuel Rene Jose, residents of Pennsylvania, Philadelphia, USA are the owners of a parcel of land situated in Sitio Inarawan, San Isidro, Antipolo, Rizal, with an area of 232,942 sq. M. The land was originally registered on 5 August 1948 in the Office of the Register of Deeds Rizal as OCT 19, pursuant to a Homestead Patent granted by the President of the Philippines on 27 July 1948. On 26 February 1982, the spouses Jose executed a special power of attorney authorizing German Management Services to develop their property into a residential subdivision. Consequently, the German Management obtained Development Permit 00424 from the Human Settlements Regulatory Commission for said development. Finding that part of the property was occupied by Gernale and Villeza and 20 other persons, German Management advised the occupants to vacate the premises but the latter refused. Nevertheless, German Management proceeded with the development of the subject property which included the portions occupied and cultivated by Gernale, et.al. Gernale, et.al. filed an action for forcible entry against German Management before the MTC Antipolo, Rizal, alleging that they are mountainside farmers of Sitio Inarawan who have occupied and tilled their farmholdings some 12 to 15 years prior to the promulgation of PD 27, and that they were deprived of their property without due process of law when German Management forcibly removed and destroyed the barbed wire fence enclosing their farmholdings without notice and bulldozing the rice, corn, fruit bearing trees and other crops that they planted by means of force, violence and intimidation The MTC dismissed Gernale et.al.'s complaint for forcible entry. On appeal, the RTC sustained the dismissal by the MTC. Gernale then filed a petition for review with the Court of Appeals. Said court gave due course to their petition and reversed the decisions of the MTC and the RTC. The Appellate Court held that since Gernale, et.al. were in actual possession of the property at the time they were forcibly ejected by German Management, they have a right to commence an action for forcible entry regardless of the legality or illegality of possession. German Management moved to reconsider but the same was denied by the Appellate Court. Hence, here is the present recourse. ISSUE: Whether the doctrine of self-help may be availed of when respondents refused to vacate the premises. HELD: No. The justification that the drastic action of bulldozing and destroying the crops of the prior possessor on the basis of the doctrine of self help (enunciated in Article 429 NCC) is unavailing because the such doctrine can only be exercised at the time of actual or threatened dispossession, which is absent in the present case. When possession has already been lost, the owner must resort to judicial process for the recovery of property. This is clear from Article 536 New Civil Code which provides that "in no case may possession be acquired through force or intimidation as long as there is a possessor who objects thereto. He, who believes that he has an action or right to

26

deprive another of the holding of a thing, must invoke the aid of the competent court, if the holder should refuse to deliver the thing."

Caisip v. People of the Philippines G.R. No. L-28716, November 18, 1970, 36 SCRA 17 Concepcion, C. J. FACTS: Spouses Marcelino Guevarra and Gloria Cabalag cultivated a parcel of land known as Lot 105-A of Hacienda Palico situated in Nasugbu, Batangas, the same land used to be tenanted by Cabalag’s father when he was still alive. Hacienda Palico is owned by Roxas y Cia, administered by Antonio Chuidian, and supervised by the overseer, Felix Caisip. Prior to the incident involved, Guevarra sought recognition as a lawful tenant of Royas y Cia from the Court of Agrarian Relations but his action was dismissed. Thereafter, Roxas y Cia filed an action against Guevarra for forcible entry with prayer that Guevarra be ejected from the premises of Lot 105-A. The Justice of the Peace of Court of Nasugbu decided in favor of Roxas y Cia and on June 6, 1959, a trouble between Cabalag and Caisip occurred regarding the cutting of sugarcane. A day later, Cabalag entered again the premises of Lot 105-A and refused to be driven out by Caisip. Due to Cabalag’s tenacious attitude, Caisip sought the help of the Chief of Police of Nasugbu. The Deputy Sheriff, however, informed Caisip that his request to eject Cabalag cannot be acted upon without a proper court order. Nevertheless, the Chief of Police assigned Sergeant Ignacio Rojales and Corporal Frederico Villadelrey to Haciendo Palico. On June 17, 1959, Cabalag was seen weeding a portion of Lot 105-A which was a ricefield. Caisip approached her and bade her to leave, but she refused to do so. So, Caisip went to Sgt. Rojales and Cpl. Villadelrey and brought them to Cabalag. Rojales told Cabalag to stop weeding but she insisted on her right to stay in the said lot. While in squatting position, Cabalag was grabbed by Rojales who twisted her right arm and wrested the trowel she was holding. Villadelrey held her left hand and together Rojales forcibly dragged her towards a banana plantation while Caisip stood nearby, with a drawn gun. Cabalag shouted, “Ina ko po! Ina ko po!” and was heard by some neighbors. Zoilo Rivera, head of the tenant organization to which Cabalag was affiliated, went with them on their way to the municipal building. Upon arrival, Cabalag was turned over by Rojales and Villadelrey to the policemen on duty, who interrogated her. But upon representations made by Rivera, she was released and allowed to go home. Cabagan then filed a complaint charging Caisip, Rojales and Villadelrey of the crime of “grave coercion.” The Court of First Instance of Batangas found them guilty as charged. On appeal, The Court of Appeals affirmed the trial court’s decision. ISSUE: Whether or not the force employed by Caisip and others, in the exercise of his right granted by Article 429, is reasonably necessary to repel or prevent an actual or threatened unlawful physical invasion or usurpation of his property. 27

HELD: No. Caisip was not even entitled to the right granted by Article 429. This is totally inapplicable to the case, for, having been given 20 days from June 6th within which to vacate the lot, Cabalag did not, on June 17th and within said period, invades or usurps the said lot. She had merely remained in possession thereof, even though the hacienda owner may have become its co-possessor by reason of the prior order of the Justice of Peace Court of Nasugbu. Caisip and others did not repel or prevent an actual or threatened physical invasion or usurpation. They expelled Cabalag from a property which she and her husband were in possession, despite the fact that the Sheriff had explicitly authorized Guevarra and Cabalag to stay in said property up to June 26th, and had expressed the view that he could not oust them without a judicial order. It is clear, therefore, that Caisip, Rojales and Villadelrey, by means of violence, and without legal authority, had prevented the complainant from doing something not prohibited by law (weeding and being in Lot 105-A), and compelled her to do something against her will (stopping the weeding and leaving said lot), whether it be right or wrong, thereby taking the law into their hands, in violation of Article 286 of the Revised Penal Code.

People of the Philippines v. Pletcha G.R. No. 19029-CR, June 27, 1977, 22 CA Rep. 807 Sison, J. FACTS: Tito Pletcha, Jr., farmer, invoking ‘self-help’ in defense of the land he inherited from his father 19 years ago against the workers of Radeco Corporation, who without court order, were constructing a fence in a hacienda allegedly leased by the corporation from a certain Lopinco. Claiming actual possession and ownership and believing that the land sought to be fenced was an integral part of the land he inherited, Pletcha asked the group to desist from fenicing pending a resurvey he proposed, but he was totally ignored, thus he fought off and prevented the workers. As a result of such resistance he was prosecuted and convicted of grave coercion by the Municipal Trial Court. Pletcha appealed the decision of the MTC with the Court of Appeals. ISSUE: Whether the appellant’s action is a legitimate exercise of a private citizen’s ‘selfhelp. HELD: Yes. In the instant case,the usurper’s possession has not yet become complete and the complainants were in the act of building a fence. Such an act constitutes force in contemplation of the law. This act of trespass justified the appellant to drive them away, even by means of bolo because they refused to listen to his appeal which is reasonable. The appellant need not rush to the court to seek redress before reasonably resisting the invasion of his property. The situation required immediate action and Art. 429 gave him the self executory mechanics of self-defense and self-reliance. The provision in Art 429 of the New Civil Code confirms the right of the appellant, an owner 28

and lawful possessor, to use reasonable force to repel an invasion or usurpation, actual, threatened or physical of his property. The principle of self-defense and the protective measures related thereto, covers not only his life, but also his liberty and property. “The principle of self-help authorizes the lawful possessor to use force, not only to prevent a threatened unlawful invasion or usurpation thereof; it is a sort of self-defense. It is lawful to repel force by force. He who merely uses force to defend his possession does not possess by force. The use of such necessary force to protect propriety or possessory rights constitutes a justifying circumstance under the Penal Code.”

29

Andamo v. Intermediate Appellate Court G.R. No. 74761, November 6, 1990, 191 SCRA 195 Fernan, C.J. FACTS: Petitioner spouses Emmanuel and Natividad Andamo are the owners of a parcel of land situated in Biga (Biluso) Silang, Cavite which is adjacent to that of private respondent, Missionaries of Our Lady of La Salette, Inc., a religious corporation. Within the land of respondent corporation, waterpaths and contrivances, including an artificial lake, were constructed, which allegedly inundated and eroded petitioners' land, caused a young man to drown, damaged petitioners' crops and plants, washed away costly fences, endangered the lives of petitioners and their laborers during rainy and stormy seasons, and exposed plants and other improvements to destruction. Petitioners instituted a criminal action against the officers and directors of respondent corporation, for destruction by means of inundation under Article 324 of the Revised Penal Code. Subsequently, petitioners filed a civil action against respondent corporation for damages. The trial court dismissed the civil case for lack of jurisdiction, as the criminal case which was instituted ahead of the civil case was still unresolved. The appellate court affirmed the order of the trial court. The motion for reconsideration was also denied. ISSUE: Whether a corporation, which has built through its agents, waterpaths, water conductors and contrivances within its land, thereby causing inundation and damage to an adjacent land, can be held civilly liable for damages. HELD: Yes. Petitioners' complaint sufficiently alleges that petitioners have sustained and will continue to sustain damage due to the waterpaths and contrivances built by respondent corporation. It must be stressed that the use of one's property is not without limitations. Article 431 of the Civil Code provides that "the owner of a thing cannot make use thereof in such a manner as to injure the rights of a third person." SIC UTERE TUO UT ALIENUM NON LAEDAS. Moreover, adjoining landowners have mutual and reciprocal duties which require that each must use his own land in a reasonable manner so as not to infringe upon the rights and interests of others. Although we recognize the right of an owner to build structures on his land, such structures must be so constructed and maintained using all reasonable care so that they cannot be dangerous to adjoining landowners and can withstand the usual and expected forces of nature. If the structures cause injury or damage to an adjoining landowner or a third person, the latter can claim indemnification for the injury or damage suffered. Article 2176 of the Civil Code imposes a civil liability on a person for damage caused by his act or omission constituting fault or negligence. However, responsibility for fault or negligence under the said article is entirely separate and distinct from the civil liability arising from negligence under the Penal Code. The plaintiff cannot recover damages twice for the same act or omission of the defendant. The decision is reversed and set aside.

30

Republic of the Philippines v. Court of Appeals G.R. No. L-43938, April 15, 1988, 160 SCRA 228 Cruz, J. FACTS: An application for registration of a parcel of land was filed on February 11, 1965, by Jose de la Rosa on his own behalf and on behalf of his three children. The land, situated in Tuding, Itogon, Benguet Province, was divided into 9 lots. According to the application, Lots 1-5 were sold to Jose de la Rosa and Lots 6-9 to his children by Mamaya Balbalio and Jaime Alberto, respectively, in 1964. In support of the application, both Balbalio and Alberto testified that they had acquired the subject land by virtue of prescription Balbalio claimed to have received Lots 1-5 from her father shortly after the Liberation. Alberto said he received Lots 6-9 in 1961 from his mother, Bella Alberto. She was corroborated by Felix Marcos, who recalled the earlier possession of the land by Alberto's father. Benguet opposed on the ground that the June Bug mineral claim covering Lots 1-5 was sold to it on September 22, 1934, by the successors-in-interest of James Kelly, who located the claim in September 1909 and recorded it on October 14, 1909. From the date of its purchase, Benguet had been in actual, continuous and exclusive possession of the land in concept of owner. Atok alleged that a portion of Lots 1-5 and all of Lots 6-9 were covered by the Emma and Fredia mineral claims located by Harrison and Reynolds on December 25, 1930, and recorded on January 2, 1931, in the office of the mining recorder of Baguio. These claims were purchased from these locators on November 2, 1931, by Atok, which has since then been in open, continuous and exclusive possession of the said lots. The Bureau of Forestry Development also interposed its objection, arguing that the land sought to be registered was covered by the Central Cordillera Forest Reserve under Proclamation No. 217 dated February 16, 1929. Moreover, by reason of its nature, it was not subject to alienation under the Constitutions of 1935 and 1973. The trial court denied the application, holding that the applicants had failed to prove their claim of possession and ownership of the land sought to be registered. The applicants appealed to the respondent court, which reversed the trial court and affirmed the surface rights of the de la Rosas over the land while at the same time reserving the sub-surface rights of Benguet and Atok by virtue of their mining claims. Both Benguet and Atok appealed to the Supreme Court, invoking their superior right of ownership. The Republic filed its own petition for review and reiterated its argument that neither the private respondents nor the two mining companies had any valid claim to the land because it was not alienable and registerable. ISSUE: Whether or not Benguet and Atok have a better right over the property in question. HELD: Yes. It is true that the subject property was considered forest land and included in the Central Cordillera Forest Reserve, but this did not impair the rights already vested in Benguet and Atok at that time. The perfection of the mining claim converted the property to mineral land and under the laws then in force removed it from the public domain. By such act, the locators acquired exclusive rights over the land, against even 31

the government, without need of any further act such as the purchase of the land or the obtention of a patent over it. As the land had become the private property of the locators, they had the right to transfer the same, as they did, to Benguet and Atok. It is true, as the Court of Appeals observed, that such private property was subject to the "vicissitudes of ownership," or even to forfeiture by non-user or abandonment or, as the private respondents aver, by acquisitive prescription. The Court of Appeals justified this by saying there is "no conflict of interest" between the owners of the surface rights and the owners of the sub-surface rights. Under the aforesaid ruling, the land is classified as mineral underneath and agricultural on the surface, subject to separate claims of title. However, the rights over the land are indivisible and that the land itself cannot be half agricultural and half mineral. The classification must be categorical; the land must be either completely mineral or completely agricultural. In the instant case, as already observed, the land which was originally classified as forest land ceased to be so and became mineral — and completely mineral — once the mining claims were perfected. As long as mining operations were being undertaken thereon, or underneath, it did not cease to be so and become agricultural, even if only partly so, because it was enclosed with a fence and was cultivated by those who were unlawfully occupying the surface. This is an application of the Regalian doctrine. If a person is the owner of agricultural land in which minerals are discovered, his ownership of such land does not give him the right to extract or utilize the said minerals without the permission of the State to which such minerals belong. Benguet and Atok have exclusive rights to the property in question by virtue of their respective mining claims which they validly acquired before the Constitution of 1935 prohibited the alienation of all lands of the public domain except agricultural lands, subject to vested rights existing at the time of its adoption. The land was not and could not have been transferred to the private respondents by virtue of acquisitive prescription, nor could its use be shared simultaneously by them and the mining companies for agricultural and mineral purposes. The decision is set aside and that of the trial court is reinstated.

32

Custodio v. Court of Appeals G.R. No. 116100, February 9, 1996, 253 SCRA 483 Regalado, J. FACTS: Pacifico Mabasa owns a parcel of land with a two-door apartment. Said property may be described to be surrounded by other immovables owned by petitioner Spouses Custodio, Spouses Santos and Rosalina Morato. From the main street P. Burgos, there are two possible passageways to Mabasa’s property. One of the tenants of the apartment vacated because an adobe fence was constructed thereby making the first passageway narrower in width. Ma. Cristina Santos testified that she constructed said fence for security reasons. Morato also constructed her fence and even extended it in such a way that the entire passageway was enclosed. It was then that the remaining tenants of the apartment left. Thereafter, Mabasa filed a case for the grant of an easement of right of way against petitioners. The RTC granted the easement of right of way sought by private respondent. On appeal, the CA affirmed the decision of the RTC and furthermore, ordering petitioners to pay private respondent a sum of money for damages. ISSUE: Whether the award of damages to private respondent is proper. HELD: No, the act of petitioners in constructing a fence within their lot is a valid exercise of their right as owners. Article 430 of the Civil Code provides that “every owner may enclose or fence his land or tenements by means of walls, ditches, live or dead hedges, or by any other means without detriment to servitudes constituted thereon. The proper exercise of a lawful right cannot constitute a legal wrong for which an action will lie, although the act may result in damage to another. The courts can give no redress for hardship to an individual resulting from action reasonably calculated to achieve a lawful end by lawful means.

33

Abejaron v. Nabasa G.R. No. 84831, June 20, 2001, 359 SCRA 47 Puno, J. FACTS: Petitioner Abejaron avers that he is the actual and lawful possessor and claimant of a 118-square meter portion of a 175-square meter residential lot in Silway, General Santos City. Petitioner Abejaron and his family occupied the 118-square meter land. At that time, the land had not yet been surveyed. They fenced the area and built thereon a family home with nipa roofing and a small store. Petitioner later improved their abode to become a two-storey house. This house, which stands to this day, occupies a portion of Lot 1, Block 5 and a portion of the adjoining Lot 2 of the same Psu. Lot 2 belongs to petitioners' daughter, Conchita. The small store was eventually destroyed and in its stead, petitioner Abejaron another store. He later planted five coconut trees on the property. Knowing that the disputed land was public in character, petitioner declared only his house, and not the disputed land, for taxation purposes. The last two declarations state that petitioners' house stands on Lots 1 and 2, Block 5. Petitioner stated that respondent Nabasa resided on the remaining 57-square meter portion of Lot 1. Nabasa built his house about 4 meters away from petitioner Abejaron's house. Employees of the Bureau of Lands surveyed the area. Abejaron did not apply for title of the land on the belief that he could not secure title over it as it was government property. Without his knowledge and consent, Nabasa applied for and caused the titling in his name the entire Lot 1, including petitioner Abejaron's 118-square meter portion. Nabasa was issued an Original Certificate of Title pursuant to a Free Patent covering Lot 1. As the title included petitioner Abejaron’s portion of the lot, he filed a protest with the Bureau of Lands against Nabasa's title and application. The protest was dismissed for failure of the petitioner to attend the hearings. Petitioner Abejaron then filed an action for reconveyance with damages against respondent Nabasa before the RTC. The RTC The Regional Trial Court ruled in favor of petitioner in its reconveyance case declaring the possession and occupancy of Abejaron over 118 square meters of lot in good faith and thereby declaring the inclusion of said portion in the OCT issued in the name of Nabasa erroneous. On appeal, the CA reversed the decision of the RTC stating that the only basis for reconveyance is actual fraud which in this case was failed to be substantiated by Abejaron. Without proof of irregularity neither in the issuance of title nor in the proceedings incident thereto nor a claim that fraud intervened in the issuance of the title, the title would become indefeasible. The petitioner hence resorts to the Supreme Court. ISSUE: Whether or not petitioner has acquired title over the disputed land. HELD: An action for reconveyance of a property is the sole remedy of a landowner whose property has been wrongfully or erroneously registered in another's name after one year from the date of the decree so long as the property has not passed to an innocent purchaser for value. The action does not seek to reopen the registration proceeding and set aside the decree of registration but only purports to show that the person who secured the registration of the property in controversy is not the real owner thereof. Fraud is a ground for reconveyance. For an action for reconveyance based on 34

fraud to prosper, it is essential for the party seeking reconveyance to prove by clear and convincing evidence his title to the property and the fact of fraud. Reconveyance is a remedy granted only to the owner of the property alleged to be erroneously titled in another's name. In the case at bench, petitioner does not claim to be the owner of the disputed portion. Admittedly, what he has is only a "preferential right" to acquire ownership thereof by virtue of his actual possession since January 1947. Title to alienable public lands can be established through open, continuous, and exclusive possession for at least 30 years. Not being the owner, petitioner cannot maintain the present suit. Persons who have not obtained title to public lands could not question the titles legally issued by the State.

35

Bachrach Motor Co., Inc. v. Talisay – Silay Milling Co. G.R. No. 35223, September 17, 1931, 56 Phil. 117 Romualdez, J. FACTS: On December 22, 1923, the Talisay-Silay Milling Co., Inc., was indebted to the Philippine National Bank. To secure the payment of its debt, it succeeded in inducing its planters, among whom, was Mariano Lacson Ledesma, to mortgage their land to the creditor bank. And in order to compensate those planters for the risk they were running with their property under the mortgage, the aforesaid central, by a resolution passed on that same date, i.e., December 22, 1923, undertook to credit the owners of the plantation thus mortgaged every year with a sum equal to two per centum of the debt secured according to yearly balance, the payment of the bonus being made at once, or in part from time to time, as soon as the central became free of its obligations to the aforesaid bank, and of those contracted by virtue of the contract of supervision, and had funds which might be so used, or as soon as it obtained from said bank authority to make such payment. Bachrach Motor Co., Inc. filed a complaint against the Talisay-Silay Milling Co., Inc., for the delivery of the amount P13,850 or promissory notes or other instruments or credit for that sum payable on June 30, 1930, as bonus in favor of Mariano Lacson Ledesma. The Philippine National Bank filed a third party claim alleging a preferential right to receive any amount which Mariano Lacson Ledesma might be entitled to from the Talisay-Silay Milling Co. as bonus, because that would be civil fruits of the land mortgaged to said bank by said debtor for the benefit of the central referred to, and by virtue of a deed of assignment, and praying that said central be ordered to delivered directly to the intervening bank said sum on account of the latter's credit against the aforesaid Mariano Lacson Ledesma. ISSUE: Whether or not the bonus in question is civil fruits HELD: No. The said bonus bears no immediate, but only a remote accidental relation to the land mentioned, having been granted as compensation for the risk of having subjected one's land to a lien in favor of the bank, for the benefit of the entity granting said bonus. If this bonus be income or civil fruits of anything, it is income arising from said risk, or, if one chooses, from Mariano Lacson Ledesma's generosity in facing the danger for the protection of the central, but certainly it is not civil fruits or income from the mortgaged property. Hence, the amount of the bonus, according to the resolution of the central granting it, is not based upon the value, importance or any other circumstance of the mortgaged property, but upon the total value of the debt thereby secured, according to the annual balance, which is something quite distinct from and independent of the property referred to.

Equatorial Realty Development, Inc. v. Mayfair Theater, Inc. 36

G.R. No. 136221, May 12, 2000, 370 SCRA 56 Pardo, J. FACTS: Carmelo and Bauermann, Inc. use to own a parcel of land, together with two 2-storey buildings constructed thereon. Carmelo entered into a Contract of Lease with Mayfair Theater Inc. for a period of 20 years. The lease covered a portion a portion of the second floor and mezzanine of a 2-storey building which respondent used as a movie house known as Maxim Theater. Two years later, Mayfair entered into a second Contract of Lease with of Carmelo for the lease of another portion of the latter’s property – namely, part of the second floor of the 2-storey building and two store spaces on the ground floor and the mezzanine, on which Mayfair put up another movie house known as Miramar Theater. The contract was likewise for a period of 20 years. Both leases contained a provision granting Mayfair a right of first refusal to purchase the subject properties. However, the subject properties were sold by Carmelo to Equatorial Realty Development, Inc. without offering it first to Mayfair. Mayfair filed a Complaint before the RTC of Manila for the annulment of the Deed of Absolute Sale between Carmelo and Equatorial. The RTC rendered its decision in favour of Carmelo and Equatorial. The Court of Appeals completely reversed and set aside the judgment of the lower court. The Supreme Court denied the petition for review and rescinded the contract of sale between Carmelo and Equatorial and ordered Carmelo to allow Mayfair to buy the lots. However, Carmelo could no longer be located. Thus, following the order of execution of the trial court, Mayfair deposited with the clerk of court a quo its payment to Carmelo. The lower court issued a Deed of Reconveyance in favour of Carmelo and a Deed of Sale in favor of Mayfair. Later, Equatorial filed with the trial court an action for the collection of the sum of money against Mayfair, claiming payment of rentals or reasonable compensation for the defendant’s use of subject premises after its lease contract had expired. ISSUE: Whether or not Equatorial should be entitled to back rentals. HELD: No. Rescission creates the obligation to return the things which were the object of the contract, together with their fruits, and the price with its interest. It is clear the Equatorial never took actual control and possession of the property sold, in view of Mayfair’s timely objection to the sale and continued actual possession of the property. Furthermore, the fact that Mayfair paid rentals to Equatorial during the litigation should not be interpreted to mean actual delivery or ispo facto recognition of Equatorial’s title. They were made merely to avoid imminent eviction and should not be construed as recognition of Equatorial as new owner.

37

Ignacio v. Hilario G.R. No. L-175, August 30, 1946, 76 Phil. 605 Moran, C. J. FACTS: This case concerns the ownership of a parcel of land, partly rice-land and partly residential. The lower court rendered judgment holding plaintiffs as the legal owners of the whole property but conceding to defendants the ownership of the houses and granaries built by them on the residential portion with the rights of a possessor in good faith, in accordance with article 361 of the Civil Code. Subsequently, the plaintiffs prayed for an order of execution alleging that since they chose neither to pay defendants for the buildings nor to sell to them the residential lot, said defendants should be ordered to remove the structure at their own expense and to restore plaintiffs in the possession of said lot. Defendants objected to this motion which, after hearing, was granted by Judge Natividad. Hence, this petition by defendants praying for (a) a restraint and annulment of the order of execution issued by Judge Natividad; (b) an order to compel plaintiffs to pay them the sum of P2,000 for the buildings, or sell to them the residential lot for P45; or (c), a rehearing of the case for a determination of the rights of the parties upon failure of extra-judicial settlement. ISSUE: Whether the respondent Court erred in its judgment. HELD: Yes. The Civil Code provides: ART. 361. The owner of land on which anything has been built, sown or planted in good faith, shall have the right to appropriate as his own the work, sowing or planting, after the payment of the indemnity stated in articles 453 and 454, or to oblige the one who built or planted to pay the price of the land, and the one who sowed, the proper rent. ART. 453. Necessary expenses shall be refunded to every possessor; but only the possessor in good faith may retain the thing until such expenses are made good to him. Useful expenses shall be refunded to the possessor in good faith with the same right of retention, the person who has defeated him in the possession having the option of refunding the amount of the expenses or paying the increase in value which the thing may have acquired in consequence thereof. The owner of the building erected in good faith on a land owned by another, is entitled to retain the possession of the land until he is paid the value of his building, under article 453. The owner of the land, upon the other hand, has the option, under article 361, either to pay for the building or to sell his land to the owner of the building. But he cannot, as respondents here did, refuse both to pay for the building and to sell the land and compel the owner of the building to remove it from the land where it is erected. He is entitled to another motion only when, after having chosen to sell his land, the other party fails to pay for the same.

38

The Court holds, therefore, that the order of Judge Natividad compelling defendantspetitioners to remove their buildings from the land belonging to plaintiffs-respondents only because the latter chose neither to pay for such buildings not to sell the land, is null and void, for it amends substantially the judgment sought to be executed and is, furthermore, offensive to articles 361 and 453 of the Civil Code.

Ignao v. Intermediate Appellate Court G.R. No. 72876, January 18, 1991, 193 SCRA 17 Fernan, C. J. FACTS: Petitioner Florencio Ignao and his uncles Juan Ignao and Isidro Ignao were coowners of a 534sqm land located in Cavite. Pursuant to an action for partition filed by petitioner, the CFI of Cavite directed the partition of the said land. A total of 133.5 sqm was allotted to the petitioner’s uncles while the remaining 266.5 was allotted to the petitioner. However, when Juan and Isidro built their houses they encroached upon a portion of land belonging to Florencio. A geodetic engineer surveyed the land and it was found out that Juan and Isidro occupied a total of 101sqm of Florencio’s lot. The trial court which based its decision on Article 448 of the Civil Code, ruled that Florencio should have the choice to either appropriate to himself that part of the house standing on his lot or to require Juan and Isidro to pay the price of the land. But since the first option seems to be impractical, it ordered to sell to Juan and Isidro those portions occupied by them because it is the “workable solution”. Upon appeal petitioner contends that Article 448 cannot be applied because they are co-owners of he subject property. However, the appellate court affirmed in toto the decision of the trial court. ISSUE: Whether or not Article 448 of the Civil Code is applicable in the case at bar. HELD: Yes. It is true that Article 448 cannot be applied where a co-owner builds upon a land owned in common. However, in the case at bar, the co-ownership has already been terminated by virtue of the partition, thus, Article 448 now applies since the builder is not anymore considered as an owner of the land where the house was built. As to the workable solution applied by the lower court, the same cannot be upheld because Article 448 clearly states that the right of choice belongs to the land owner and not upon the builder and the courts. Thus, whether it might seem impractical, the landowner may choose to appropriate the improvements.

Filipinas Colleges, Inc. v. Garcia Timbang, et. al., G.R. No. L-12812, September 29, 1989, 164 SCRA 287 Barrera, J.

39

FACTS: After appropriate proceedings, the Court of Appeals held, among other things, that Filipinas Colleges, Inc. are declared to have acquired the rights of the spouses Timbang in the questioned lots, they are ordered to pay the spouses Timbang in the amount of P15,807.90 plus such other amount which said spouses might have paid or had to pay. On the other hand, Maria Gervacio Blas was also declared to be a builder in good faith of the school building constructed in the lot in question and was entitled to be paid the amount of P19,000.00 for the same. Also, in case that Filipinas Colleges, Inc. failed to deposit the value of the land, which after liquidation was fixed at P32,859.34, within the 90-day period set by the Court, Filipinas Colleges would lose all its rights to the land and the spouses Timbang would then become the owners thereof. If that is the case, the Timbangs are ordered to make known to the court their option under Article 448 of the Civil Code whether they would appropriate the building in question, in which even they would have to pay Filipinas Colleges, Inc. the sum of P19,000.00, or would compel the latter to acquire the land and pay the price thereof. Filipinas Colleges, Inc. failed to pay the sum of P32,859.34 so the spouses Timbang made known to the court their decision that they had chosen not to appropriate the building but to compel Filipinas Colleges, Inc., for the payment of the sum of P32,859,34 which was granted by the Court. As a consequence of which, a writ of execution was issued. Meanwhile, Blas filed a motion for execution of her judgment representing the unpaid portion of the price of the house sold to Filipinas which was granted. Levy was made on the house in virtue of the writs of execution. Then, the Sheriff of Manila sold the building in public auction in favor of the spouses Timbang, as the highest bidders. Several motion were the subsequently filed before the lower court wherein the court held that: a) the Sheriff's certificate of sale covering a school building sold at public auction was null and void unless within 15 days from notice of said order spouses Timbang shall pay to Blas the sum of P5,750.00 that the spouses Timbang had bid for the building at the Sheriff's sale; b) that Filipinas is owner of 245.00/32,859.34 undivided interest in Lot No. 2-a on which the building sold in the auction sale is situated; and c) that the undivided interest of the Filipinas in the lot should be sold to satisfy the unpaid portion of the judgment in favor of Blas and against Filipinas in the amount of P8,200.00 minus the sum of P5,750.00. The spouses Timbang contends that because the builder in good faith has failed to pay the price of the land after the owners thereof exercised their option under Article 448 of the Civil Code, the builder lost his right of retention provided in Article 546 and that by operation of Article 445, the spouses Timbang as owners of the land automatically became the owners ipso facto of the school building. ISSUE: Whether or not the spouses Timbang automatically become the owners of the building upon failure of Filipinas to pay the value of the land. HELD: No. Based on Article 448 and 546 of the New Civil Code, the owner of the land has the right to choose between appropriating the building by reimbursing the builder of the value thereof or compelling the builder in good faith to pay for his land. Even this second right cannot be exercised if the value of the land is considerably more than that of the building. In addition to the right of the builder to be paid the value of his improvement, Article 546 gives him the corollary right of retention of the property until he is indemnified by the owner of the land. There is nothing in the language of these 40

two articles, 448 and 546, which would justify the conclusion of appellants that, upon the failure of the builder to pay the value of the land, when such is demanded by the land-owner, the latter becomes automatically the owner of the improvement under Article 445. The case of Bataclan vs Bernardo cannot be applied in this case in the sense that although it is true it was declared therein that in the event of the failure of the builder to pay the land after the owner thereof has chosen this alternative, the builder's right of retention provided in Article 546 is lost, nevertheless there was nothing said that as a consequence thereof, the builder loses entirely all rights over his own building. Also, in the present case, the Court of Appeals has already adjudged that appellee Blas is entitled to the payment of the unpaid balance of the purchase price of the school building. Blas is actually a lien on the school building are concerned. The order of the lower court directing the Timbang spouses, as successful bidders, to pay in cash the amount of their bid in the sum of P5,750.00 is therefore correct.

Manotok Realty v. Tecson G.R. No. L-47475 August 19, 1988, 164 SCRA 287 Gutierrez Jr., J. FACTS: Petitioner Manotok Realty filed a complaint against Nilo Madlangawa for recovery of possession with damages with the Court of First Instance of Manila. Said court rendered judgment declaring Madlangawa as a builder-possessor in good faith; ordering the company to recognize the right of Madlangawa to remain in Lot 345, Block 1, of the Clara Tambunting Subdivision until after he shall have been reimbursed by the company the sum of P7,500.00, without pronouncement as to costs. Not satisfied with the trial court’s decision, petitioner appealed to the Court of Appeals and upon affirming the trial court’s decision, it elevated the case to the Supreme Court. On July 13, 1977, the Supreme Court issued a resolution denying Manotok’s petition for lack of merit. Petitioner then filed with the trial court (Judge Jose H. Tecson), a motion for the approval of the company’s exercise of option and for satisfaction of judgment. However, Judge Tecson denied the motion for approval. Hence, this petition is filed. ISSUE: Whether or not respondent Judge Tecson can deny petitioner’s (landowner) motion to avail of its option. HELD: No. There is, therefore, no basis for the respondent judge to deny the petitioner’s motion to avail of its option to appropriate the improvements made on its property. Neither can the judge deny the issuance of a writ of execution because the private respondent was adjudged a builder in good faith or on the ground of “peculiar circumstances which supervened after the institution of this case, like, for instance, the introduction of certain major repairs of and other substantial improvements…” because the option given by law belongs to the owner of the land. Under Article 448 of the Civil Code, the right to appropriate the works or improvements or to oblige the one who built or planted to pay the proper price of the land belongs to the owner of the land. The only 41

right given to the builder in good faith is the right of reimbursement of necessary expenses for the preservation of the land; the builder cannot compel the landowner to sell such land to the former.

Bernardo v. Bataclan G.R. No. L-44606, November 28, 1938, 66 Phil. 598 Laurel, J. FACTS: Bernardo bought a parcel of land from Samonte which was located in Cavite. In order that he may take possession and occupy the said land, he filed a case in the CFI for such purpose and the court rendered a favorable decision for Bernardo. However, when he was supposedly set in occupying the said land, he found Bataclan. He was within the premises because he was authorized by the previous owners to clear the land and make the necessary improvements he deems fit, further claiming that such authorization was granted to him ever since 1922. Since Bataclan was not a party in the first case, Bernardo filed against him a separate case. Bernardo was declared owner but the defendant was held to be a possessor in good faith for whom the work done and improvements made by him should be reimbursed. An appeal to the decision of the court was filed by both Bernardo and Bataclan. The decision was modified by lowering the price of the land from P300 to P200 per hectare. Bernardo was given 30 days to exercise his option, whether to sell the land to Bataclan or to buy the improvements from him. Bernardo chose the option which would require Bataclan to pay him the value of the land at the rate of P200 per hectare. However, Bataclan informed the court that he will not be able to pay for the price of the land. The court then gave Bataclan 30 days to pay the price of the property and after the lapse of the period, the land shall be sold in a public auction. After 30 days, the land was sold to Teodoro at a public auction, after failure of Bataclan to pay within the period the purchase price. ISSUE: Whether or not Bataclan has the right of retention over the parcel of land in question. HELD: No. Bataclan no longer has lost the right of retention. The option of the owner was already exercised where he decided that he will just allow the defendant to purchase the land such that Bataclan was to comply with the option if he wants to retain the land. From the moment that he told the courts of his inability to pay for the price of the land, he already lost his right to retain the land.

Heirs of Ramon Durano, Sr. v. Uy G.R. No. 136456 October 24, 2000, 344 SCRA 238 Gonzaga – Reyes, J. FACTS: Respondents stated that sometime in August 1970 and months thereafter they 42

received mimeographed notices dated August 2, 1970 and signed by the late Ramon Durano, Sr., informing them that the lands which they are tilling and residing in, formerly owned by the Cebu Portland Cement Company (hereafter, “Cepoc”), had been purchased by Durano & Co., Inc. The notices also declared that the lands were needed by Durano & Co. for planting to sugar and for roads or residences, and directed respondents to immediately turn over the said lands to the representatives of the company. Simultaneously, tall bamboo poles with pennants at the tops thereof were planted in some areas of the lands and metal sheets bearing the initials “RMD” were nailed to posts. As early as the first week of August 1970, and even before many of the respondents received notices to vacate, men who identified themselves as employees of Durano & Co. proceeded to bulldoze the lands occupied by various respondents, destroying in their wake the plantings and improvements made by the respondents therein. On September 15, 1970, Durano & Co. sold the disputed property to petitioner Ramon Durano III, who procured the registration of these lands in his name under TCT No. T103 and TCT No. T-104. Respondents contended that the display of force and the known power and prestige of petitioners and their family restrained them from directly resisting this wanton depredation upon their property. Respondents urged the Department of Justice to conduct the preliminary investigation. The RTC found that the case preponderated in favor of respondents, who all possessed their respective portions of the property covered by TCT Nos. T-103 and T-104 thinking that they were the absolute owners thereof. A number of these respondents alleged that they inherited these properties from their parents, who in turn inherited them from their own parents. Some others came into the properties by purchase from the former occupants thereof. They and their predecessors were responsible for the plantings and improvements on the property. They were the ones who sought for the properties to be tax-declared in their respective names, and they continually paid the taxes thereto. Respondents maintained that they were unaware of anyone claiming adverse possession or ownership of these lands until the bulldozing operations in 1970. Dissatisfied, petitioners appealed the RTC decision to the Court of Appeals, which, in turn, affirmed the said decision and ordered the return of the property to all the respondents-claimants. ISSUE: Whether or not the Court of Appeals erred in its decision ordering the petitioners to return the properties to the respondents. HELD: No. The evidence shows that respondents successfully complied with all the requirements for acquisitive prescription to set in. The properties were conveyed to respondents by purchase or inheritance, and in each case the respondents were in actual, continuous, open and adverse possession of the properties. They exercised rights of ownership over the lands, including the regular payment of taxes and introduction of plantings and improvements. They were unaware of anyone claiming to 43

be the owner of these lands other than themselves until the notices of demolition in 1970 --- and at the time each of them had already completed the ten-year prescriptive period either by their own possession or by obtaining from the possession of their predecessors-in-interest. Furthermore, a purchaser of a parcel of land cannot close his eyes to facts which should put a reasonable man upon his guard, such as when the property subject of the purchase is in the possession of persons other than the seller. A buyer who could not have failed to know or discover that the land sold to him was in the adverse possession of another is a buyer in bad faith. In the case, respondents were in open possession and occupancy of the properties when Durano & Co. supposedly purchased the same from Cepoc. Petitioners made no attempt to investigate the nature of respondents’ possession before they ordered demolition in August 1970. In the same manner, the purchase of the property by petitioner Ramon Durano III from Durano & Co. could not be said to have been in good faith. It is not disputed that Durano III acquired the property with full knowledge of respondents’ occupancy thereon. There even appears to be undue haste in the conveyance of the property to Durano III, as the bulldozing operations by Durano & Co. were still underway when the deed of sale to Durano III was executed on September 15, 1970. There is not even an indication that Durano & Co. attempted to transfer registration of the property in its name before it conveyed the same to Durano III. Since petitioners knew fully well the defect in their titles, they were correctly held by the Court of Appeals to be builders in bad faith. The Civil Code provides: Art. 449. He who builds, plants or sows in bad faith on the land of another, loses what is built, planted or sown without right of indemnity. Art. 450. The owner of the land on which anything has been built, planted or sown in bad faith may demand the demolition of the work, or that the planting or sowing be removed, in order to replace things in their former condition at the expense of the person who built, planted or sowed; or he may compel the builder or planter to pay the price of the land, and the sower the proper rent. Art. 451. In the cases of the two preceding articles, the landowner is entitled to damages from the builder, planter or sower. Based on these provisions, the owner of the land has three alternative rights: (1) to appropriate what has been built without any obligation to pay indemnity therefor, or (2) to demand that the builder remove what he had built, or (3) to compel the builder to pay the value of the land. In any case, the landowner is entitled to damages under Article 451.

44

The Court sustains the return of the properties to respondents and the payment of indemnity as being in accord with the reliefs under the Civil Code.

Ballatan v. Court of Appeals G.R. No. 125683, March 2, 1999, 304 SCRA 34 Puno, J. FACTS: Ballatan, Martinez and Ling are the owners of adjacent lots in Malabon, Metro Manila. Lot No. 24, 414 square meters in area, is registered in the name of petitioners Eden Ballatan and spouses Betty Martinez and Chong Chy Ling. Lots Nos. 25 and 26 are registered in the name of respondent Gonzalo Go, Sr. On Lot No. 25, respondent Winston Go, son of Gonzalo Go, Sr., constructed his house. Adjacent to Lot No. 26 is Lot No. 27, registered in the name of respondent Li Ching Yao. In 1985, petitioner Ballatan constructed her house on Lot No. 24. During the construction, she noticed that the concrete fence and side pathway of the adjoining house of respondent Winston Go encroached on the entire length of the eastern side of her property. Her building contractor informed her that the area of her lot was actually less than that described in the title. Forthwith, Ballatan informed respondent Go of this discrepancy and his encroachment on her property. Go, however, claimed that his house, including its fence and pathway, were built within the parameters of his father's lot; and that this lot was surveyed by Engineer Jose Quedding, the authorized surveyor of the Araneta Institute of Agriculture (AIA), the owner-developer of the subdivision project. So Ballatan called the attention of the IAI and after another survey of the land, Engineer Quedding found that the lot area of petitioner Ballatan was less by few meters and that of respondent Li Ching Yao, which was three lots away, increased by two meters. Engineer Quedding declared that he made a verification survey of Lots Nos. 25 and 26 of respondents Go in 1983 and allegedly found the boundaries to have been in their proper position. He, however, could not explain the reduction in Ballatan's area since he was not present at the time respondents Go constructed their boundary walls. On 10 June 1985, petitioner Ballatan made a written demand on respondents Go to remove and dismantle their improvements on Lot No. 24 but Go refused. So Ballatan instituted against Go a civil case for recovery of possession the RTC of Malabon decided in favor of Ballatan, ordering the Go's to vacate the subject portion of Lot No. 24, demolish their improvements and pay petitioner Ballatan actual damages, attorney's fees and the costs of the suit. Go appealed. ISSUE: Whether or not Ballatan have a right of remotion. HELD: All the parties have acted in good faith so Article 448 must apply. Petitioners are ordered to exercise within thirty (30) days from finality of the decision their option to either buy the portion of respondents Go's improvement on their Lot No. 24, or sell to said respondents the portion of their land on which the improvement stands. If petitioners elect to sell the land or buy the improvement, the purchase price must be at 45

the prevailing market price at the time of payment. If buying the improvement will render respondents Go's house useless, then petitioners should sell the encroached portion of their land to respondents Go. If petitioners choose to sell the land but respondents Go are unwilling or unable to buy, then the latter must vacate the subject portion and pay reasonable rent from the time petitioners made their choice up to the time they actually vacate the premises. But if the value of the land is considerably more than the value of the improvement, then respondents Go may elect to lease the land, in which case the parties shall agree upon the terms, the lease. Should they fail to agree on said terms, the court of origin is directed to fix the terms of the lease.

46

Spouses Del Ocampo v. Abesia G.R. No. L-49219, April 15, 1998, 160 SCRA 379 Gancayco, J. FACTS: Plaintiffs – spouses Concepcion Fernandez and Estanislao Del Campo and defendant Bernarda Fernandez Abesia are co-owners of parcel of land with an area of 45 square meters and divided in the proportion of 2/3 and 1/3 share each, respectively. A commissioner, who is appointed by the court, conducted a survey and recommended that the property be divided into two lots: Lot 1161 – A with an area of 30 square meters for the plaintiffs and Lot 1161 – B with an area of 15 square meters for the defendants. However, it was shown in the sketch plan that the house of the defendant occupied the portion with an area of 5 square meters of Lot 1161 – A of plaintiffs. The parties asked the court to finally settle and adjudicate who among the parties should take possession of the 5 square meters of land. ISSUES: 1.) Whether or not Article 448 of the Civil Code, the rights of a builder in good faith, should be applied to the plaintiff-spouses Del Campo. 2.) Whether or not the house of the defendant Abesia should be removed and demolished at their expense. HELD: 1.) Yes. Article 448 of the Civil Code cannot apply where a co-owner builds, plants or sows on the land owned in common for then, he did not build, plant or sow upon land that exclusively belongs to another but of which he is a co-owner. The coowner is not a third person under the circumstances, and the situation is governed by the rules of co-ownership. However, when, as in this case, the co-ownership is terminated by the partition and it appears that the house of defendants overlaps or occupies a portion of 5 square meters of the land pertaining to plaintiffs which the defendants obviously built in good faith, then the provisions of Article 448 of the new Civil Code should apply. Manresa and Navarro Amandi agree that the said provision of the Civil Code may apply even when there was co-ownership if good faith has been established. 2.) It depends. Applying Article 448 of the Civil Code, the plaintiffs have the right to appropriate said portion of the house of defendants upon payment of indemnity to defendants as provided for in Article 546 of the Civil Code. Otherwise, the plaintiffs may oblige the defendants to pay the price of the land occupied by their house. However, if the price asked for is considerably much more than the value of the portion of the house of defendants built thereon, then the latter cannot be obliged to buy the land. The defendant shall then pay the reasonable rent to the plaintiffs upon such terms and conditions that they may agree. In case of disagreement, the trial court shall fix the terms thereof. Of course, defendants may demolish or remove the said portion of their house, at their own expense, if they so decide.

47

Pacific Farms Inc. v. Esguerra G.R. No. L-21783, November 29, 1969, 30 SCRA 684 Castro, J. FACTS: On October 1, 1956 to March 2, 1957 the Company sold and delivered lumber and construction materials to the Insular Farms Inc. which the latter used in the construction of the si buildings at its compound in Bolinao, Pangasinan, of the total procurement price of P15,000.00, the sum of P4,710.18 has not been paid. Consequently, the Company instituted a civil case to recover the unpaid balance and the court sustained their claim. The defendant sheriff levied th six buildings. The Pacific Farms, Inc. filed a suit against the Company and the sheriff asserting ownership over the levied buildings which it had acquired from the Insular Farms by virtue of absolute sale executed on March 21, 1958. Pacific prays that the judicial sale of the six buildings be declared null and void. The trial court rendered judgment annulling the levy and the certificate of sale. However, it denied the plaintiff's claim for actual and exemplary damages on the ground that it was not "prepared to find there was gross negligence or bad faith on the part of any defendants". ISSUE: Whether or not the application by analogy of the rules of accession would suffice for a just adjudication. HELD: Article 447 of the Civil Code contemplates a principal and an accessory; the land being considered the principal, and the plantings, constructions or works, the accessory. The owner of the land who in good faith - whether personally or through another makes constructions or works thereon, using materials belonging to somebody else, becomes the owner of the said materials with the obligation however of paying for their value. On the other hand, the owner of the materials is entitled to remove them, provided no substantial injury is caused to the landowner. Otherwise, he has the right to reimbursement for the value of his materials, Applying article 447 by analogy, the Court consider the buildings as the principal and the lumber and construction materials that went into their construction as the accessory. Thus the appellee, if it does own the six buildings, must bear the obligation to pay for the values of the said materials; the appellant — which apparently has no desire to remove the materials, and, even if it were minded to do so, cannot remove them without necessarily damaging the buildings — has the corresponding right to recover the value of the unpaid lumber and construction materials.

Pecson v. Court of Appeals G.R. No. 115814, May 26, 1995, 244 SCRA 407 Davide, Jr. J. FACTS: Pedro Pecson owned a commercial lot situated in Kamias street, Quezon City, on which he built a a four-door, two-storey apartment building. But because of failure to 48

pay realty taxes amounting to P12,000.00, the commercial lot owned was sold at a public auction. It was purchased by Nepomuceno, which later sold the same to the Nuguid spouses for P103,000 on October 12, 1983. Pecson then challenged the sale, alleging that the apartment building, contrary to the claim of the Nuguid spouses, was not included in the sale. The lower court judged in favor of Pecson, declaring that the apartment building was indeed not included in the subject sale. The Court of Appeals affirmed the same. The Spouses Nuguid then filed a motion for delivery of possession of the lot and the apartment building. The lower court ruled in favor of the private respondents, but subject to the reimbursement to Pecson of the cost of constructing the apartment building minus the rents due to the spouses (calculated at P21,000 from June 23, 1993 to September 23, 1993). With the said decision at hand, the spouses then made a move to eject Pecson and as well as the tenants residing therein. However, the spouses have yet to pay Pecson for the construction costs. ISSUE: Whether the Nuguid Spouses can eject Pecson even if reimbursement hasn’t been given for the construction costs. HELD: No. The Court ruled that since the spouses still haven’t reimbursed Pecson for the cost of construction of the building, the latter has the right to retain the property, and along with it, the fruits of which during such possession. The court ruled that though Article 448 do not apply in the case at bar. By its clear language, Article 448 refers to a land whose ownership is claimed by two or more parties, one of whom has built some works, or sown or planted something. The building, sowing or planting may have been made in good faith or in bad faith. As in this case, since the owner himself was the one who constructed the improvement, good faith and bad faith becomes irrelevant. However, by analogy, the indemnity may be applied, considering that the primary intent of Article 448 is to avoid a state of forced coownership and that the parties agree that Articles 448 and 546 of the Civil Code are applicable and indemnity for the improvements may be paid, although they differ as to the basis of the indemnity. Since the spouses have opted to appropriate the apartment building, Pecson is thus entitled to the possession and enjoyment of the apartment building, until he is paid the proper indemnity, as well as of the portion of the lot where the building has been constructed. This is so because the right to retain the improvements while the corresponding indemnity is not paid implies the tenancy or possession in fact of the land on which it is built, planted or sown. The petitioner not having been so paid, he was entitled to retain ownership of the building and, necessarily, the income therefrom.

49

Technogas Philippines Manufacturing Corporation v. Court of Appeals G.R. No. 108894, February, 10, 1997, 268 SCRA 5 Panganiban, J. FACTS: Technogas purchased a parcel of land from Pariz Industries, Inc. In the same year, Eduardo Uy purchased the land adjacent to it. The following year, Uy bought another lot adjoining the lot of Technogas. Portions of the buildings and wall bought by Technogas together with the land from Pariz Industries are occupying a portion of Uy’s adjoining land. The knowledge of some encroachment was only made known to both parties after their parties of their respective parcels of land. ISSUES: 1.) Whether or not petitioner Technogas Philippines is a possessor in bad faith. 2.) Whether or not petitioner Technogas Philippines has stepped into the shoes of the seller. HELD: 1.) No. Unless one is versed in the science of surveying, no one can determine the precise extent or location of his property by merely examining his paper title. There is no question in that when Technogas purchased the land from Pariz Industries, the buildings and other structures were already in existence. Furthermore, it is not clear as to who actually built these structures but it can be assumed that the predecessor-ininterest of Technogas, Pariz Industries, did so. An article 527 of the New Civil Code presumes good faith. Since no proof exists to show that the builder built the encroaching structures in bad faith, the structures should be presumed to have been built in good faith. Good faith consists in the belief of the builder that the land he is building on is his, and his ignorance of any defect or flaw in his title. Furthermore, possession acquired in good faith does not lose this character except in case and from the moment facts exist which show that the possessor is not aware that he possesses the thing improperly or wrongfully. The good faith ceases from the moment the defects in the title are made known to the possessor, by extraneous evidence or by suit for recovery of the property of the true owner. 2.) Yes. Has been shown, contrary as to the good faith of Technogas has not been overthrown. Similarly, upon delivery of the property to Pariz Industries, as seller, to Technogas, as buyer, the latter acquired ownership of the property. Consequently, Technogas is deemed to have stepped into the shoes of the seller with regard to all the rights of ownership of the property over the immovable sold, including the right to compel Uy to exercise either of the two options under Article 448 of the New Civil Code. Thus, the landowner’s exercise of his option can only take place after the builder shall have to know the intrusion – in short, when both parties shall have become aware of it. Only then will the occasion for exercising the option arise, for it is only then that both parties will have been aware that a problem exists with regard to their property rights.

50

Pleasantville Development Corporation v. Court of Appeals G.R. No. 79688, February 1, 1996, Panganiban, J. FACTS: On March 26, 1974, Wilson Kee on installment Lot 8 from C.T. Torres Enterprises Inc. the exclusive real estate agent of petitioner. Under the Contract to Sell on installment. Kee can exercise possession over the parcel of land even before the completion of installment payments. On January 20, 1975, Kee paid CTTEI relocation fee of Php 50.00 and another on January 27, 1975 for the preparation of lot plan. These amounts were paid by Kee before he took possession of Lot 8. After the preparation of the lot plan and a copy was presented to Kee, Zenaida Octaviano, employee of CTTEI accompanied Donnabelle Kee the wife of Wilson Kee to inspect Lot 8. Unfortuantely, Octaviano pointed Lot 9. Thereafter, Kee constructed his residence on the said Lot 9 together a store, repair shop and other improvements. Edith Robillo purchased from Pleasantville Development Corporation Lot 9. Sometime in 1975, she sold the said parcel of land, Lot 9, to Eldred Jardinico which at that time is vacant. Upon paying completely to Robillo, Jardinico secured from the Register of Deeds of Bacolod City on December 19, 1978 Transfer Certificate of Title No. 106367 in his name. It was only that time that he discovered that Wilson Kee take possession of that lot and that the same have introduced improvements to the same lot. Jardinico confronted Kee and tried to reach for an amicable settlement, but failed. On January 30, 1981, Jardinico, through his lawyer, demanded that Kee vacate Lot 9 and remove all the improvements introduced by the latter. Kee refused which made Jardinico filed with the Municipal Trial Court in Cities, Branch 3, Bacolod City a complaint for ejectment with damages against Kee. Kee, in turn filed a third-party complaint against Pleasantville Development Corporation and CTTEI. The MTCC held that the erroneous delivery was attributable to CTTEI and the Kee has no rights to Lot 9 because of the rescission made by CTTEI of their contract due to Kee’s failure to pay the installment. MTCC also held that Kee must pay reasonable rental for the use of Lot 9 and furthermore he cannot claim reimbursement for the improvements introduced by him. On appeal, the Regional Trial Court held that Pleasantville and CTTEI were not negligent and that Kee was in bad faith. Kee appealed directly to the Supreme Court which referred the matter to the Court of Appeals. The Appellate Court overturned the ruling of the RTC and held the Kee was a builder in good faith and the erroneous delivery was attributable to the negligence of CTTEI. Hence the instant petition filed by Pleasantville. ISSUES: 1.) Whether or not, Wilson Kee is a builder in good faith. 2.) Whether or not petitioner is liable for the acts of its agent CTTEI. HELD: 1.) Petitioner fails to persuade the Court to abandon the findings and 51

conclusions of the Court of Appeals that Kee was a builder in good faith. Good faith consists in the belief of the builder that the land he is building on is his and his ignorance of any defect or flaw in his title. And as good faith is presumed, petitioner has the burden of proving bad faith on the part of Kee. At the time he built improvements on Lot 8, Kee believed that said lot was what he bought from petitioner. He was not aware that the lot delivered to him was not Lot 8. Thus, Kee is in good faith. Petitioner failed to prove otherwise. To demonstrate Kee's bad faith, petitioner points to Kee's violation of paragraphs 22 and 26 of the Contract of Sale on Installment. It has no merit. Such violations have no bearing whatsoever on whether Kee was a builder in good faith, that is, on his state of mind at the time he built the improvements on Lot 9. These alleged violations may give rise to petitioner's cause of action against Kee under the said contract (contractual breach), but may not be the basis to negate the presumption that Kee was a builder in good faith. 2.) Yes. The rule is that the principal is responsible for the acts of the agent done within the scope of his authority, and should bear the damage caused to third persons. On the other hand, the agent who exceeds his authority is personally liable for the damage. But CTTEI was acting within its authority as the sole real estate representative of petitioner when it made the delivery to Kee, only that in so acting, it was negligent. It is this negligence that is the basis of petitioner's liability, as principal of CTTEI, per Articles 1909 and 1910 of the Civil Code. For such negligence, the petitioner should be held liable for damages. The rights of Kee and Jardinico vis-a-vis each other, as builder in good faith and owner in good faith, respectively, are regulated by law (i.e., Arts. 448, 546 and 548 of the Civil Code). It was error for the Court of Appeals to make a "slight modification" in the application of such law [by holding petitioner and CTTEI solidarily liable], on the ground of "equity".

Germiniano v. Court of Appeals G.R. No. 120303, July 24, 1996, 259 SCRA 344 Davide, Jr., J. FACTS: This is a petition for review on certiorari which has its origins in Civil Case No. 9214 of Branch 3 of the Municipal Trial Court in Cities (MTCC) in Dagupan City for unlawful detainer and damages. During the pre-trial conference, the parties agreed to confine the issues to: (1) whether there was an implied renewal of the lease which expired in November 1985; (2) whether the lessees were builders in good faith and entitled to reimbursement of the value of the house and improvements; and (3) the value of the house. On the first issue, the court held that since the petitioners' mother was no longer the owner of the lot in question at the time the lease contract was executed in 1978, in view of its acquisition by Maria Lee as early as 1972, there was no lease to speak of, much 52

less, a renewal thereof. And even if the lease legally existed, its implied renewal was not for the period stipulated in the original contract, but only on a month-to-month basis pursuant to Article 1687 of the Civil Code. The refusal of the petitioners' mother to accept the rentals starting January 1986 was then a clear indication of her desire to terminate the monthly lease. As regard the petitioners' alleged failed promise to sell to the private respondents the lot occupied by the house, the court held that such should be litigated in a proper case before the proper forum, not an ejectment case where the only issue was physical possession of the property. The court resolved the second issue in the negative, holding that Articles 448 and 546 of the Civil Code, which allow possessors in good faith to recover the value of improvements and retain the premises until reimbursed, did not apply to lessees like the private respondents, because the latter knew that their occupation of the premises would continue only during the life of the lease. Besides, the rights of the private respondents were specifically governed by Article 1678, which allow reimbursement of up to one-half of the value of the useful improvements, or removal of the improvements should the lessor refuse to reimburse. On the third issue, the court deemed as conclusive the private respondents' allegation that the value of the house and improvements was P180,000.00, there being no controverting evidence presented. On appeal by the private respondents, the RTC of Dagupan City reversed the trial court's decision. ISSUE: Whether or not Article 448 or Article 1678 of the Civil Code should apply in the instant case. HELD: In this case, both parties admit that the land in question was originally owned by the petitioners' mother. The land was allegedly acquired later by one Maria Lee by virtue of an extrajudicial foreclosure of mortgage. Lee, however, never sought a writ of possession in order that she gain possession of the property in question. The petitioners' mother therefore remained in possession of the lot. It has been said that while the right to let property is an incident of title and possession, a person may be lessor and occupy the position of a landlord to the tenant although he is not the owner of the premises let. There is no need to apply by analogy the provisions of Article 448 on indemnity as was done in Pecson vs. Court of Appeals, because the situation sought to be avoided and which would justify the application of that provision, is not present in this case. Suffice it to say, "a state of forced co-ownership" would not be created between the petitioners and the private respondents. For, as correctly pointed out by the petitioners, the right of the private respondents as lessees is governed by Article 1678 of the Civil Code which allows reimbursement to the extent of one-half of the value of the useful improvements. It must be stressed, however, that the right to indemnity under Article 1678 of the Civil Code arises only if the lessor opts to appropriate the improvements. Since the 53

petitioners refused to exercise that option the private respondents cannot compel them to reimburse the one-half value of the house and improvements. Neither can they retain the premises until reimbursement is made. The private respondents' sole right then is to remove the improvements without causing any more impairment upon the property leased than is necessary.

54

Agustin v. Intermediate Appellate Court G.R. No. 66075-76, July 5, 1990, 187 SCRA 218 Grino – Aquino, J. FACTS: The Cagayan River separates the towns of Solana on the west and Tuguegarao on the east in the province of Cagayan. In 1919 the lands of the east of the river were covered by the Tuguegarao Cadastre. In 1925, OCT 5472 was issued for land east of the Cagayan River owned by Eulogio Agustin. As the years went by, the Cagayan River moved gradually eastward, depositing silt on the west bank. The shifting of the river and siltation continued until 1968. In 1950, all lands west of the river were included in the Solana Cadastre. Among these occupying lands covered by Solana Cadastre were Pablo Binayug and Maria Melad. Through the years, the Cagayan River eroded lands of the Tuguegarao Cadastre on its eastern bank among which was Agustin’s Lot 8457, depositing the alluvium as accretion on the land possessed by Binayug on the western bank. However, 1968, after a big flood, the Cagayan River changed its course, returned to its 1919 bed and in the process, cut across the lands of Maria Melad, Timoteo Melad, and the spouses Pablo Binayug and Geronima Ubina whose lands were transferred on the eastern, or Tuguegarao, side of the river. To cultivate those lots they had to cross the river. In April 1969, while the Melads, Binayug, Urbina and their tenants were planting corn on their lots located on the easter side of Cagayan River, Agustin, the heirs of Baldomero Langcay, Juan Langcay, and Arturo Balisi, accompanied by the mayor and some policemen of Tuguegarao, claimed the same lands as their own and drove away the Melads, Binayug and Urbina from the premises. ISSUE: Whether or not ownership of accretion is lost upon sudden and abrupt change of the river. HELD: No. The ownership of the accretion to the lands was not lost upon sudden and abrupt change of the course of the river (Cagayan River in 1968 or 1969 when it reverted to its old 1919 bed), and separated or transferred said accretions to the other side (eastern bank) of the river. Articles 459 and 463 of the New Civil Code apply to this situation. Article 459 provides that “whenever the current of a river, creek or torrent segregates from an estate on its bank a known portion of land and transfer it to another estate, the owner of the land to which the segregated portion belonged retains the ownership of it, provided that he removes the same within two years.” Article 463 provides that,” whenever the current of a river divides itself into branches, leaving a piece of land or part thereof isolated, the owner of the land retains his ownership. He also retains it if a portion of land is separated from the estate by the current.”

Cureg v. Intermediate Appellate Court G.R. No. 73465, September 7, 1989, 177 SCRA 313 Medialdea, J. 55

FACTS: On November 5, 1982, private respondents Domingo Apostol et al. filed a complaint for quieting of title against petitioners Leonida Cureg et al. The complaint alleged that private respondents, except Apostol, are the legal and/or the forced heirs of the late Domingo Gerardo, and his predecessors-in-interest have been in actual, open, peaceful and continuous possession, under a bona fide claim of ownership of a parcel of land (referred to as their “motherland”). Subsequently, the heirs verbally sold the “motherland” to Apostol. The “motherland” showed signs of accretion caused by the movement of the Cagayan River. When private respondents were about to cultivate their “motherland” together with its accretion, they were prevented by the petitioners. Petitioners alleged that the “motherland” claimed by the private respondents is nonexistent, that the “subject land” is an accretion to their registered land, and that petitioners have been in possession and cultivation of the “accretion” for many years now. ISSUE: Whether or not the petitioners have the better right of accretion. HELD: Yes. The petitioners are entitled to the accretion. The “subject land” is an alluvial deposit left by the northward movement of the Cagayan River and pursuant to Article 457 of the New Civil Code: “To the owners of land adjoining the banks of river belong the accretion which they gradually receive from the effects of the current of the waters.” However, the increase in the area of the petitioner’s land, being an accretion left by the change of course or the northward movement of the Cagayan River does not automatically become registered land just because the lot which receives such accretion is covered by a Torrens title. As such, it must also be placed under the operation of the Torrens system.

Viajar v. Court of Appeals G.R. No. 77294, December 12, 1988, 168 SCRA 405 Medialdea, J. FACTS: The spouses Ricardo and Leonor Ladrido were the owners of Lot 7511. Spouses Rosendo and Ana Te were also the registered owners of a parcel of land described in their title as Lot 7340 of the Cadastral Survey of Pototan. On 6 September 1973, Rosendo Te, with the conformity of his wife, sold this lot to Angelica F. Viajar and Celso F. Viajar for P5,000. A Torrens title was later issued in the latter’s names. Later, Angelica Viajar had Lot 7340 relocated and found out that the property was in the possession of Ricardo Y. Ladrido. Consequently, she demanded its return but Ladrido refused. The piece of real property which used to be Lot 7340 of the Cadastral Survey of Pototan was located in barangay Guibuanogan, Pototan, Iloilo; that at the time of the cadastral survey in 1926, Lot 7511 and Lot 7340 were separated by the Suague River; that Lot 7340 has been in the possession of Ladrido; that the area of 14,036 sq.ms., which was formerly the river bed of the Suague River per cadastral survey of 1926, has also been in the possession of Ladrido; and that the Viajars have never been in actual 56

physical possession of Lot 7340. On 15 February 1974, Angelica and Celso Viajar instituted a civil action for recovery of possession and damages against Ricardo Y. Ladrido. The trial court rendered its decision in favor of Ladrido, dismissing the complaint of Angelica and Celso Viajar with costs against them, declaring the Ladridos are entitled to the possession thereof. Not satisfied with the decision, the Viajars appealed to the Court of Appeals. The Court of Appeals affirmed the decision of the court. The Viajars filed a petition for review on certiorari. ISSUE: Whether the respondents are entitled to the land on the ground of accretion. HELD: Article 457 of the New Civil Code provides that “to the owners of lands adjoining the banks of rivers belong the accretion which they gradually receive from the effects of the current of the waters." The presumption is that the change in the course of the river was gradual and caused by accretion and erosion. In the present case, the lower court correctly found that the evidence introduced by the Viajars to show that the change in the course of the Suague River was sudden or that it occurred through avulsion is not clear and convincing. The Ladridos have sufficiently established that for many years after 1926 a gradual accretion on the eastern side of Lot 7511 took place by action of the current of the Suague River so that in 1979 an alluvial deposit of 29,912 sq.ms. more or less, had been added to Lot 7511. The established facts indicate that the eastern boundary of Lot 7511 was the Suague River based on the cadastral plan. For a period of more than 40 years (before 1940 to 1980) the Suague River overflowed its banks yearly and the property of the defendant gradually received deposits of soil from the effects of the current of the river. The consequent increase in the area of Lot 7511 due to alluvion or accretion was possessed by the defendants whose tenants plowed and planted the same with corn and tobacco. The quondam river bed had been filled by accretion through the years. The land is already plain and there is no indication on the ground of any abandoned river bed. Under the law, accretion which the banks or rivers may gradually receive from the effects of the current of the waters becomes the property of the owners of the lands adjoining the banks. Therefore, the accretion to Lot 7511 which consists of Lots A and B belong to the Ladridos.

Vda. De Nazareno v. Court of Appeals G.R. No. 98045, June 26, 1996, 257 SCRA 589 Romeo, J. FACTS: The subject of this controversy is a parcel of land formed as a result of sawdust dumped into the dried-up Balacanas Creek and along the banks of the Cagayan river. Private respondents Salasalan and Rabaya leased the subject lots on which their houses stood from Antonio Nazareno, petitioners’ predessor-in-interest. Private respondents allegedly stopped paying rentals. As a result, Nazareno and petitioners filed a case for ejectment with the MTC of Cagayan de Oro City. The MTC rendered a decision against private respondents which was affirmed by the RTC. After several petitions for annulmentof judgment by private respondents which were all dismissed, the 57

decision of the lower court was finally enforced with the private respondents being ejected from portions of the subject lots they occupied. Before Nazareno died, he caused the approval by the Bureau of lands of the survey plan with a view to perfecting his title over the accretion area being claimed by him. The said petition was protested by private respondents. After conducting a survey of the subject land, land investigator Avelino labis recommended that the survey plan be cancelled and that private respondents be directed to file appropriate public land application covering their respective portions. Nazareno filed a motion for reconsideration with the Undersecretary of the Department of Natural Resources and OIC of the Bureau of lands Ignacio who denied the Motion. Respondent Director of lands Abelardo Palad ordered Nazareno to vacate the portions adjudicated to private respondents and remove whatever improvements they have introduced; he also ordered that private respondents be placed in possession thereof. A petitioner filed a case for annulment of the previous decisions with the RTC but was dismissed. The CA affirmed the RTC decision contending that the approved of the survey plan belongs exclusively to the Director of lands and the same shall be conclusive when approved by the Secretary of Agriculture and Natural Resources. ISSUE: Whether or not petitioners can claim ownership of the subject land by virtue of Art 457 of the Civil Code. HELD: No, accretion as a mode of acquiring property under Art 457 of the NCC requires the concurrence of the requisites mentioned in the Article. These are called rules on alluvion, which if present in a case, give to the owners of lands adjoining the banks of rivers or streams any accretion gradually received from the effects of the current of waters. The word “current” indicates the participation of the body of water in the flow of waters due to high and low tide. Petitioners, however, admit that the accretion was formed by the dumping of boulders, soil and other filling materials on portions of the Balacanas creek and the Cagayan River. The Bureau of lands classified the subject land as an accretion area which was formed by deposits of sawdust. Petitioner’s submission not having met the first and second requirements of the rules of alluvion, they cannot claim the rights of a riparian owner. The subject being public land is under the jurisdiction of the Bureau of lands, respondent Palad is authorized to exercise executive control over any form of concession, disposition and management of the lands of public dominion.

Heirs of Navarro v. Intermediate Appellate Court G.R. No. 68166, February 12, 1997, 268 SCRA 589 Hermosisima, J: FACTS: On October 3, 1946, Sinforoso Pascual, filed an application for foreshore lease covering a tract of foreshore land in Sibocon, Balanga, Bataan, having an area of approximately seventeen (17) hectares. Subsequently, petitioners' predecessor-ininterest, Emiliano Navarro, filed a fishpond application with the Bureau of Fisheries 58

covering twenty five (25) hectares of foreshore land also in Sibocon, Balanga, Bataan. Initially, such application was denied by the Director of Fisheries on the ground that the property formed part of the public domain. Sometime in the early part of 1960, Sinforoso Pascual flied an application to register and confirm his title to a parcel of land, situated in Sibocon, Balanga, Bataan, described in Plan Psu-175181 and said to have an area of 146,611 square meters. Pascual claimed that this land is an accretion to his property, situated in Barrio Puerto Rivas, Balanga, Bataan, and covered by Original Certificate of Title No. 6830. It is bounded on the eastern side by the Talisay River, on the western side by the Bulacan River, and on the northern side by the Manila Bay. The Talisay River as well as the Bulacan River flow downstream and meet at the Manila Bay thereby depositing sand and silt on Pascual's property resulting in an accretion thereon. Sinforoso Pascual claimed the accretion as the riparian owner. On March 25, 1960, the Director of Lands, represented by the Assistant Solicitor General, filed an opposition thereto stating that neither Pascual nor his predecessors-ininterest possessed sufficient title to the subject property, the same being a portion of the public domain and, therefore, it belongs to the Republic of the Philippines. ISSUE: Whether or not the land sought to be registered is accretion or foreshore land, or, whether or not said land was formed by the action of the two rivers of Talisay and Bulacan or by the action of the Manila Bay. HELD: Accretion as a mode of acquiring property under said Article 457, requires the concurrence of the following requisites: (1) that the accumulation of soil or sediment be gradual and imperceptible; (2) that it be the result of the action of the waters of the river; and (3) that the land where the accretion takes place is adjacent to the bank of the river. If the accretion were to be attributed to the action of either or both of the Talisay and Bulacan Rivers, the alluvium should have been deposited on either or both of the eastern and western boundaries of petitioners' own tract of land, not on the northern portion thereof which is adjacent to the Manila Bay. Clearly lacking, thus, is the third requisite of accretion, which is, that the alluvium is deposited on the portion of claimant's land which is adjacent to the river bank. The disputed land, thus, is an accretion not on a river bank but on a sea bank, or on what used to be the foreshore of Manila Bay which adjoined petitioners' own tract of land on the northern side. Applicant Pascual has not presented proofs to convince the Court that the land he has applied for registration is the result of the settling down on his registered land of soil, earth or other deposits so as to be rightfully be considered as an accretion [caused by the action of the two rivers]. Article 457 finds no applicability where the accretion must have been caused by action of the bay. The conclusion formed by the trial court on the basis of the aforegoing observation is that the disputed land is part of the foreshore of Manila Bay and therefore, part of the public domain. Thus, the disputed property is an accretion on a sea bank, Manila Bay 59

being an inlet or an arm of the sea; as such, the disputed property is, under Article 4 of the Spanish Law of Waters of 1866, part of the public domain.

60

Del Banco v. Intermediate Appellate Court G.R. No. 72694, December 1, 1987, 156 SCRA 55 Paras, J. FACTS: In a document executed in the Municipality of San Rafael, Bulacan, on February 11, 1859, three brothers, Benedicto Pansacola, Jose Pansacola and Manuel Pansacola (known as Fr. Manuel Pena) entered into an agreement which provided, among others: (1) That they will purchase from the Spanish Government the lands comprising the Island of Cagbalite which is located within the boundaries of the Municipality of Mauban, Province of Tayabas (now Quezon) and has an approximate area of 1,600 hectares; (2) That the lands shall be considered after the purchase as their common property; (3) That the co-ownership includes Domingo Arce and Baldomera Angulo, minors at that time represented by their father, Manuel Pansacola (Fr. Manuel Pena) who will contribute for them in the proposed purchase of the Cagbalite Island; (4) That whatever benefits may be derived from the Island shall be shared equally by the co-owners in the following proportion: Benedicto Pansacola-1/4 share; Jose Pansacola-1/4 share; and, Domingo Arce and Baldomera Angulo-2/4 shares which shall be placed under the care of their father, Manuel Pansacola (Fr. Manuel Pena). On August 14, 1866, co-owners entered into the actual possession and enjoyment of the Island purchased by them from the Spanish Government. On April 11, 1868 they agreed to modify the terms and conditions of the agreement entered into by them on February 11, 1859. About one hundred years later, on November 18, 1968, private respondents brought a special action for partition in the Court of First Instance of Quezon, under the provisions of Rule 69 of the Rules of Court, including as parties the heirs and successors-ininterest of the co-owners of the Cagbalite Island in the second contract of co-ownership dated April 11, 1968. In their answer some of the defendants, petitioners herein, interposed such defenses as prescription, res judicata, exclusive ownership, estoppel and laches. After trial on the merits, the trial court rendered a decision dated November 6, 1981 dismissing the complaint. The motion for reconsideration filed by the plaintiffs, private respondents herein, was denied by the trial court in an order dated February 25, 1982. On appeal, respondent Court reversed and set aside the decision of the lower court .It also denied the motion for reconsideration and the supplement to motion for reconsideration filed by private respondents, in its resolution dated October 15, 1983. ISSUES: 1.) Whether or not Cagbalite Island is still undivided property owned in common by the heirs and successors-in-interest of the brothers, Benedicto, Jose and Manuel Pansacola. 2.) Whether or not a prescription may run in favor of a co-owner against his co- owners or co-heirs.

61

HELD: 1.) On the first issue, there is nothing in all four agreements that suggests that actual or physical partition of the Island had really been made by either the original owners or their heirs or successors-in-interest. The agreement entered into in 1859 simply provides for the sharing of whatever benefits can be derived from the island. The agreement, in fact, states that the Island to be purchased shall be considered as their common property. In the second agreement entered in 1868 the co-owners agreed not only on the sharing proportion of the benefits derived from the Island but also on the distribution of the Island each of the brothers was allocated a 1/4 portion of the Island with the children of the deceased brother, Eustaquio Pansacola allocated a 1/4 portion and the children of Manuel Pansacola (Fr. Manuel Pena) also allocated a 1/4 portion of the Island. With the distribution agreed upon each of the co-owner is a co-owner of the whole, and in this sense, over the whole he exercises the right of dominion, but he is at the same time the sole owner of a portion, in the instant case, a 1/4 portion (for each group of co-owners) of the Island which is truly abstract, because until physical division is effected such portion is merely an Ideal share, not concretely determined (3 Manresa, Codigo Civil, 3rd Ed., page 486, cited in Lopez vs. Cuaycong, 74 Phil. 601; De la Cruz vs. Cruz, 32 SCRA 307 [1970]; Felices vs. Colegado, 35 SCRA 173 [1970],; Dultra vs. CFl 70 SCRA 465 [1976]; Gatchalian vs. Arlegui, 75 SCRA 234 [1977].) In the agreement of January 20, 1907, the heirs that were represented agreed on how the Island was to be partitioned. The agreement of April 18, 1908 which supplements that of January 20, 1907 reveals that as of the signing of the 1908 agreement no actual partition of the Island had as yet been done. The second and fourth paragraphs of the agreement speaks of a survey yet to be conducted by a certain Amadeo and a plan and description yet to be made. Virgilio Pansacola, a son of the surveyor named Amadeo who is referred to in the contract dated April 18, 1908 as the surveyor to whom the task of surveying Cagbalite Island pursuant to said agreement was entrusted, however, testified that said contracts were never implemented because nobody defrayed the expenses for surveying the same. It is not enough that the co-owners agree to subdivide the property. They must have a subdivision plan drawn in accordance with which they take actual and exclusive possession of their respective portions in the plan and titles issued to each of them accordingly (Caro vs. Court of Appeals, 113 SCRA 10 [1982]). The mechanics of actual partition should follow the procedure laid down in Rule 69 of the Rules of Court. Maganon vs. Montejo, 146 SCRA 282 [1986]). Neither can such actual possession and enjoyment of some portions of the Island by some of the petitioners herein be considered a repudiation of the co-ownership. It is undisputed that the Cagbalite Island was purchased by the original co-owners as a common property and it has not been proven that the Island had been partitioned among them or among their heirs. While there is co-ownership, a co-owner's possession of his share is co-possession which is linked to the possession of the other co-owners (Gatchalian vs. Arlegui, 75 SCRA 234 [1977]).

62

2.) On the second issue, no prescription shall run in favor of a co-owner against his coowners or co-heirs so long as he expressly or impliedly recognizes the co-ownership (Valdez vs. Olonga, 51 SCRA 71 [1973], Tero vs. Tero, 131 SCRA 100 [1984]). Coowners cannot acquire by prescription the share of the other co-owners, absent a clear repudiation of the co-ownership clearly communicated to the other co-owners. An action for partition does not prescribe. Article 403 of the Old Civil Code, now Article 497, provides that the assignees of the co-owners may take part in the partition of the common property, and Article 400 of the Old Code, now Article 494 provides that each co-owner may demand at any time the partition of the common property, a provision which implies that the action to demand partition is imprescriptible or cannot be barred by laches (Budlong vs. Pondoc, 79 SCRA 24 [1977]). An action for partition does not lie except when the co-ownership is properly repudiated by the co- owner.

63

Pardell v. Bartolome G.R. No. L-4656, November 18, 1912, 23 Phil. 450 Torres, J. FACTS: Plaintiff Vicenta Ortiz and defendant Matilde Ortiz are the duly recognized natural daughters of the spouses Miguel and Calixta who died in Vigan, Ilocos Sur. Prior to the death of their mother, she executed a will whereby Matilde and Vicenta became the heirs of all her property. Subsequently, defendants, without judicial authorization or extrajudicial agreement took over the administration and enjoyment of the properties as well as collection of the rents, fruits and products thereof. Moreover, Matilde and her husband occupied the upper storey of the house and the room of the lower floor as an office. With this, Vicenta demanded that she be given rental payments by Matilde in occupying the house since she is a co-owner of the property not occupying the same and as such is entitled to its enjoyment and/or fruits. ISSUE: Whether or not Vicenta can collect rentals from Matilde who occupies and enjoy the property alone as a co-owner. HELD: No. The law grants each co-owner the right to use the property for the purpose intended provided that the interest of the co-ownership must not be injured or prejudiced and the other co-owners must not be prevented from using it according to their rights. Matilde occupied the property owned in common in accordance with the purpose for which it is intended. Records show no proof that she neither occasioned any detriment to the interest of the community property nor prevented her sister from utilizing the said property in accordance to her right as a co-owner thereof. Matilde was excercising her right as a co-owner without being prejudicial to Vicenta who could have also occupied her property had she wanted to. Each co-owner of a property has the right pro-indiviso over the whole property and may use and enjoy the same with no other limitation than that he shall not injure the interests of his co-owners, for the reason that until a division is made, the respective part of each holder of a right as a co-owner cannot be determined and every co-owner exercises joint ownership over the pro-indiviso property in addition to his use and enjoyment of the same.

64

Caro v. Court of Appeals G.R. No. L-46001, March 25, 1982, 113 SCRA 10 Guerrero, J. FACTS: Alfredo Benito, Mario Benito and Benjamin Benito were the original co-owners of two parcels of land somewhere in Sorsogon. Sometime in 1957, Mario died. His wife, Basilia Lahorra and his father, Saturnino Benito, were subsequently appointed as joint administrators of Mario’s estate by the CFI of Sorsogon. On August 26, 1959, Benjamin executed a deed of absolute sale of his one-third undivided portion over said parcels of land in favor of herein petitioner, Luz Caro for the sum of 10,000. Subsequently, with the consent of Saturnino Benito and Alfredo Benito as shown in their affidavits, a subdivision title was issued to petitioner Luz Caro over the lot. Sometime in May 1966, when private respondent Basilia Lahorra learned from a pleading sent to her that petitioner Luz Caro acquired from Benjamin Benito the aforesaid one-third of the undivided share of the subject lands. She sent to petitioner thru counsel, a written offer to redeem the said one-third share. However, this offer was ignored by the petitioner. Hence, private respondent Basilia Lahorra filed a case for legal redemption and sought to prove that as joint administrator of the estate of Mario Benito, she had not been notified of the sale as required by articles 1620 and 1623 of the Civil Code. During the hearing of the case, petitioner presented the following secondary evidence to prove the service of notice of the intended sale to possible redemptioners: (1) affidavit of Benjamin Benito attesting to the fact that the possible redemptioners were formally notified in writing of his intention to sell his undivided share; (2) deposition of Saturnino’s widow that she received and showed the notice to husband but the latter was not interested to buy the property. The trial court ruled in favor of the petitioner. However, the decision was reversed by the CA. Hence, the case was brought to the SC. ISSUE: Whether or not co-ownership on the lots in question still exist thereby allowing private respondent Basilia Lahorra to exercise the right of legal redemption. HELD: The court held that as early as 1960, co-ownership of the parcels of land covered by TCT Nos. T-609 and T-610 was terminated when Alfredo Benito, Luz Caro and the intestate estate of Mario Benito, represented by administrators Saturnino Benito, as trustee and representative of the heirs of Mario Benito, agreed to subdivide the property. It added that an “agreement of partition, though oral, is valid and consequently binding upon the parties.”

65

A partition for subdivision was then filed for the purpose. This was accompanied by the affidavits of Alfredo Benito and Saturnino Benito to the effect that they agree to the segregation of the land owned in common by the three amigos. A subdivision plan was made and by common agreement Lot 1-C, with an area of 163 hectares, was ceded to petitioner, to wit, TCT no. T-4978. In addition, notwithstanding the ruling in the Caram case wherein the sale of the property took place after the partition agreement, the court therein saw no difference with respect to a conveyance which took place before the partition agreement. Regarding the contention of private respondent that she was not notified of the sale, the court ruled that since the right of legal redemption does not exist nor apply in this case because admittedly a subdivision title has already been issued in the name of the petitioner on Lot 1-C sold to her, it becomes moot and academic. It becomes unnecessary to decide whether private respondent complied with the requirements for the exercise of legal redemption under Article 1623 of the New Civil Code.

Bailon – Casilao v. Court of Appeals G.R. No. 78178, April 15, 1988, 160 SCRA 738 Cortes, J. FACTS: The Roman Catholic Archbishop [sic] of Manila was the owner of a parcel of land (Lot No. 1272, Balanga Cadastre) situated in the Barrio of Puerto Rivas, Municipality of Balanga, Bataan, having an area of 3,368 sq. m., more or less covered by OCT No. 14379 of de Registry of Deeds for the province of Bataan. With respect to its rights over its properties in Bataan (inclusive of Lot No. 1272), the said church was succeeded by the Roman Catholic Bishop of San Fernando, Pampanga which was, likewise, succeeded by Catholic Bishop of Balanga — registered as a corporation on 15 December 1975.Prior thereto, or on 23 August 1936, by virtue of the authority given him by the Roman Catholic Archbishop of Manila to donate a portion of Lot No. 1272, the then parish priest and administrator of all the properties of the said church in the Municipality of Balanga Bataan, Rev. Fr. Mariano Sarili, executed an Escritura De Donacion donating an area of 12.40 meters by 21.40 meters or 265.36 sq. m (the subject property) of Lot No. 1272 to Ana de los Reyes and her heirs, as a reward for her long and satisfactory service to the church. Her acceptance of the donation, as well as her possession of the subject property, is indicated in the deed of donation, which deed, for unknown reasons, was refused registration by the Register of Deeds. Six (6) years later, or in 1939, Ana de los Reyes died without issue. Nevertheless, before her death, she had given the subject property to her nephew who had been living with her, the herein defendant-appellant [private respondent]. The latter immediately took possession of the property in the concept of owner, built his house thereon and, through the years, declared the land for taxation purposes as well as paid the taxes due thereon. His possession of the subject property was never disturbed by anybody until plaintiff-

66

appellee [petitioner] filed the instant complaint against him on 5 November 1985, or more than 49 years after the deed of donation was executed. ISSUE: Whether or not petitioner is barred to recover the property by the doctrine of laches. HELD: Yes. Laches means the failure or neglect for an unreasonable and unexplained length of time, to do that which, by exercising due diligence, could or should have been done earlier; it is negligence or omission to assert a right within a reasonable time, warranting the presumption that the party entitled to assert it either has abandoned or declined to assert it. It has also been defined as such neglect or omission to assert a right taken in conjunction with the lapse of time and other circumstances causing prejudice to an adverse party, as will operate as a bar in equity. The following are the essential elements of laches: (1) Conduct on the part of the defendant, or of one under whom he claims, giving rise to the situation complained of; (2) Delay in asserting complainant's right after he had knowledge of the defendant's conduct and after he has an opportunity to sue; (3) Lack of knowledge or notice on the part of the defendant that the complainant would assert the right on which he bases his suit; and (4) Injury or prejudice to the defendant in the event relief is accorded to the complainant. 32 Under the present circumstances, all of the aforegoing elements are attendant in this case. Finally, we agree with the respondent Court of Appeals that, while petitioner is admittedly still the registered owner of the donated property, and jurisprudence is settled as to the imprescriptibility and indefeasibility of a Torrens Title, there is equally an abundance of cases in the annals of our jurisprudence where we categorically ruled that a registered landowner may lose his right to recover the possession of his registered property by reason of laches.

Roque v. Intermediate Appellate Court G.R. No. L-75886, August 30, 1988, 165 SCRA 118 Feliciano, J. FACTS: Petitioner Concepcion Roque, on 6 December 1977, filed a Complaint for "Partition with Specific Performance" (docketed as Civil Case No. 5236-M) with Branch 2 of the then Court of First Instance of Malolos against respondents Emesto Roque and the heirs of Victor Roque. In her complaint, petitioner (plaintiff below) claimed legal ownership of an undivided three-fourths (3/4) portion of Lot No. 1549, by virtue of the 27 November 1961 "Bilihan Lubos at Patuluyan" executed in her favor by Emesto Roque and Victor Roque. In support of this claim, petitioner also presented an undated and unnotarized "Kasulatang Pagkilala sa Bilihan Patuluyan ng Bahagui at Pagmamana sa Labas ng Hukuman at Paghahati-hati at Abuyan ng Bahagui" said to have been signed by the respondents in acknowledgment of the existence and validity of the Bilihan in favor of 67

petitioner. Finally, petitioner alleged that, as a co-owner of Lot No. 1549, she had a right to seek partition of the property, that she could not be compelled to remain in the coownership of the same. Respondents Ernesto Roque and the legal heirs of Victor Roque, however, refused to acknowledge petitioner's claim of ownership of any portion of Lot No. 1549 and rejected the plan to divide the land. ISSUE: Whether or not petitioner can be compelled to remain in the co-ownership. HELD: No. Article 494 of the Civil Code provides that "no co-owner shall be obliged to remain in the co-ownership" and that "each co-owner may demand at any time the partition of the thing owned in common, insofar as his share is concerned." The facts on record clearly show that petitioner Concepcion Roque had been in actual, open and continuous possession of a three-fourths (3/4) portion of Lot No. 1549 ever since execution of the "Bilihan Lubos at Patuluyan" in November of 1961. The Court notes that it was only in their Answer with Compulsory Counterclaim filed with the trial court in December of 1977 — more than sixteen (16) years later — that respondents first questioned the genuineness and authenticity of the "Bilihan Lubos at Patuluyan." Not once during those sixteen (16) years did respondents contest petitioner's occupation of a three-fourths (3/4) portion of Lot No. 1549. Furthermore, if indeed it is true that respondents, as they claim, are the absolute owners of the whole of Lot No. 1549, it is most unusual that respondents would have allowed or tolerated such prolonged occupation by petitioner of a major portion (3/4) of the land while they, upon the other hand, contented themselves with occupation of only a fourth thereof. This latter circumstance, coupled with the passage of a very substantial length of time during which petitioner all the while remained undisturbed and uninterrupted in her occupation and possession, places respondents here in laches: respondents may no longer dispute the existence of the co-ownership between petitioner and themselves nor the validity of petitioner's claim of a threefourths (3/4) interest in Lot No. 1549, as they are deemed, by their unreasonably long inaction, to have acquiesced in the coownership.

Delima v. Court of Appeals G. R. No. L-46296, September 24, 1991, 201 SCRA 641 Medialdea J. FACTS: Lino Delima acquired a lot from the friar lands. Later, he died, leaving as his only heirs three brothers and sisters namely: Eulalio Delima, Juanita Delima, Galileo Delima and Vicente Delima. Galileo was the caretaker of the property. He was able to execute an affidavit adjusting to himself the parcel of land and was able to secure the issuance of a Transfer Certificate of Title in his name. This prompted the heirs of his siblings to file a action for reconveyance. ISSUE: Whether or not the property is subject to prescription. 68

HELD: Yes. From the moment one of the co-owners claims that he is the absolute and exclusive owner of the properties and denies the others any share therein, the question involved is no longer one of partition but of ownership. In such case, the imprescriptibility of the action for partition can no longer be invoked or applied when one of the co-owners has adversely possessed the property as exclusive owner for a period sufficient to vest ownership by prescription. It is settled that possession by the co-owner or co-heir is that of a trutee. In order that such possession is considered adverse to the cestui que trust amounting to a repudiation of the co-ownership, the following elements must concur: 1) that the trustee has performed unequivocal acts amounting to an ouster of cestui que trust; 2) that such positive acts of repudiation had been made known to the cestui que trust; and 3) that the evidence thereon should be clear and conclusive. When the co-owner of the property executed a deed of partition and on the strength thereof, obtained a cancellation of the title in the name of their predecessor and the issuance of a new title in his name as the owner, the statute of limitations started to run for the purposes of the action instituted by the latter seeking a declaration of the existence of the co-ownership and their rights thereafter. The issuance of a new title constituted a clear act of repudiation of the trust and co-ownership.

Aguilar v. Court of Appeals G.R. No. 76351, October 29, 1993, 227 SCRA 472 Bellosillo, J. FACTS: Petitioner Virgilio and respondent Senen are brothers, and were among the seven (7) children of the late Maximiano Aguilar. In 1969, the two brothers purchased a house and lot in Parañaque where their father could spend and enjoy his remaining years in a peaceful neighborhood. Initially, the brothers agreed that Virgilio's share in the co-ownership was two-thirds while that of Senen was one-third. By virtue of a written memorandum, Virgilio and Senen agreed that henceforth their interests in the house and lot should be equal, with Senen assuming the remaining mortgage obligation of the original owners with the SSS in exchange for his possession and enjoyment of the house together with their father. Since Virgilio was then disqualified from obtaining a loan from SSS, the brothers agreed that the deed of sale would be executed and the title registered in the meantime in the name of Senen. It was further agreed that Senen would take care of their father and his needs since Virgilio and his family were staying in Cebu. After Maximiano Aguilar died in 1974, petitioner demanded from private respondent that the latter vacate the house and that the property be sold and proceeds thereof divided among them. Because of the refusal of respondent to give in to petitioner's demands, the latter filed an action to compel the sale of the house and lot so that the they could divide the proceeds between them. In his complaint, petitioner prayed that the proceeds of the sale, be divided on the basis of two-thirds (2/3) in his favor and one-third (1/3) to 69

respondent. Petitioner also prayed for monthly rentals for the use of the house by respondent after their father died. In his answer with counterclaim, respondent alleged that he had no objection to the sale as long as the best selling price could be obtained; that if the sale would be effected, the proceeds thereof should be divided equally; and, that being a co-owner, he was entitled to the use and enjoyment of the property. Rendering judgment by default against defendant, for failure to appear at pre- trial, the trial court found him and plaintiff to be co-owners of the house and lot, in equal shares on the basis of their written agreement. However, it ruled that plaintiff has been deprived of his participation in the property by defendant's continued enjoyment of the house and lot, free of rent, despite demands for rentals and continued maneuvers of defendants, to delay partition. The trial court also upheld the right of plaintiff as co-owner to demand partition. Since plaintiff could not agree to the amount offered by defendant for the former's share, the trial court held that this property should be sold to a third person and the proceeds divided equally between the parties. The CA set aside the order of the trial court. ISSUE: Whether or not petitioner may demand partition of the property. HELD: Yes. We uphold the trial court in ruling in favor of petitioner, except as to the effectivity of the payment of monthly rentals by respondent as co-owner which we here declare to commence only after the trial court ordered respondent to vacate in accordance with its order. Article 494 of the Civil Code provides that no co-owner shall be obliged to remain in the co-ownership, and that each co-owner may demand at any time partition of the thing owned in common insofar as his share is concerned. Corollary to this rule, Art. 498 of the Code states that whenever the thing is essentially, indivisible and the co-owners cannot agree that it be, allotted to one of them who shall indemnify the others, it shall be sold and its proceeds accordingly distributed. This is resorted to (1) when the right to partition the property is invoked by any of the co-owners but because of the nature of the property it cannot be subdivided or its subdivision would prejudice the interests of the co-owners, and (b) the co-owners are not in agreement as to who among them shall be allotted or assigned the entire property upon proper reimbursement of the co-owners. However, being a co-owner respondent has the right to use the house and lot without paying any compensation to petitioner, as he may use the property owned in common long as it is in accordance with the purpose for which it is intended and in a manner not injurious to the interest of the other co-owners. 9 Each co-owner of property held pro indiviso exercises his rights over the whole property and may use and enjoy the same with no other limitation than that he shall not injure the interests of his co-owners, the reason being that until a division is made, the respective share of each cannot be determined and every co-owner exercises, together with his co-participants joint ownership over the pro indiviso property, in addition to his use and enjoyment of the same. Since petitioner has decided to enforce his right in court to end the co-ownership of the house and lot and respondent has not refuted the allegation that he has been preventing the sale of the property by his continued occupancy of the premises, justice and equity demand that respondent and his family vacate the property so that the sale 70

can be effected immediately. In fairness to petitioner, respondent should pay a rental of P1,200.00 per month, with legal interest; from the time the trial court ordered him to vacate, for the use and enjoyment of the other half of the property appertaining to petitioner. When petitioner filed an action to compel the sale of the property and the trial court granted the petition and ordered the ejectment of respondent, the co-ownership was deemed terminated and the right to enjoy the possession jointly also ceased. Thereafter, the continued stay of respondent and his family in the house prejudiced the interest of petitioner as the property should have been sold and the proceeds divided equally between them. To this extent and from then on, respondent should be held liable for monthly rentals until he and his family vacate.

Tomas Claudio Memorial College v. Court of Appeals G.R. No. 124262, October 12, 1999, 316 SCRA 502 Quisimbing, J. FACTS: Juan De Castro died intestate in 1993 leaving a parcel of land located in Morong, Rizal to his heirs. Mariano De Castro one of the heirs sold the said lot to petitioner Tomas Claudio Memorial College by representing that he is the sole owner of the property. The other heirs filed an action for partition before the Regional Trial Court of Rizal alleging that the sale made by Mariano affected only his undivided share of the lot but not the shares of the other co-owners. Petitioner filed a motion to dismiss the partition for the reason that it has already been barred by prescription. The Regional Trial Court of Rizal dismissed the petitioner’s motion. The Court of Appeals affirmed the decision. ISSUES: 1.) Whether or not the sale affected only the undivided share of Mariano 2.) Whether or not the action to file for partition has already prescribed. HELD: 1.) Yes. The Court has consistently ruled that even if a co-owner sells the whole property as his, the sale will affect only his own share but not those of the other coowners who did not consent to the sale. The sale of the whole property by a co-owner does not make the sale null and void but it only transfers the rights to the undivided share of the co-owner who made the sale. The proper action in a case like this is not nullification nor recovery but a division or partition of the entire property. 2.) No. As to the issue on prescription, the Civil Code provides that no prescription shall lie in favor of a co-owner or co-heirs as long as he expressly or impliedly recognizes the co-ownership.

Robles v. Court of Appeals 71

GR. No. 123509, March 14, 2000, 328 SCRA 97 Panganiban, J. FACTS: Leon Robles originally owned the land which was inherited by his son Silvino Robles. The latter then took possession of the land and declared it in his name for taxation purposes. Upon his death, the same was inherited by his widow Maria dela Cruz and his children. The plaintiffs entrusted the payment of the land taxes to their coheir and half-brother, Hilario Tobles. For unknown reasons, the tax declaration of the parcel of land in the name of Silvino Robles was cancelled and transferred to one Exequiel Ballena, father of Andres Robles who is the wife of the defendant Hilario Robles. He secured a loan from the Cardona Rural Bank, Inc. which was foreclosed for failure to pay the mortgage debt wherein the defendant bank emerged as the highest bidder during the auction sale. Defendant Rural Bank sold the same to the Spouses Santos. A n action for quieting of title was filed by respondent Santos. The plaintiffs alleged that they had been in possession of the land since 1942 and it was only in 1987 that they knew about the foreclosure of the mortgage. The Court of Appeals ruled that because of the plaintiff’s inaction for more than 20 years, prescription had already set in. ISSUE: Whether or not the action has prescribed in favour of Hilario Robles. HELD: Yes. Hilario effected no clear and evident repudiation of the co-ownership. It is a fundamental principle that a co-owner cannot acquire by prescription the share of the other co-owners, absent any clear repudiation of the co-ownership. In order that the title may prescribe in favor of a co-owner, the following requisites must concur: (1) the coowner has performed unequivocal acts of repudiation amounting to an ouster of the other co-owners; (2) such positive acts of repudiation have been made known to the other co-owner; and (3) the evidence thereof is clear and convincing. In the present case, Hilario did not have possession of the subject property; neither did he exclude the petitioners from the use and the enjoyment thereof, as they had indisputably shared in its fruits. Likewise, his act of entering into a mortgage contract with the bank cannot be construed to be a repudiation of the co-ownership. As absolute owner of his undivided interest in the land, he had the right to alienate his share, as he in fact did. Neither should his payment of land taxes in his name, as agreed upon by the co-owners, be construed as a repudiation of the co-ownership. The assertion that the declaration of ownership was tantamount to repudiation was belied by the continued occupation and possession of the disputed property by the petitioners as owners.

Galvez vs. Court of Appeals G.R. No. 157954, March 24, 2006 Chico – Nazario, J. FACTS: Timotea F. Galvez died intestate and left a parcel of land in La Union. She left behind her children Ulpiano and petitioner Paz Galvez. Ulpiano who died before Timotea was survived by his son, private respondent, Porfirio Galvez. With regards to 72

the property of Timotea, it is supposed to pass to Paz and Porfirio. However, Porifirio was surprised to discover that Paz executed an affidavit of adjudication stating that she is the true and lawful owner of the said property. Moreover, without the knowledge and consent of Porfirio, Paz sold the property to petitioner Carlos Tam for P10,000.00. Tam thereafter filed an application for registration for said parcel of land. Subsequently, Tam sold the property to Tycoon Properties, Inc. Having knowledge of such sale, Porfirio filed a complaint for Legal Redemption with Damages and Cancellation of documents against petitioner which was affirmed by the lower court and the Court of Appeals. ISSUES: 1.) Whether or not the claim of Porfirio Galvez which is based on an implied trust has already prescribed because the action was filed 24 years after Paz Galvez repudiated the said trust? 2.) Whether or not the claim of Porfirio Galvez which is based on an implied trust is already banned by laches because he failed to assert his alleged right for almost 24 years? 3.) Whether or not Carlos Tam and Tycoon Properties are buyers in good faith and for value and has the right to rely on the face of the title? HELD: 1.) No. Article 494 of the Civil Code provides that "a prescription shall not run in favor of a co-owner or co-heir against his co-owners or co-heirs as long as he expressly or impliedly recognizes the co-ownership." It is a fundamental principle that a co-owner cannot acquire by prescription the share of the other co-owners, absent any clear repudiation of the co-ownership. Prescription, as a mode of terminating a relation of coownership, must have been preceded by repudiation (of the co-ownership). The act of repudiation, in turn, is subject to certain conditions: (1) a co-owner repudiates the coownership; (2) such an act of repudiation is clearly made known to the other co-owners; (3) the evidence thereon is clear and conclusive; and (4) he has been in possession through open, continuous, exclusive, and notorious possession of the property for the period required by law. In this case, we find that Paz Galvez effected no clear and evident repudiation of the co-ownership. The execution of the affidavit of selfadjudication does not constitute such sufficient act of repudiation as contemplated under the law as to effectively exclude Porfirio Galvez from the property. This Court has repeatedly expressed its disapproval over the obvious bad faith of a co-heir feigning sole ownership of the property to the exclusion of the other heirs essentially stating that one who acts in bad faith should not be permitted to profit from it to the detriment of others. 2.) No. On the matter of laches, it is hornbook doctrine that laches is a creation of equity and its application is controlled by equitable considerations. Laches cannot be used to defeat justice or perpetrate fraud and injustice. Neither should its application be used to prevent the rightful owners of a property from recovering what has been fraudulently registered in the name of another. The equitable remedy of laches is, therefore, unavailing in this case.

73

3.) No. As to petitioners Carlos Tam and Tycoon Properties, Inc.’s claim that they are buyers in good faith, same fails to persuade. A purchaser in good faith and for value is one who buys the property without notice that some other person has a right to or interest in such property and pays its fair price before he has notice of the adverse claims and interest of another person in the same property. So it is that the "honesty of intention" which constitutes good faith implies a freedom from knowledge of circumstances which ought to put a person on inquiry. "Tam did not exert efforts to determine the previous ownership of the property in question" and relied only on the tax declarations in the name of Paz Galvez. It must be noted that Carlos Tam received a copy of the summons and the complaint on 22 September 1994. This notwithstanding, he sold the property to Tycoon Properties, Inc. on 27 September 1994. Significantly, Carlos Tam is also an owner of Tycoon Properties, Inc. to the extent of 45%. A notice of lis pendens dated 8 July 1997 filed with the Registry of Deeds of the Province of La Union was inscribed on TCT No. T- 40390. Despite the inscription, Tycoon Properties, Inc. mortgaged the land to Far East Bank and Trust Company for the sum of P11,172,600. All these attendant circumstances negate petitioners’ claim of good faith.

Adille vs. Court of Appeals G.R. No. L-45546, January 29, 1988 Sarmiento, J. FACTS: Felisa Alzul, who owned a parcel of lot in Albay was married twice. The first was with Bernabe Adille whom she had an only child, herein petitioner Rustico Adille. The second was with Procopio Asejo whom she had three children, herein the private respondents. It was alleged that Felisa sold the property in pacto de retro to certain 3 rd persons, for a period of repurchase being 3 years. However, she died without being able to redeem the lot. After her death but during the period of redemption, petitioner Rustico repurchased, by himself alone the said lot. Afterwards, he executed a deed of extrajudicial partition by himself. Efforts to compromise were made but failed. Thus, his halfbrothers and sisters, private respondents filed a present case of partition with accounting on the position that he was only a trustee on an implied trust when he redeemed the lot. Moreover, it turned out that one of the private respondents, Emeteria Asejo was occupying a portion. The lower court was in favor of the petitioner; however, it was reversed by the Court of Appeals. ISSUES: 1.) Whether or not a co-owner can acquire an exclusive ownership over the property held in common. 2.) Whether or not prescription has set in. HELD: 1.) No. The right of repurchase may be exercised by a co-owner with aspect to his share alone. While the records show that the petitioner redeemed the property in its entirety, shouldering the expenses therefore, that did not make him the owner of all of it. In other words, it did not put to end the existing state of co-ownership. Necessary 74

expenses may be incurred by one co-owner, subject to his right to collect reimbursement from the remaining co-owners. There is no doubt that redemption of property entails a necessary expense. Under Article 488 of the Civil Code, it provides that each co-owner shall have a right to compel the other co-owners to contribute to the expenses of preservation of the thing or right owned in common and to the taxes. Any one of the latter may exempt himself from this obligation by renouncing so much of his undivided interest as may be equivalent to his share of the expenses and taxes. No such waiver shall be made if it is prejudicial to the co-ownership. The result is that the property remains to be in a condition of co-ownership. While a vendee a retro, under Article 1613 of the Code, “may not be compelled to consent to a partial redemption,” the redemption by one co-heir or co-owner of the property in its totality does not vest him ownership over it. Failure on the part of all the co-owners to redeem it entitles the vendee a retro to retain the property and consolidate title thereto in his name. But the provision does not give to the redeeming co-owner the right to the entire property. It does not provide for a mode of terminating a co-ownership. Neither does the fact that the petitioner had succeeded in securing title over the parcel in his name terminate the existing co-ownership. While his half-brothers and sisters are, as we said, liable to him for reimbursement as and for their shares in redemption expenses, he cannot claim exclusive right to the property owned in common. Registration of property is not a means of acquiring ownership. It operates as a mere notice of existing title, that is, if there is one. 2.) We hold in the negative. Prescription, as a mode of terminating a relation of coownership, must have been preceded by repudiation (of the co-ownership). The act of repudiation, in turn is subject to certain conditions: (1) a co-owner repudiates the coownership; (2) such an act of repudiation is clearly made known to the other co-owners; (3) the evidence thereon is clear and conclusive, and (4) he has been in possession through open, continuous, exclusive, and notorious possession of the property for the period required by law. The instant case shows that the petitioner had not complied with these requisites. We are not convinced that he had repudiated the co-ownership; on the contrary, he had deliberately kept the private respondents in the dark by feigning sole heirship over the estate under dispute. He cannot therefore be said to have "made known" his efforts to deny the co-ownership. Moreover, one of the private respondents, Emeteria Asejo, is occupying a portion of the land up to the present; yet, the petitioner has not taken pains to eject her therefrom. As a matter of fact, he sought to recover possession of that portion Emeteria is occupying only as a counterclaim, and only after the private respondents had first sought judicial relief.

Adlawan vs. Adlawan G.R. No. 161916, January 20, 2006 Ynares – Santiago, J. FACTS: Petitioner Arnelito Adlawan, the acknowledged illegitimate child of Dominador Adlawan filed an ejejctment suit against the siblings of his father, respondents Narcisa 75

and Emeterio Adlawan. Being the sole heir of Dominador, he executed an affidavit adjudicating the house and lot owned by his father. However, he alleged that out of respect and generosity to respondents, he granted their plea to occupy the subject property provided they would vacate the same should his need for the property arise. Later, when he verbally requested respondents to vacate the house and lot, they refused and filed instead an action for quieting of title. He then also filed a complaint for ejectment. In answer, the respondents, 70 and 59 years of age respectively denied that they begged petitioner to allow them to say on the property since they have been staying there since birth. They claimed that the said lot was originally registered in the name of their deceased parents, Ramon and Oligia Adlawan. Spouses Ramon and Oligia needed money to finance the renovation of their house. Since they were not qualified to obtain a loan, they transferred ownership of the lot to Dominador who was the only one in the family who had a college education. Dominador and his wife, Graciana did not disturb respondents’ possession of the property until they died. They also argued that even if petitioner is indeed Dominador’s acknowledged illegitimate son, his right to succeed is doubtful because Dominador was survived by his wife, Graciana. ISSUE: Whether or not the petitioner can validly maintain the instant case of ejectment. HELD: No. Petitioner averred that he is an acknowledged illegitimate son and the sole heir of Dominador. However, the RTC lost sight of the fact that the theory of succession invoked by petitioner would end up proving that he is not the sole owner of the subject lot. This so because Dominador was survived not only by petitioner but also by his legal wife, Graciana, who died 10 years after the death of Dominador. By intestate succession, Graciana and petitioner became co-owners of the subject lot and house. Petitioner then contended that even granting that he is a co-owner, he can file the instant case pursuant to Article 487 of the Civil Code. This article covers all kinds of actions for the recovery of possession. It includes forcible entry and unlawful detainer (accion interdictal), recovery of possession (accion publiciana) and recovery of ownership (accion de reinvindicacion). A co-owner may bring such action without the necessity of joining all the other co-owners as co-plaintiffs because the suit is presumed to have been filed to benefit his co-owners. It should be stressed, however, that where the suit is for the benefit of the petitioner alone who claims to be the sole owner and entitled to the possession of the litigated property, the action should be dismissed. According to the renowned civilest, Professor Arturo M. Tolentino, he explained that “a co-owner may bring such an action, without the necessity of joining all the other coowners as co-plaintiffs, because the suit is deemed to be instituted for the benefit of all. If the action is for the benefit of the plaintiff alone, such that he claims possession for himself and not for the co-ownership, the action will not prosper. In this case, it is not disputed that petitioner brought the suit for unlawful detainer in his name alone and for his own benefit to the exclusion of the heirs of Graciana as he even executed an affidavit of self-adjudication over the disputed property. It is clear therefore that petitioner cannot validly maintain the instant action considering that he does not recognize the co-ownership that necessarily flows from his theory of succession to the property of his father, Dominador. 76

Sumipat v. Banga G.R. No. 155810, August 13, 2004 Tinga, J. FACTS: The spouses Placida Tabo-tabo and Lauro Sumipat acquired three parcels of land. The couple was childless. Lauro Sumipat, however, sired five illegitimate children. They are the petitioners herein. Lauro executed a document denominated “Deed of Absolute Transfer and/or Quit-Claim over Real Properties” in favor of the petitioners. On the document, it appears that the signature of his wife, Placida which indicates that she gave her marital consent. Moreover, it was alleged that Lauro executed it when he was already very sick and bedridden that upon petitioner Lydia’s request, their neighbor Benjamin Rivera lifted the body of Lauro whereupon Lydia guided his hand in affixing his signature on the document. Lydia left but later returned on the same day and requested Lauro’s unlettered wife, Placida to sign on the said document. After Lauro’s death, his wife, Placida and petitioners jointly administered the properties, 50% of the produce went to his wife. As wife’s share in the produce of the properties dwindled, she filed a complaint for declaration of partition disclaiming any partition in the execution of the subject document. ISSUE: Whether or not a co-ownership was formed from the said deed. HELD: No. A perusal of the deed reveals that it is actually a gratuitous disposition of property — a donation — although Lauro Sumipat imposed upon the petitioners the condition that he and his wife, Placida, shall be entitled to one-half (1/2) of all the fruits or produce of the parcels of land for their subsistence and support. Where the deed of donation fails to show the acceptance, or where the formal notice of the acceptance, made in a separate instrument, is either not given to the donor or else not noted in the deed of donation and in the separate acceptance, the donation is null and void. In this case, the donees’ acceptance of the donation is not manifested either in the deed itself or in a separate document. Hence, the deed as an instrument of donation is patently void. The Court declared that the deeds of sale questioned therein are not merely voidable but null and void ab initio as the supposed seller declared under oath that she signed the deeds without knowing what they were. The significant circumstance meant, the Court added, that her consent was not merely marred by vices of consent so as to make the contracts voidable, but that she had not given her consent at all.

77

Rizal Cement Co., Inc. v. Villareal G.R. No. L-30272, February 28, 1985, 135 SCRA 15 Cuevas, J. FACTS: Respondents are applicants for the registration of two agricultural lands located in Rizal. They presented testimonial and documentary evidence appearing that the property applied for, designated as Lot Nos. 1 and 2 of Plan Psu-147662, have a total area of 26,015 sq. m.; that these lots originally belong to one Maria Certeza; that upon her death, the property was involved in a litigation between her grandchildren and Gonzalo Certeza, and that the lots were given by the latter to Justice de Joya as the latter’s attorney’s fees; that the lots were then sold by de Joya to Filomeno Sta. Ana, who in turn sold the same to spouses Victoriano Cervo and Ignacia Guillermo in 1939; that sometime in November 1955, the said spouses sold the lots to herein applicants as shown by a duly notarized deed of sale. The spouses Cervo declared the property for taxation purposes in the name of the wife, Ignacia Guillermo, and paid for the realty taxes thereon; that prior to the sale, the spouses Cervo had the two lots surveyed first in 1950 and then in 1955. On the other hand, oppositor (Rizal Cement Company) claims to be the owner of the subject lots, having bought the same from Maria Certeza, and to have been in continuous and adverse possession of the property since 1911. To substantiate this claim, petitioner submitted documentary evidence, one of which is a tax declaration of the said lots. The Court of First Instance denied the application for registration of respondents and ordered the issuance of a decree of registration in the name of Rizal Cement Co., after finality of said decision. On appeal, the Court of Appeals reversed and set aside the decision of the CFI. The CA denied petitioner’s motion for reconsideration. Hence, this petition was filed. ISSUE: Whether or not respondents had been in actual possession of the land in question. HELD: Yes. The CA gave credence to the testimony of the witnesses for respondents. As a general rule, it is provided in the Civil Code that possession is acquired by the material occupation of a thing or the exercise of a right or by the fact that it is subject to the action of our will, or by the proper acts or legal formalities established for acquiring such right. Petitioner’s evidence, consisting of tax receipts, tax declaration and survey plan are not conclusive and indisputable basis of one’s ownership of the property in question. Assessment alone is of little value as proof of title. Mere tax declaration does not vest ownership of the property upon defendant.

Wong v. Carpio G.R. No. 50264, October 21, 1991, 203 SCRA 118 Bidin, J. FACTS: William Giger sold a parcel of land through a pacto de recto sale to Manuel Mercado. Mercado only began to harvest the coconut fruits but he never placed anyone 78

over the land to watch it. Neither did he reside in the land nor was there any hut constructed thereon to show possession. Thereafter, Ignacio Wong inspected the land to see if whether there was anyone claiming the land. After finding there was none, he bought the land from Giger. He placed workers on the land, constructed a farmhouse, and fenced the boundaries. He couldn't register the sale due to some technicalities. ISSUE: Whether or not the possession of the disputed land belongs to Ignacio Wong. HELD: It should be stressed that "possession is acquired by the material occupation of a thing or the exercise of a right, or by the fact that it is subject to the action of our will, or by the proper acts and legal formalities for acquiring such right." And that the execution of a sale thru a public instrument shall be equivalent to the delivery of the thing, unless there is stipulation to the contrary. If, however, notwithstanding the execution of the instrument, the purchaser cannot have the enjoyment and material tenancy of the thing and make use of it herself, because such tenancy and enjoyment are opposed by another, then delivery has not been effected. Applying the above pronouncements on the instant case, it is clear that possession passed from vendor William Giger to private respondent Manuel Mercado by virtue of the first sale a retro, and accordingly, the later sale a retro in favor of petitioner failed to pass the possession of the property because there is an impediment — the possession exercised by private respondent. Possession as a fact cannot be recognized at the same time in two different personalities except in the cases of co-possession. Should a question arise regarding the fact of possession, the present possessor shall be preferred; if there are two possessions, the one longer in possession, if the dates of possession are the same, the one who presents a title; and if these conditions are equal, the thing shall be placed in judicial deposit pending determination of its possession or ownership through proper proceedings.

Somodio v. Court of Appeals G.R. No. 82680, August 15, 1994, 235 SCRA 307 Quiason, J. FACTS: Wilfredo Mabugat and Nicanor Somodio bought a residential lot situated at Rajah Muda, Bula, General Santos. Petitioner and Mabugat partitioned the property into two portions, with petitioner taking the western part. Immediately after the partition, petitioner took possession of his portion and planted thereon ipil-ipil trees, coconut trees and other fruit-bearing trees. In 1976, petitioner began construction of a structure with a dimension of 22-by-18 feet on his lot. His employment, however, took him to Kidapawan, North Cotabato, and he left the unfinished structure to the case of his uncle. He would visit the property every three months or on weekened when he had time. Sometime in October 1977, petitioner allowed respondent Felomino Ayco, to transfer his hut to petitioner's lot. About six years later, petitioner demanded that Ayco vacate the premises but such demand proved futile. Hence, on August 23, 1983, petitioner filed an action for unlawful detainer with damages against respondent Ayco. Meanwhile, on 79

June 26, 1983, respondent Ebenecer Purisima entered the land and constructed a house thereon. Four days later, petitioner filed against respondent Purisima a complaint for forcible entry before the same court docketed as Civil Case No. 2013-I. Said case was later consolidated with Civil Case No. 2032-II. ISSUE: Whether or not Somodio has actual possession of the property. HELD: Yes. Article 531 of the Civil Code of the Philippines provides that possession is acquired by the material occupation of a thing or the exercise of a right, or by the fact that it is subject to the action of our will, or by the proper acts and legal formalities established for acquiring such right. Petitioner took possession of the property sometime in 1974 when he planted the property to coconut trees, ipil- ipil trees and fruit trees. In 1976, he started the construction of a building on the property. It is immaterial that the building was unfinished and that he left for Kidapawan for employment reasons and visited the property only intermittently. Possession in the eyes of the law does not mean that a man has to have his feet on every square meter of ground before it can be said that he is in possession (Ramos v. Director of Lands, 39 Phil. 175 [1918]). It is sufficient that petitioner was able to subject the property to the action of his will.

Maglucot – Aw v. Maglucot G.R. No. 132518, March 28, 2000, 329 SCRA 78 Kapunan, J. FACTS: Sometime in 1946 there was a prior oral agreement to tentatively partition Lot No. 1639. By virtue of this agreement, the original co-owners occupied specific portions of Lot No. 1639. It was only in 1952 when the petition to subdivide Lot No. 1639 was filed because two of the co-owners, namely Hermogenes Olis and heirs of Pascual Olis, refused to have said lot subdivided and have separate certificates of title. Significantly, after the 1952 proceedings, the parties in this case by themselves and/or through their predecessors-in-interest occupied specific portions of Lot No. 1639 in accordance with the sketch plan. Sometime in 1963, Guillermo Maglucot rented a portion of the subject lot. Subsequently, Leopoldo and Severo, both surnamed Maglucot, rented portions of subject lot in 1964 and 1969, respectively, and each paying rentals therefor. Said respondents built houses on their corresponding leased lots. They paid the rental amount of P100.00 per annum to Mrs. Ruperta Salma, who represented the heirs of Roberto Maglucot, petitioners’ predecessor-in-interest. In December 1992, however, said respondents stopped paying rentals claiming ownership over the subject lot alleging they had a right over the land because such was not partitioned and they were co-owners. Manglucot-Aw thus filed a complaint for recovery of possession and damages against Manglucot. ISSUE: Whether or not Manglucot-Aw may recover possession by virtue of a valid partition.

80

HELD: Yes. An order for partition is final and not interlocutory and, hence, appealable because it decides the rights of the parties upon the issue submitted. In this case, both the order of partition and the unconfirmed sketch plan are, thus, interlocutory. Nevertheless, where parties do not object to the interlocutory decree, but show by their conduct that they have assented thereto, they cannot thereafter question the decree, especially, where, by reason of their conduct, considerable expense has been incurred in the execution of the commission. Respondents in this case have occupied their respective lots in accordance with the sketch/subdivision plan. They cannot after acquiescing to the order for more than forty (40) years be allowed to question the binding effect thereof. Under the present rule, the proceedings of the commissioners without being confirmed by the court are not binding upon the parties. However, this rule does not apply in case where the parties themselves actualized the supposedly unconfirmed sketch/subdivision plan. The purpose of court approval is to give effect to the sketch/subdivision plan. In this case, the parties themselves or through their predecessors-in-interest implemented the sketch plan made pursuant to a court order for partition by actually occupying specific portions of Lot No. 1639 in 1952 and continue to do so until the present until this case was filed, clearly, the purpose of the court approval has been met. This statement is not to be taken to mean that confirmation of the commissioners may be dispensed with but only that the parties herein are estopped from raising this question by their own acts of ratification of the supposedly non-binding sketch/subdivision plan.

Cequeña v. Bolante G.R. No. 137944, April 6, 2000, 330 SCRA 216 Panganiban, J. FACTS: The petitioners Fernanda Mendoza Cequeña and Eduarda Apiado sought for the ownership and possession of the land occupied by the respondent Honorata Bolante. Prior to 1954, the land in Binangonan, Rizal was declared for taxation purposes in the name of Sinforoso Mendoza, the father of respondent. Sinforoso died in 1930. On the basis of an affidavit, the tax declaration in the name of Sinforoso Mendoza of the contested lot was cancelled and subsequently declared in the name of Margarito Mendoza, the father of the petitioners. Margarito and Sinforoso are brothers. During the cadastral survey, respondent Honorata is the present occupant of the land together with Miguel Mendoza, another brother of the petitioners. The trial court rendered the petitioners as the lawful owner and possessors of the land. However, the Court of Appeals reversed the decision because the genuineness and the due execution of the affidavit. It was said to be insufficient to overcome the denial of respondent and her mother. Moreover, the probative value of petitioners’ tax receipts and declarations paled in comparison with respondent’s proof of ownership of the disputed parcel. The actual, physical, exclusive and continuous possession by respondent since 1985 gave her a better title under Article 538 of the Civil Code. The petitioners contended otherwise that she came into possession through force and violence, contrary to Article 536 of the Civil Code. 81

ISSUES: 1.) Whether or not the respondent has the actual, physical, exclusive and continuous possession of the land. 2.) Whether or not tax declarations and receipts are conclusive evidence of ownership or possession. HELD: 1.) Yes. Possession by the petitioner before 1985 was not exclusive, as the respondent also acquired it before 1985. The records show that the petitioners’ father and brother, as well as the respondent and her mother were simultaneously in adverse possession of the land. Based on Article 538 of the Civil Code, the respondent is the preferred possessor because, benefitting from her father’s tax declaration of the subject lot since 1926, she has been in possession thereof for a longer period. On the other hand, petitioners’ father acquired joint possession only in 1952. 2.) No. Tax declarations and receipts are not conclusive evidence of ownership. At most, they constitute mere prima facie proof of ownership or possession of the property for which taxes have been paid. In the absence of actual public and adverse possession, the declaration of the land for tax purposes does not prove ownership. The petitioners’ claim of ownership of the whole parcel has no legal basis.

82

Aragon v. Insular Government G.R. No. L-6019, March 25, 1911, 19 Phil. 223 Carson, J. FACTS: The Government of the Philippine Islands, through its proper representatives, objected to the application for registration pursuant to the Land Registration Act of a small lot of parcel of land being instituted by herein petitioner, Juan Aragon on the ground that said land forms part of the public domain applying the provisions of subsection 1 of Article 339 of the old Civil Code, now Article 420, paragraph 1 of the New Civil Code which provides that the following things are property of public dominion: (1) Those intended for public use, such as roads, canals, rivers, torrents, ports and bridges constructed by the State, banks, shores, roadstead, and others of similar character. It appears, however, that possessory title over the land in question was duly registered in favor of petitioner, and that the applicant and their predecessors in interest have been in possession of the parcel of land in question, under an undisputed claim of ownership. That there are strong reasons to believe that the land in question was originally well above the ebb and flow of the tide and only in later years have the waters risen to such a height along the shores of the Bay of Manila at this point as to cover the land in question completely at high tide, though, it cannot be ascertained definitely whether it is due to changes in the current and flow of the waters in the bay, or to the gradual sinking of the land along the coast. ISSUE: Whether or not petitioner is entitled ownership over the land in question. HELD: The Court affirmed the decree entered by the lower court in favor of petitioner applying the provisions of Article 446 of the old Civil Code, Article 539 of the New Civil Code which provides that every possessor has a right to be protected in his possession; and should he be disturbed therein, he shall be protected in or restored to said possession by the means established by the laws and the Rules of Court. Corollary, a possessor may lose his possession under the circumstances provided under Article 555 of the New Civil Code, to wit: (1) By the abandonment of the thing; (2) By an assignment made to another either by onerous or gratuitous title; (3) By the destruction or total loss of the thing, or because it goes out of commerce; and (4) By the possession of another, subject to the provisions of Article 537, if the new possession has lasted longer than one year. But the real right of possession is not lost till after the lapse of ten years. The Court held that since the foregoing enumerations with respect to the loss of possession was not conclusively established by the representatives of the government, and the fact that the owners of the land in question have never intended to abandon the same, then it is just and proper to register said land in their name.

Catholic Vicar Apostolic of the Mountain Province v. Court of Appeals G.R. No. 80294, March 23, 1990, 183 SCRA 639 Gancayco, J.

83

FACTS: CA-G.R. No. 38830-R was a land registration case where petitioner and private respondents were asking for confirmation of their alleged imperfect titles to the lots in question under Section 49 (b) of the Public Land Act. In the said decision, the appellate court found that the petitioner was not entitled to confirmation of its imperfect title to Lots 2 and 3. In separate motions for reconsideration filed by private respondents Heirs of Octaviano and Heirs of Juan Valdez relating to the same decision, they also asked that said two lots be registered in their names. On August 12, 1977, the Court of Appeals denied both motions. Effectively, therefore, in the said decision the appellate court ruled that neither the petitioner nor the private respondents are entitled to the confirmation of imperfect title over said two lots. Pursuant to the said decision in CA-G.R. No. 38830-R, the two lots in question remained part of the public lands. This is the only logical conclusion when the appellate court found that neither the petitioner nor private respondents are entitled to confirmation of imperfect title over said lots. The present actions that were instituted in the Regional Trial Court by private respondents are actions for recovery of possession (accion publiciana) and not for recovery of ownership (accion reivindicatoria). ISSUE: Whether or not petitioner is entitled to the possession of the subject lots. HELD: Yes. Under Article 555 (4) of the Civil Code, it is provided that a possessor may lose his possession “by the possession of another, subject to the provisions of Article 537, if the new possession has lasted longer than one year. But the real right of possession is not lost till after the lapse of ten years.” In the case at bar, it is clear that the petitioner was in possession of the said property as borrower in commodatum from private respondents since 1906. However, in 1951 petitioner repudiated the trust when it declared the property for tax purposes under its name. Thus, when petitioner filed its application for registration of the said property in 1962, it had been in adverse possession of the same for at least 11 years. Hence, the action for recover of possession of said property filed by private respondents against petitioner must fail. The Court, therefore, finds that the trial court and the Court of Appeals erred in declaring the private respondents to be entitled to the possession thereof. Much less can they pretend to be owners thereof. Said lots are part of the public domain.

84

EDCA Publishing & Distributing Corp. v. Santos, G.R. No. 80298, April 26, 1990, 134 SCRA 614 Cruz, J. FACTS: Jose Cruz ordered by telephone 406 books from EDCA Publishing and Distributing Corp. (EDCA), payable on delivery. EDCA prepared the corresponding invoice and delivered the books as ordered, for which Cruz issued a check. Subsequently, Cruz sold 120 of the books to Leonor Santos who paid him after verifying the seller's ownership from the invoice he showed her. Meanwhile, EDCA having become suspicious over a second order placed by Cruz even before clearing of his first check, made inquiries with the De la Salle College where he had claimed to be a dean and was informed that there was no such person in its employ. Further, Cruz had no account with the Philippine Amanah Bank, against which he had drawn the check. EDCA went to the police, which arrested Cruz whose real name was Tomas de la Peña. EDCA sought the assistance of the police, and forced their way into the store of the Santos and threatened her with prosecution for buying stolen property. They seized the 120 books. Santos sued for recovery of the books after demand for their return was rejected by EDCA. ISSUES: 1.) Whether or not EDCA was unlawfully deprived of the books because the check issued by the impostor in payment therefor was dishonored. 2.) Whether or not EDCA had the right to cease the books that were sold to Santos. HELD: 1.) No. EDCA was not unlawfully deprived of the books. Article 559 of the Civil Code provides that the possession of movable property acquired in good faith is equivalent to a title. Nevertheless, one who has lost any movable or has been unlawfully deprived thereof, may recover it from the person in possession of the same. If the possessor of a movable lost or of which the owner has been unlawfully deprived has acquired it in good faith at a public sale, the owner cannot obtain its return without reimbursing the price paid therefor. A contract of sale is perfected once agreement is reached between the parties on the subject matter and the consideration. Ownership in the thing sold shall not pass to the buyer until full payment of the purchase only if there is a stipulation to that effect. Otherwise, the rule is that such ownership shall pass from the vendor to the vendee upon the actual or constructive delivery of the thing sold even if the purchase price has not yet been paid. Non-payment only creates a right to demand payment or to rescind the contract, or to criminal prosecution in the case of bouncing checks. But absent the stipulation above noted, delivery of the thing sold will effectively transfer ownership to the buyer who can in turn transfer it to another. 2.) No. Actual delivery of the books having been made, Cruz acquired ownership over the books which he could then validly transfer to the private respondents. The fact that he had not yet paid for them to EDCA was a matter between him and EDCA and did not impair the title to the books acquired by the Santos spouses. Therefore, EDCA was not unlawfully deprived of the books and Santos had rights over the books.

85

De Garcia v. Hon. Court of Appeals G.R. No. L-20264, January 30, 1971, 37 SCRA 129 Fernando, J. FACTS: On October 11, 1953, Angelina Guevarra, while talking to Consuelo de Garcia, recognized her ring in the finger of the latter which she lost sometime in February 1952. Guevarra asked where de Garcia bought the ring to which de Garcia answered that she bought it from her comadre. Guevarra explained to de Garcia that that ring was the very same ring stolen from her. De Garcia handed the ring to Guevarra and the ring fitted her finger. Two or three days later, at the request of Guevarra, she, her husband Lt. Col. Juan Guevara, Lt. Cementina of Pasay PD, de Garcia and her attorney proceeded to the store of Mr. Rebullida to whom they showed the ring in question. Mr. Rebullida examined the ring with the aid of high power lens and after consulting the stock card thereon, concluded that it was the very ring that plaintiff bought from him in 1947. The ring was returned to defendant who despite a written request therefor failed to deliver the ring to plaintiff. In trial, de Garcia said that she bought the ring from Mrs. Miranda who got it from Mrs. Angelita Hinahon who in turn got it from, Aling Petring who was boarding in her house. ISSUE: Whether or not de Garcia’s possession of the ring in good faith confers her title to the said ring. HELD: No. The controlling provision is Article 559 of the Civil Code which provides that possession of movable property acquired in good faith is equivalent to a title. Nevertheless, one who has lost any movable or has been unlawfully deprived thereof may recover it from the person in possession of the same. If the possessor of a movable lost of which the owner has been unlawfully deprived, has acquired it in good faith at a public sale, the owner cannot obtain its return without reimbursing the price paid therefor. Respondent Angelina D. Guevara, having been unlawfully deprived of the diamond ring in question, was entitled to recover it from petitioner Consuelo S. de Garcia who was found in possession of the same. The only exception the law allows is when there is acquisition in good faith of the possessor at a public sale, in which case the owner cannot obtain its return without reimbursing the price. The common law principle that where one of two innocent persons must suffer by a fraud perpetrated by the another, the law imposes the loss upon the party who, by his misplaced confidence, has enabled the fraud to be committed, cannot be applied in a case which is covered by an express provision of the new Civil Code, specifically Article 559. Between a common law principle and statutory provision, the latter must prevail in this jurisdiction. It is thus immediately apparent that there is no merit to the contention raised in the first assigned error that her possession in good faith, equivalent to title, sufficed to defeat respondent Guevara's claim. As the above cases demonstrate, even on that assumption the owner can recover the same once she can show illegal deprivation. Respondent Court of Appeals was so convinced from the evidence submitted that the owner of the ring in litigation is such respondent. 86

Dizon v. Suntay G.R. No. L-30817, September 29, 1972, 47 SCRA 160 Fernando, J. FACTS: Lourdes Suntay is the owner of a 3 carat diamond ring. She entered into a transaction with Clarita Sison, wherein said ring was delivered to the latter for sale on commission. Upon receiving the ring, the receipt was delivered to Suntay. After a lapse of a considerable amount of time, the ring was not yet returned and so Suntay demanded for its return from Sison but the latter could not comply as she had already pledged it with Dizon’s pawnshop for P 2,600.00. After insistent demands, Sison delivered the pawnshop ticket to Suntay. Suntay through her counsel, wrote to Dizon asking for the delivery of the ring pledged but, the latter refused. She filed an action for recovery with P 500 as attorney’s fees and costs. She asked for the remedy of replevin upon filing the requisite bond pending final determination of the action. The CFI of Manila issued the writ and Suntay was able to regain possession during the pendency of the action. The lower court rendered a decision in favor of Suntay. On appeal, Dizon sought the reversal of the lower court’s decision and invoking estoppel. CA affirmed the lower court’s decision. SC affirmed CA decision. ISSUE: Whether or not the owner of the ring may recover its possession from the pawnshop owner. HELD: Yes. Owner of a diamond ring may recover the possession of the same from a pawnshop where another person had pledged it without authority to do so. Art. 559 of the civil code applies and the defense that the pawnshop acquired possession of the without notice of any defect in the title of the pledgor is unavailing. Neither the promptings of equity nor the mandates of moral right and natural justice come to his rescue. Dizon is engaged in a business where presumably ordinary prudence would manifest itself to ascertain whether or not an individual who is offering a jewelry by way of a pledge is entitled to do so. If no such care be taken he should be the last to complain if thereafter the right of the true owner of such jewelry should be recognized.

Ledesma v. Court of Appeals G.R. No. 86051, September 1, 1992, 213 SCRA 195 Davide, J. FACTS: Two motor vehicles—Honda Gemini and Holden Premiere Model—were purchased from Citiwide Motors by a person who identified himself as Jojo Consunji. He bought the vehicles purportedly for his father. Upon delivery to him of the vehicles, he paid a manager’s check drawn against PCIB. The check though was dishonored by the bank on the ground that the check’s value has been materially altered. This was 87

reported to the police authorities and it was found out that the person misrepresenting himself was actually Suarez who had a long line of criminal cases against him for his modus operandi. The Holden car was recovered after being abandoned somewhere in Quezon City. The Honda on the other hand, was discovered to be sold to Ledesma. Ledesma averred he purchased the vehicle in good faith from one Neyra, as evidenced by his certificate of registration. Citiwide Motors was able to recover. ISSUE: Whether or not CITIWIDE MOTORS has been unlawfully deprived. HELD: No. There was a perfected unconditional contract of sale between Citiwide Motors and Suarez. The subsequent dishonor of the check merely amounted to failure of consideration which doesn't render a contract of sale void, but merely allows the prejudiced party to sue for specific performance or rescission of the sale. This being the case, Citiwide motors wasn't unlawfully deprived of the property. It is thus not entitled to the return of the vehicle from Ledesma who bought the property in good faith and for consideration.

88

Azarcon and Abobo v. Eusebio G.R. No. L-11977, April 29, 1959, 105 SCRA 569 Labrador, J. FACTS: Victor Eusebio had a dispute over a parcel of land with Leonardo Azarcon, Manuel Azarcon and Esteban Abobo. Eusebio filed a lease application for a parcel of land, a portion thereof was occupied by Azarcon et al. under a homestead application. Before the dispute could be settled, Eusebio filed a complaint in the CFI of Nueva Ecija, alleging that he had acquired a big parcel of land by lease from the Bureau of Lands, and that while he was in possession thereof, Azarcon et al. occupied a portion. The trial court ruled in favor of Eusebio, and a writ of execution ordering Azarcon et al. to restore possession of the land to Eusebio was issued on October 3, 1955. However, in spite of the receipt of the notice of writ of execution, Azarcon et al. nevertheless entered the land to gather palay which was then pending harvest. ISSUE: Whether or not Azarcon and Abobo are entitled to the pending fruits of the land. HELD: Yes. While the court order of October 3, 1955 ordered them to move out of the premises, it did not prohibit them from gathering the crop then existing thereon. Under the law, a person, who is in possession and who is being ordered to leave a parcel of land while products thereon are in pending harvests, has the right to a part of the net harvest, as expressly provided by Article 545 of the Civil Code. Hence, as the order of execution did not expressly prohibit Azarcon et al. from gathering the pending fruits, which fruits were the result of their possession and cultivation of the land, it cannot be said that they committed an act which is clear violation of the court’s order.

Cordero v. Cabral G.R. No. L-36789, July 25, 1983, 123 SCRA 532 Abad Santos, J. FACTS: Mr. Gregorio Z. Ocampo of Meycauayan, Bulacan, husband of the plaintiff Felipa Cordero and father of the other plaintiffs surnamed Ocampo, died on May 17, 1958. The said deceased left several properties, which were inherited by the plaintiffs including the land in question which parcel of land was originally registered in accordance with the Land Registration Act on December 14, 1933, and was registered and/or transferred in the name of Mr. Gregorio Z. Ocampo on July 31, 1934. After the death of the said Mr. Gregorio Z. Ocampo, the plaintiffs herein took possession of the said parcel of land which is a riceland, but they found out that the southern portion of the same with an area 4,303 square meters, more or less, upon verification, was possessed by the defendants herein, Victoria P. Cabral, Alejandro Berboso and Dalmacio Montaos. Victoria P. Cabral claimed to be the owner of said portion while her co-defendants co-possessed the same as her tenants. The plaintiffs demanded of the defendants to surrender to the former possession of the portion of land and/or vacate it but they refused and failed to do so, and the defendant Victoria P. Cabral continued 89

claiming to be the owner of the same while her co-defendants continued recognizing her as the owner thereof instead of the plaintiffs. Plaintiffs alleged that because of the defendants' occupancy of the aforementioned plaintiffs' portion of land with the area of 4,303 square meters, more or less, to the exclusion of the latter, the said plaintiffs failed to realize a yearly harvest of at least ten (10) cavanes of palay at the rate of P10.00 per cavan, from the harvest-time of 1958 up to the present. ISSUE: Whether or not the defendants must reimburse the fruits receive. HELD: Yes. The disputed land is included in T.C.T. No. 14513 issued to Gregorio Z. Ocampo, the predecessor of the plaintiffs. The original registration which includes the disputed land was not vitiated by error or fraud. The defendants, by their own admission, are in possession of the disputed land. There is no evidence that they were possessors in bad faith. However, their good faith ceased when they were served with summons to answer the complaint. As possessors in bad faith from the service of the summons they "shall reimburse the fruits received and those which the legitimate possessor could have received.

90

Mendoza and Enriquez v. De Guzman G.R. No. L-28721, October 5, 1928, 52 Phil. 164 Malcolm, J. FACTS: In the cadastral proceedings of the municipality of Sariaya, Tayabas, a piece of land identified as lot No. 687 was adjudicated in favor of Martin Mendoza and Natalio Enriquez in equal parts pro indiviso subject to the right of retention on the part of Manuel de Guzman until he shall have been indemnified for the improvements existing on the land. Mendoza has possessed it since 1916. By virtue of this judgment, De Guzman presented a motion requesting the issuance of a writ of possession for lot No. 687 in his favor which was granted on June 25, 1924. Since then De Guzman has had dominion over the land. Being unable to come to an agreement as to the amount which should be allowed for the improvements made on the land, Martin Mendoza and Natalio Enriquez began an action requesting the court to (a) fix the value of the necessary and useful expenses incurred by Manuel de Guzman in introducing the improvements; (b) require the defendant to render an accounting of the fruits received by him and order that the value of the fruits be applied to the payment of the necessary and useful expenses; and (c) decree the restitution of the possession to the plaintiffs. Max. B. Solis, one of the persons who were ejected from the land, asked leave to intervene, alleging, among other things, that De Guzman had transferred all his rights in the improvements and in the lot to him with the exception of two hundred coconut trees. This petition was granted. At the trial which followed and at the instance of the parties, two commissioners were appinted with instructions to inspect the land and to count the number of coconut trees planted thereon, determining the number of fruit-bearing trees and those that are not fruit-bearing as well as the condition of the same. After trial, Judge of First Instance Gloria rendered judgment declaring (a) that the defendant Manuel de Guzman and the intervenor Bernardo Solis have the right to collect from the plaintiffs Martin Mendoza and Natalio Enriquez the sum of P2,046 as compensation for the necessary and useful expenditures in the proportion of 20 per cent for Manuel de Guzman and 80 per cent for Bernardo Solis; and (b) that Manuel de Guzman and Bernardo Solis are obliged to pay to the plaintiffs the sum of P666.93 per annum from June 25, 1924, one-fifth of this amount to be paid by Manuel de Guzman and the other four-fifths by Bernardo Solis. As on the date when this judgment was rendered, that is on September 23, 1927, the amount that the plaintiffs were required to pay to the defendant and intervenor exceeded the amount that the latter were to pay the former, the defendant and intervenor were ordered to deliver the land and its improvement as soon as the plaintiffs have paid the difference, without special pronouncement as to costs. ISSUE: Whether or not the trial court correctly declared the amount to be paid as "indemnizacion" in the form of necessary and useful expenditures incurred by the defendant. HELD: Yes. Article 361 of the Civil Code in the original Spanish text uses the word "indemnizacion." However one may speculate as to the true meaning of the term "indemnizacion" whether correctly translated as "compensation" or "indemnity," the 91

amount of the "indemnizacion" is the amount of the expenditures mentioned in articles 453 and 454 of the Civil Code, which in the present case is the amount of the necessary and useful expenditures incurred by the defendant. Necessary expenses have been variously described by the Spanish commentators as those made for the preservation of the thing; as those without which the thing would deteriorate or be lost; as those that augment the income of the things upon which they are expanded. Among the necessary expenditures are those incurred for cultivation, production, upkeep, etc. Here the plaintiffs have chosen to take the improvements introduced on the land and are disposed to pay the amount of the necessary and useful expenses incurred by the defendant. Inasmuch as the retentionist, who is not exactly a posessor in good faith with in the meaning of the law, seeks to be reimbursed for the necessary and useful expenditures, it is only just that he should account to the owners of the estate for any rents, fruits, or crops he has gathered from it.

Robles and Martin v. Lizarraga Hermanos G.R. No. L-16736, December 22, 1921, 42 Phil. 584 Romualdez, J. FACTS: Anastasia de la Rama died on the 17th of October, 1916, leaving six children, to wit, Magdalena, Jose, Evarista, Zacarias, Felix, and Purificacion, surnamed Robles, and some properties, among which is house No. 4 on Iznart Street in the city of Iloilo. The children and heirs of Anastasia de la Rama entered into partnership with Lizarraga Hermanos in liquidation and settlement of their accounts, by virtue of which the competent court awarded to said partnership the properties left by the deceased, including the aforesaid house No. 4 on Iznart Street. Evarista Robles, one of the heirs, since before the death of her mother Anastasia de la Rama, has been with her husband occupying the aforesaid house No. 4 on Iznart Street, at the beginning, by permission of her mother, later on by the consent of her coheirs, and lastly by agreement with the partnership, Lizarraga Hermanos, to whom it had been awarded, having made some improvements on the house, the value of which is fixed at four thousand five hundred pesos (P4,500), and paying to said partnership forty pesos (P40) monthly as rent of the upper story. On March 18, 1918, Lizarraga Hermanos notified Evarista Robles (Exhibit J) that beginning April next the rent of the upper story of the house would be raised to sixty pesos (P60) a month, and that, if she did not agree to the new rate of rent, she might vacate the house. Evarista Robles refused to pay such a new rate of rent and to vacate the house, and Lizarraga Hermanos brought suit against her for ejectment. Evarista Robles sued Lizarraga Hermanos afterwards to recover the value of the improvements. ISSUES: 1.) Whether or not Evarista Robles is the owner of the aforesaid improvements and has the right to demand payment of their value. 2.) Whether or not she has any right to retain the building until the said value is paid to her. 92

HELD: 1.) Yes. Robles is the owner of the improvements. The expenditures incurred in these improvements were not necessary inasmuch as without them the house would have continued to stand just as before, but were useful, inasmuch as with them the house better serves the purpose for which it was intended, being used as a residence, and the improvements consisting of the addition of a dining room, kitchen, closet, and bathroom in the lower and upper stories of the house, and a stable, suitable as a coach house and dwelling, it is beyond doubt that such improvements are useful to the building. Since the improvements are useful and Robles’ possession is in good faith, applying Article 453, it is beyond question that Evarista Robles is the owner of such improvements, and entitled to reimbursement therefor. 2.) Yes. It is a fact that the value of the improvements in question has not as yet been paid by Lizarraga Hermanos. Wherefore, if Evarista Robles and her husband are entitled to retain the building until the value of such improvements is paid them, Lizarraga Hermanos have not yet any right to oust them from the building, nor, therefore, to be indemnified for any damages caused by the refusal of the plaintiffs found on their legitimate rights. Hence, due to the non-reimbursement of the aforesaid useful expenditures, the possessor in good faith has the right of retention until she has been fully reimbursed with the same.

Metropolitan Waterworks and Sewerage System v. Court of Appeals G.R. No. L-54526, August 25, 1986, 143 SCRA 623 Martinez, J. FACTS: Sometime in 1965, petitioner MWSS (then known as NAWASA) leased around one hundred twenty eight (128) hectares of its land (hereafter, subject property) to respondent CHGCCI (formerly the International Sports Development Corporation) for twenty five (25) years and renewable for another fifteen (15) years or until the year 2005, with the stipulation allowing the latter to exercise a right of first refusal should the subject property be made open for sale. The terms and conditions of respondent CHGCCI's purchase thereof shall nonetheless be subject to presidential approval. Pursuant to Letter of instruction (LOI) No. 440 issued on July 29,1976 by then President Ferdinand E. Marcos directing petitioner MWSS to negotiate the cancellation of the MWSS-CHGCCI lease agreement for the disposition of the subject property, Oscar Ilustre, then General Manager of petitioner MWSS, sometime in November of 1980 informed respondent CHGCCI, through its president herein respondent Pablo Roman, Jr., of its preferential right to buy the subject property which was up for sale. Valuation thereof was to be made by an appraisal company of petitioner MWSS' choice, the Asian Appraisal Co., Inc. which, on January 30, 1981, pegged a fair market value of P40.00 per square meter or a total of P53,800,000.00 for the subject property. Upon being informed that petitioner MWSS and respondent CHGCCI had already agreed in principle on the purchase of the subject property, President Marcos expressed his approval of the sale as shown in his marginal note on the letter sent by respondents 93

Jose Roxas and Pablo Roman, Jr. dated December 20, 1982.The Board of Trustees of petitioner MWSS thereafter passed Resolution 36-83, approving the sale of the subject property in favor of respondent SILHOUETTE, as assignee of respondent CHGCCI. The MWSS-SILHOUETTE sales agreement eventually pushed through. Per the Agreement dated May 11, 1983 covering said purchase, the total price for the subject property is P50,925,200, P25 Million of which was to be paid upon President Marcos' approval of the contract and the balance to be paid within one (1) year from the transfer of the title to respondent SILHOUETTE as vendee with interest at 12% per annum. The balance was also secured by an irrevocable letter of credit. A Supplemental Agreement was forged between petitioner MWSS and respondent SILHOUETTE on August 11, 1983 to accurately identify the subject property. Subsequently, respondent SILHOUETTE, under a deed of sale dated July 26, 1984, sold to respondent AYALA about sixty-seven (67) hectares of the subject property at P110.00 per square meter. Of the total price of around P74 Million, P25 Million was to be paid by respondent AYALA directly to petitioner MWSS for respondent SILHOUETTE's account and P2 Million directly to respondent SILHOUETTE. P11,600,000 was to be paid upon the issuance of title in favor of respondent AYALA, and the remaining balance to be payable within one (1) year with 12% per annum interest. Respondent AYALA developed the land it purchased into a prime residential area now known as the Ayala Heights Subdivision. Almost a decade later, petitioner MWSS on March 26, 1993 filed an action against all herein named respondents before the Regional Trial Court of Quezon City seeking for the declaration of nullity of the MWSS-SILHOUETTE sales agreement and all subsequent conveyances involving the subject property, and for the recovery thereof with damages. ISSUE: Whether or not MWSS failed to provide appropriate security measures over its own records; Circumstances led NBI to believe that the fraudulent encashment as an “inside job”. HELD: Yes. The records likewise show that MWSS failed to provide appropriate security measures over its own records thereby laying confidential records open to unauthorized persons. MWSS's own Fact Finding Committee, in its report submitted to their General Manager underscored this laxity of records control. It observed that the "office of Mr. Ongtengco (Cashier VI of the Treasury Department at the NAWASA) is quite open to any person known to him or his staff members and that the check writer is merely on top of his table. Relying on the foregoing statement of Mr. Ongtengco, the NBI concluded in its Report dated 2 November 1970 that the fraudulent encashment of the 23 checks in question was an "inside job". Thus the NBI believe that the fraudulent act was an inside job or one pulled with inside connivance at NAWASA. The serial numbers of the checks in question conform with the numbers in current use of NAWASA, aside from the fact that these fraudulent checks were found to be of the same kind and design as that of NAWASA's own checks. While knowledge as to such facts may be obtained through the possession of a NAWASA check of current issue, an outsider without information from the inside can not possibly pinpoint which of NAWASA's various accounts has sufficient balance to cover all these fraudulent checks. None of these checks, it should be noted, was dishonored for insufficiency of funds.

94

Bachrach v. Seifert and Elianoff G.R. No. L-2659, October 12, 1950, 87 Phil. 483 Ozaeta, J. FACTS: The deceased E. M. Bachrach, who left no forced heir except his widow Mary McDonald Bachrach, in his last will and testament made various legacies in cash and willed the remainder of his estate. The estate of E. M. Bachrach, as owner of 108,000 shares of stock of the Atok-Big Wedge Mining Co., Inc., received from the latter 54,000 shares representing 50 per cent stock dividend on the said 108,000 shares. On June 10, 1948, Mary McDonald Bachrach, as usufructuary or life tenant of the estate, petitioned the lower court to authorize the Peoples Bank and Trust Company, as administrator of the estate of E. M. Bachrach, to transfer to her the said 54,000 shares of stock dividend by indorsing and delivering to her the corresponding certificate of stock, claiming that said dividend, although paid out in the form of stock, is fruit or income and therefore belonged to her as usufructuary or life tenant. Sophie Seifert and Elisa Elianoff, legal heirs of the deceased, opposed said petition on the ground that the stock dividend in question was not income but formed part of the capital and therefore belonged not to the usufructuary but to the remainderman. While appellants admit that a cash dividend is an income, they contend that a stock dividend is not, but merely represents an addition to the invested capital. ISSUE: Whether or not a dividend is an income and whether it should go to the usufructuary. HELD: Yes. The usufructuary shall be entitled to receive all the natural, industrial, and civil fruits of the property in usufruct. The 108,000 shares of stock are part of the property in usufruct. The 54,000 shares of stock dividend are civil fruits of the original investment. They represent profits, and the delivery of the certificate of stock covering said dividend is equivalent to the payment of said profits. Said shares may be sold independently of the original shares, just as the offspring of a domestic animal may be sold independently of its mother. If the dividend be in fact a profit, although declared in stock, it should be held to be income. A dividend, whether in the form of cash or stock, is income and, consequently, should go to the usufructuary, taking into consideration that a stock dividend as well as a cash dividend can be declared only out of profits of the corporation, for if it were declared out of the capital it would be a serious violation of the law. Under the Massachusetts rule, a stock dividend is considered part of the capital and belongs to the remainderman; while under the Pennsylvania rule, all earnings of a corporation, when declared as dividends in whatever form, made during the lifetime of the usufructuary, belong to the latter. The Pennsylvania rule is more in accord with our statutory laws than the Massachusetts rule.

Hemedes v. Court of Appeals, 95

G.R. No. 107132, October 8, 1999, 316 SCRA 347 Gonzaga – Reyes, J. FACTS: Jose Hemedes, father of Maxima Hemedes and Enrique D. Hemedes. Jose Hemedes executed a document entitled "Donation Inter Vivos with Resolutory Conditions" whereby he conveyed ownership over the subject land, together with all its improvements, in favor of his third wife, Justa Kausapin, subject to the following resolutory conditions that upon her death or marriage, the donee shall revert the said property to anyone of Jose Hemedes children. On September 27, 1960 a "Deed of Conveyance of Unregistered Real Property by Reversion" was made conveying to Maxima Hemedes. She had it titled and mortgage it to R & B Insurance with an annotation of “Usufruct” in favor of her stepmother, Justa Kausapin. Unable to pay the mortgage, R & B Insurance extra-judicially foreclosed the property. However, Justa Kausapin executed another agreement or Kasunduan on May 27, 1971 to his stepson, Enrique D. Hemedes. He obtained tax declarations and pay realty taxes from thereon. The Ministry of Agrarian Reform Office conducted a cadastral survey and indicated Enrique Hemedes as the owner. Enrique Hemedes sold the property to Dominium Realty Const. Corp. (Dominium), a sister company of Asia Brewery. Asia Brewery started to introduce some improvements already when R & B insurance informed them that they are the owners of the property where these improvements are being built. ISSUE: Whether or not the kasunduan executed by Justa Kausapin in favor of Enrique D. Hemedes was valid. HELD: No. The court dismissed the petition and affirmed the decision of the CA. It held that Maxima failed to comply with the requirements of Art. 1332 of the civil code and also failed to repudiate Justa Kausapin’s allegation that she did not execute such a deed and she never allowed to use the land as security for the loan. It was found that the deed of conveyance to Maxima was spurious and it follows that the original title she had for the property was also null and void so as the mortgage to R & B Insurance. On the other hand, Kausapin executed an affidavit to affirm the authenticity of the the kasundudan in favor of his stepson, Enrique Hemedes whom she is dependent from for her financial support.

96

Fabie v. Gutierrez David G.R. No. L-123, December 12, 1945, 75 Phil. 536 Ozaeta, J. FACTS: The petitioner Josefa Fabie is the usufructuary of the income of certain houses located at 372-376 Santo Cristo, Binondo, and 950-956 Ongpin, Santa Cruz, Manila, under the ninth clause of the will of the deceased Rosario Fabie y Grey. The owner of Santo Cristo property abovementioned is the respondent Juan Grey, while those of the Ongpin property are other person not concern herein. Previous to September 1944 litigation arose between Josefa Fabie as plaintiff and Juan Grey as defendant and the owner of the Ongpin property as intervenors, involving the administration of the houses mentioned. ISSUE: Whether or not the action instituted by the petitioner Josefa Fabie is a purely possessory action and as such within the jurisdiction of said court, or an action founded on property right and therefore beyond the jurisdiction of the municipal court. HELD: Yes. It is admitted by the parties that the petitioner Josefa Fabie is the usufructuary of the income of the property in question and that the respondent Juan Grey is the owner thereof. It is likewise admitted that by virtue of a final judgment entered in Civil Case No. 1659 of the Court of First Instance of Manila between the usufructuary and the owner, the former has the right to collect all the rents of said property for herself with the obligation on her part to pay all the real estate taxes, special assessments, and insurance premiums, and make all necessary repairs thereon, and in case default on her part the owner shall have the right to do all those things, in which event he shall be entitled to collect all subsequent rents of the property concerned until the amount paid by him and the expenses of collection are fully satisfied, after which the usufructuary shall again collect the rents. There is therefore no dispute as to the title to or the respective interests of the parties in the property in question. The naked title to the property is to admittedly in the respondent Juan Grey, but the right to all the rents thereof, with the obligation to pay the taxes and insurance premiums and make the necessary repairs, is, also admittedly, vested in the usufructuary, the petitioner Josefa Fabie, during her lifetime. Construing said judgment in the light of the ninth clause of the will of the deceased Rosario Fabie y Grey, which was quoted in the decision and by which Josefa Fabie was made by the usufructuary during her lifetime of the income of the property in question, we find that the said usufructuary has the right to administer the property in question. All the acts of administration — to collect the rents for herself, and to conserve the property by making all necessary repairs and paying all the taxes, special assessments, and insurance premiums thereon — were by said judgment vested in the usufructuary

97

Vda. De Aranas v. Aranas G.R. No. L-56249, May 29, 1987, 150 SCRA 415 Paras, J. FACTS: Fr. Teodoro Aranas, a priest of the Roman Catholic Church, died on January 19, 1953. He had executed on June 6, 1946 his Last Will and Testament which was admitted to probate on August 31, 1956. In said Last Will and Testament, Fr. Teodoro Aranas stipulated the special administration of the remainder of his estate (after returning to his brothers Aniceto and Carmelo or their heirs all properties acquired by him including 10 parcels of land inherited by him from his parents) by Vicente Aranas, a faithful and serviceable nephew and designating him also as recipient of 1/2 of the produce of said properties after deducting the expenses for the administration and the other 1/2 of the produce to be given to the Catholic Church for the eternal repose of the testator's soul. Said pertinent provision reads as follows: “It is my will that the lands I had bought from other persons should be converged and placed under a special administrator. The special administrator of these lands, for his office, should receive one half of all the produce from which shall be deducted the expenses for the administration, and the other half of the produce should be received by the Roman Catholic Church and should be spent for my soul, Vicente B. Aranas (Tingting), because he is a faithful and serviceable nephew, should be the first special administrator of said properties, without bond, until his death or until he should not want to hold the said office anymore. Anyone of the sons of my brother Carmelo Aranas can hold the said office of special administrator, and none other than they. Their father, my brother Carmelo Aranas shall be the one to decide who among them shall hold the said office, but upon the death of my said brother Carmelo Aranas, his said sons will have power to select the one among them ourselves. The special administration is perpetual.” ISSUE: Whether or not perpetual inalienability and administration of the estate of the late Fr. Teodoro Aranas is null and void for being violative of Article 870 of the NCC. HELD: No. Vicente Aranas as a usufructuary has the right to enjoy the property of his uncle with all the benefits which result from the normal enjoyment (or exploitation) of another's property, with the obligation to return, at the designated time, either the same thing, or in special cases its equivalent. This right of Vicente to enjoy the fruits of the properties is temporary and therefore not perpetual as there is a limitation namely his death or his refusal. Likewise his designation as administrator of these properties is limited by his refusal and/or death and therefore it does not run counter to Art. 870 of the Civil Code relied upon by the petitioners. Be it noted that Vicente Aranas is not prohibited to dispose of the fruits and other benefits arising from the usufruct. Neither are the naked owners (the other heirs) of the properties, the usufruct of which has been given to Vicente Aranas prohibited from disposing of said naked ownership without prejudice of course to Vicente's continuing usufruct. To void the designation of Vicente Aranas as usufructuary and/or administrator is to defeat the desire and the dying wish of the testator to reward him for his faithful and unselfish services rendered during the time when said testator was seriously ill or bed-ridden. 98

Locsin v. Valenzuela G.R. No. L-51333, May 18, 1989, 173 SCRA 454 Feliciano, J. FACTS: Petitioners were co-owners of a large tract of agricultural land known as “Hacienda Villa Regalado”. A portion of this land known as Lot No. 2-C-A-3 was subject to lifetime usufructuary rights of respondent Helen Schon. The bulk of this lot was cultivated by the lessees who customarily delivered the rentals to respondent. In 1972, PD 27 was enacted, decreasing the “Emancipation of Tenants”. The tract of land owned in common by the petitioners, including the portion thereof subject to petitioner’s usufructuary rights, fell within the scope of the “Operation Land Transfer”. Petitioners sought the opinion of the Department of Agrarian Reform(DAR) as to who should be entitled to receive the rental payments which continued to be made by the tenants to respondent. The DAR District Officer rendered the opinion that the rental payments were properly considered as amortization payments for the land and as such should pertain to the landowners and not the usufructuary. ISSUE: Whether or not the usufructuary was extinguished by PD 27 and who, between the naked owner and the usufructuary, should be entitled to the amounts paid by the tenants beginning October 21, 1972. HELD: Yes. The usufruct which had therefore existed as a jus in re aliena in favour of Helen Schon was effectively extinguished by PD 27. To hold, as private respondent apparently urges would obviously defeat the purpose of the land reform statute. PD 27 was enacted to emancipate the tenants from “bondage of the soil” by giving to the tenant-farmers ownership of the land which they were cultivating. Ownership over the lands subjected to the Operation Land Transfer moved from the registered owner to the tenants. The Court holds that Lot No. 2-C-A-3 having been declared part of the land reform area and subjected to the Operation Land Transfer, the payments made on October 21, 1972 by the tenant-farmers constituted amortization payments on the cost of the land that they were required to pay under PD 27. These payments, therefore, legally pertain to the petitioners as part of the compensation for the dominion over the land of which they were deprived of by operation of PD 27.

99

Valisno v. Adriano G.R. No. L-37409, May 23, 1988, 161 SCRA 398 Grino – Aquino, J. FACTS: Plaintiff – appellant Nicolas Valisno alleges that he is the owner of a parcel of land in Nueva Ecija which he bought from his sister, Honorata Adriano Francisco. Said land is planted with watermelon, peanuts, corn, tobacco and other vegetables and adjoins the land of Felipe Adriano, on the bank of the Pampanga River. At the time of the sale of the land to Valisno, the land was irrigated by water from the Pampanga River through a canal about 70 meters long, traversing Adriano’s land. Later, Adriano levelled a portion of the irrigation canal so that Valisno was deprived of the irrigation water and prevented from cultivating his 57 – hectare land. Thus, Valisno filed a complaint for deprivation of waters rights in the Bureau of Public Works and Communications (Bureau – PWC). Bureau – PWC ruled in favour of Valisno. Instead of restoring the irrigation canal, Adriano asked for a reinvestigation of the case which was granted. In the meantime, Valisno rebuilt the irrigation canal at his own expense due to his urgent need to irrigate his watermelon fields. Valisno then filed a complaint for damages. However, the Secretary of Bureau – PWC reversed its decision and dismissed Valisno’s complaint. It held that Eladio Adriano’s water rights which had been granted in1923 ceased to be enjoyed by him in 1936 or 1937, when his irrigation canal collapsed. His non-use of the water rights since then for a period of more than five years extinguished the grant by operation of law. Hence, the water rights did not form part of his hereditary estate which his heirs partitioned among themselves. Likewise, Valisno, as vendee of the land which Honorata received from her father’s estate did not acquire any water rights with the land purchased. The trial court held that Valisno had no right to pass through the defendant's land to draw water from the Pampanga River. It pointed out that under Section 4 of the Irrigation Law, controversies between persons claiming a right to water from a stream are within the jurisdiction of the Secretary of Bureau-PWC and his decision on the matter is final, unless an appeal is taken to the proper court within thirty days. The court may not pass upon the validity of the decision of the Public Works Secretary collaterally. Furthermore, there was nothing in Valisno’s evidence to show that the resolution was not valid. It dismissed the complaint and counterclaim. Valisno’s motion for reconsideration was denied, and he appealed to the Court of the Appeals who certified the case to the Supreme Court. ISSUE: Whether the provisions of the Irrigation Act (Act No. 2152) or those of the Civil Code should apply to this case. HELD: The provisions of the Civil Code shall apply. The existence of the irrigation canal on Adriano’s land for the passage of water from the Pampanga River to Honorata's land prior to and at the time of the sale of Honorata's land to Valisno was equivalent to a title for the vendee of the land to continue using it as provided in Article 624 of the Civil Code: The existence of an apparent sign of easement between two estates, established or maintained by the owner of both shall be considered, should either of them be alienated, as a title in order that he easement may continue actively and passively, 100

unless at the time, theownership of the two estates is divided, the contrary should be provided in the title of conveyance of either of them, or the sign aforesaid should be removed before the execution of the deed. This provision shall also apply in case of the division of a thing owned in common on by two or more persons (Civil Code). This provision was lifted from Article 122 of the Spanish Law of Waters which provided: Whenever a tract of irrigated land which previously received its waters from a single point is divided through inheritance, sale or by virtue of some other title, between two or more owners, the owners of the higher estates are under obligation to give free passage to the water as an easement of conduit for the irrigation of the lower estates, and without right to any compensation therefore unless otherwise stipulated in the deed of conveyance. The deed of sale in favor of Valisno included the "conveyance and transfer of the water rights and improvements" appurtenant to Honorata Adriano's property. By the terms of the Deed of Absolute Sale, the vendor Honorata Adriano Francisco sold, ceded, conveyed and transferred to Dr. Nicolas Valisno all "rights, title, interest and participations over the parcel of land above- described, together with one Berkely Model 6 YRF Centrifugal Pump G" suction, 6" discharge 500-1500 GPM, with Serial No. 5415812 and one (1) set of suction pipe and discharge of pipe with elbow, nipples, flanges and footvalves," and the water rights and such other improvements appertaining to the property subject of this sale. According to Valisno, the water right was the primary consideration for his purchase of Honorata's property, for without it the property would be unproductive. Water rights, such as the right to use a drainage ditch for irrigation purposes, which are appurtenant to a parcel of land, pass with the conveyance of the land, although not specifically mentioned in the conveyance. The purchaser's easement of necessity in a water ditch running across the grantor's land cannot be defeated even if the water is supplied by a third person. The fact that an easement by grant may also have qualified as an easement of necessity does detract from its permanency as property right, which survives the determination of the necessity. As an easement of waters in favor of Valisno has been established, he is entitled to enjoy it free from obstruction, disturbance or wrongful interference (19 CJ 984), such as Adriano’s act of levelling the irrigation canal to deprive him of the use of water from the Pampanga River.

Ronquillo, et. al. v. Roco, et. al. G.R. No. L-10619, February 28, 1958, 103 Phil. 84 Montemayor, J. FACTS: Plaintiff Leogario Ronquillo have been in the continuous and uninterrupted use of a road which traversed the land of the defendants, Rocos, in going to Igualdad Street and the market place of Naga City for more than 20 years and that the Rocos have long recognized and respected the private legal easement of a right of way of said plaintiffs. 101

On May 12, 1953, the defendants along with a number of men maliciously obstructed plaintiff’s right of way by constructing a chapel in the middle of the said road and then later, by means of force, intimidation, and threats, illegally and violently planted wooden posts, fenced with barbed wire and closed hermitically the road passage way thereby preventing the plaintiff from using it. The plaintiff claims that he has already acquired the easement of right of way over the land thru prescription by his continuous and uninterrupted use of the narrow strip of land as passage way. However, plaintiff’s complaint was dismissed by the CFI. ISSUE: Whether or not an easement of right of way can be acquired by prescription. HELD: No. The Court held than an easement of right of way may not be acquired thru prescription because though it may be apparent, it is nevertheless discontinuous or intermittent, and therefore, under Article 622 of the New Civil Code, can be acquired only by a virtue of a title. Furthermore, a right of way cannot be acquired by prescription because prescription requires that the possession be continuous and uninterrupted.

Tañedo v. Bernad G.R. No. L-66520 August 30, 1988, 165 SCRA 86 Padilla, J. FACTS: Private respondent Antonio Cardenas owned Lot 7501-A and Lot 7501-B. On the said two lots, a septic tank was constructed for the common use of the occupants of both lots. Cardenas sold Lot 7501-A to herein petitioner Tañedo and the other Lot 7501B was also mortgaged to Tañedo as a security for the payment of loan with an agreement that Cardenas would only sell Lot 7501-B to him. However, said Lot 7501-B was sold to herein respondent Spouses Romeo and Pacita Sim. Upon learning of the said sale, Tañedo offered to redeem the property from Sim but the latter refused. Instead, Sim blocked the sewage pipe connecting the building of Eduardo Tañedo built on Lot 7501-A, to the septic tank in Lot 7501-B. He also asked Tañedo to remove that portion of his building enroaching on Lot 7501-B. Tañedo was then constrained to file an action for legal redemption and damages invoking Article 1622 of the Civil Code. On the other hand, respondent Spouses claimed they are the absolute owners of Lot 7501-B and that Eduardo Tañedo has no right to redeem the land under Art. 1622 of the Civil Code as the land sought to be redeemed is much bigger than the land owned by Tañedo. ISSUE: Whether or not the petitioner’s right to continue to use the septic tank, erected on Lot 7501-B, ceased upon the subdivision of the land and its subsequent sale to different owners who do not have the same interest.

102

HELD: No. Applying Article 631 and 624 of the Civil Code, no statement abolishing or extinguishing the easement of drainage was mentioned in the deed of sale of Lot 7501A to Eduardo Tañedo. Nor did Antonio Cardenas stop the use of the drain pipe and septic tank by the occupants of Lot 7501-A before he sold said lot to Eduardo Tafiedo. Hence, the use of the septic tank is continued by operation of law. Accordingly, the spouses Romeo and Pacita Sim the new owners of the servient estate (Lot 7501- B), cannot impair, in any manner whatsoever, the use of the servitude.

Costabella Corporation v. Court of Appeals G.R. No. 80511 January 25, 1991, 193 SCRA 333 Sarmiento, J. FACTS: Petitioner owns the real estate properties situated at Sitio Buyong, Maribago, Lapu-Lapu City, on which it had constructed a resort and hotel. The private respondents, on the other hand, are the owners of adjoining properties. Before the petitioner began the construction of its beach hotel, the private respondents, in going to and from their respective properties and the provincial road, passed through a passageway which traversed the petitioner's property. In 1981, the petitioner closed the aforementioned passageway when it began the construction of its hotel, but nonetheless opened another route across its property through which the private respondents, as in the past, were allowed to pass. Later, or sometime in August, 1982, when it undertook the construction of the second phase of its beach hotel, the petitioner fenced its property thus closing even the alternative passageway and preventing the private respondents from traversing any part of it. Therefore, an action for injunction with damages was filed against the petitioner by the private respondents before the then Court of First Instance of Cebu. The CFI rendered a decision on March 15, 1984 finding that the private respondents had acquired a vested right over the passageway in controversy based on its long existence and its continued use and enjoyment by the private respondents and also by the community at large. On appeal, Appellate Court held as without basis the trial court's finding that the private respondents had acquired a vested right over the passageway in question by virtue of prescription. The appellate court pointed out that an easement of right of way is a discontinuous one which, under Article 622 of the New Civil Code, may only be acquired by virtue of a title and not by prescription. That notwithstanding, the appellate court went on to rule that ". . . in the interest of justice and in the exercise by this Court of its equity jurisdiction, there is no reason for Us in not treating the easement here sought by appellees Katipunan Lumber Co., Inc. and Perfecta Guangco as one that is not dependent upon the claims of the parties but a compulsory one that is legally demandable by the owner of the dominant estate from the owner of the servient estate." ISSUE: Whether or not the easement may be granted to private respondent over the land of Costabella.

103

HELD: No. It is already well-established that an easement of right of way, as is involved here, is discontinuous and as such can not be acquired by prescription. Insofar therefore as the appellate court adhered to the foregoing precepts, it stood correct. Unfortunately, after making the correct pronouncement, the respondent Appellate Court did not order the reversal of the trial court's decision and the dismissal of the complaint after holding that no easement had been validly constituted over the petitioner's property. Instead, the Appellate Court went on to commit a reversible error by considering the passageway in issue as a compulsory easement which the private respondents, as owners of the "dominant" estate, may demand from the petitioner the latter being the owner of the "servient" estate. Based on Articles 649 and 650 of the Civil Code, the owner of the dominant estate may validly claim a compulsory right of way only after he has established the existence of four requisites, to wit: (1) the (dominant) estate is surrounded by other immovables and is without adequate outlet to a public highway; (2) after payment of the proper indemnity; (3) the isolation was not due to the proprietor's own acts; and (4) the right of way claimed is at a point least prejudicial to the servient estate. In the case at bar, there is absent any showing that the private respondents had established the existence of the four requisites mandated by law.

Encarnacion v. Court of Appeals G.R. No. 77628, March 11, 1991, 195 SCRA 74 Fernan, C.J. FACTS: Petitioner owns the dominant estate bounded on north by the servient estate owned by respondents and an estate owned by Magsino, all of which are located in Talisay, Batangas. The servient estate is bound on the north by the national highway. To provide access to the highway, a one meter road path was paved through in which half of its width was taken from the estate of Magsino and the other half from the estate of the respondent. Petitioner started a nursery plant type of business in which pushcarts were used to haul the plants from his estate to and from his nursery and the highway, using the one meter road path. As his business grew, he bought a jeepney to enable him to transport more plants and soil catering to the now bigger demand. The problem however was that the jeepney cannot pass through the road path since its width would not be accommodated by a one meter width. Petitioner made a request upon the respondent to sell to him 1 ½ meters of their property so that the pathway may be widened to enable his jeepney to pass through. The respondents refused. Petitioner went to court praying that he would be granted the additional land to the right of way already constituted but the trial court rendered a decision adverse to the petitioner because there was no such necessity as it was shown that there was the presence of dried river bed only 80 meters away from the property of the petitioner which he may use as an alternative route. The CA affirmed said decision of the trial court.

104

ISSUE: Whether or not petitioner is entitled to be granted his prayer to buy the additional land to increase the existing one meter road path. HELD: Yes. Even with the presence of the dried river bed, upon thorough investigation, it was found to be an inadequate right of way because a concrete bridge traverses it thereby the jeep would have to jump over said bridge which has a height of 5 meters in order to reach the highway. It was also found that during the rainy season, the same was impassable as it became flooded. This right of way could not provide adequate access to the highway thereby when an estate has no access to a public road, it may demand for a right of way. Furthermore, under Article 651 of the Civil Code, it is the needs of the dominant property which ultimately determine the width of the right of way. In this case, since the business of the petitioner grew larger and pushcarts became tedious to transport his nursery plants, it became necessary for him to do so with a jeepney. And in order to efficiently make such transportation of his plants, the right of way had to be widened to accommodate the width of the jeepney of the petitioner. The petitioner thus shall be granted the additional land to the existing right of way.

Case v. Heirs of Tuason G.R. No. L-5044, December 1, 1909, 14 Phil. 521 Torres, J. FACTS: The counsel for the heirs of Pablo Tuason and Leocadia Santibañez alleged that the parties whom he represents are owners in common of the property adjoining that of the petitioner Edwin Case on the southwest. The latter, extended his southwest boundary line to a portion of the lot of the said heirs of Tuason and Santibañez. They alleged that the true dividing line between the property of the petitioner and that of the said heirs is a belonging to the respondents, and that about two years ago, when Case made alterations in the buildings erected on his land, he improperly caused a portion of them to rest on the wall owned by the respondents. ISSUE: Whether or not the wall is the property of the heirs of the late Tuason and Santibañez. HELD: The wall in controversy belongs to the heirs of the late Tuason and Santibañez for the reason, among others, that in the public document by which one of their original ancestors acquired on the 19th of April, 1796, the property now possessed by them, it appears that property was then already inclosed by a stone wall. The wall supports only the property of the respondents and not that of the petitioner, can not be a party wall, one-half of which along its entire length would belong to the adjoining building owned by Mr. Case. There is not sufficient proof to sustain such claim, and besides, the building erected thereon disproves the pretension of the petitioner.

105

Under article 572 of the Civil Code the easement of party walls is presumed, unless there is a title or exterior sign, or proof to the contrary, among others, in dividing walls adjoining buildings up to the common point of elevation. The legal presumption as to party walls is limited to the three cases dealt with in the said article of the code, and is that of juris tantum unless the contrary appear from the title of ownership of the adjoining properties, that is to say, that the entire wall in controversy belongs to one of the property owners, or where there is no exterior sign to destroy such presumption and support a presumption against the party wall. It can not be presumed that the aforesaid portion was a party wall, and that it was not exclusively owned by the respondents, inasmuch as the latter have proven by means of a good title that has not been impugned by the petitioner, that when one of their ancestors and principals acquired the property the lot was already inclosed by the wall on which the building was erected; it must therefore be understood that in the purchase of the property the wall by which the land was inclosed was necessarily included.

Choco v. Santamaria G.R. No. 6076, December 29, 1911, 21 Phil. 132 Mapa, J. FACTS: The defendant in the building of his house, has made several openings and windows in the walls of the house on both sides overlooking then property of the plaintiff; that at the time the defendant was building his house, and the windows and the openings were being made, the plaintiffs protested, and later on and in the year 1905 made written protest and demand on the defendant, and the defendant received the written protest and referred it to his counsel, who, from the evidence, appears to have suggested an amicable and adjustment of the matter, but the adjustment was not made, and this action was brought. The Trial Court rendered judgment in favor of the plaintiffs, Severina and Flora Choco, and against the defendant, Isidro Santamaria, forever prohibiting the opening of the window stated, which must be closed, and forever prohibiting the opening of the windows and openings marked, which must be closed or made to conform to the requirements of law with regard to dimensions and an iron grate embedded in the wall, with the costs of the action. ISSUE: Whether or not the lower court erred by not ordering in his judgment the final and perpetual closing of the large window opened in the balcony of the back part of the appellee's house and that, though the appellant's lot can be seen through the window, it is not contiguous to the latter's property. HELD: To judge from the photographic views, it opens on the boundary line between the said lot and that the appellee and is situated perpendicularly above a part of the wall that belongs to the appellants. This opinion is corroborated by the testimony of the defendant's witness who took the said photographs, in so far as he said that "a part of 106

the window in question is in front of the plaintiffs' property, since between it and the plaintiffs' property there does not intervene the distance required by law — that of two meters in the first case, and 60 centimeters in the second, therefore, its opening is a manifest violation of the provisions of article 582 of the Civil Code which reads as follows: “Windows with direct views, or balconies or any similar openings projecting over the estate of the neighbor, cannot be made if there is not a distance of, at least, 2 meters between the wall in which they are built and said estate. Neither can side nor oblique views be opened over said property, unless there is a distance of 60 centimeters.” Because of the lack of the distance required by law, the window in question must be closed, and consequently the judgment appealed from should be modified in this sense, as regards this window.

Solid Manila Corporation v. Bio Hong Trading Co., Inc. G.R. No. 90596, April 8, 1991, 195 SCRA 748 Sarmiento, J. FACTS: Petitioner Solid Manila Corporation is the owner of the land in Ermita, Manila. The same lies in the vicinity of another parcel, registered in the name of the private respondent Bio Hong Trading Co., Inc. The private respondent’s title came from a prior owner, and in their deed of sale, the parties thereto reserved as easement of way. As a consequence, there is an annotation which was entered wherein a construction of private alley has been undertaken. However, the petitioner averred that they and their neighbors have been using the private alley and maintained and contributed to its upkeep until sometime in 1983. Due to this, the private respondent constructed steel gates that precluded unhampered used. The petitioner commenced suit for injunction against the private respondent to have the gates removed and to allow full access to the easement. The court a quo issued ex parte an order directing the private respondent to open the gates. However, the Court of Appeals ordered the restoration of the annotation. They ruled that an easement is a mere limitation on ownership and that it does not impair the private respondent’s title, and that since the private respondent had acquired title to the property, “merger” brought about an extinguishment of the easement. The petitioner then averred that the very deed of sale executed between the private respondent and the previous owner of the property “excluded” the alley in question, and that in any event, the intent of the parties was to retain the “alley” as an easement, notwithstanding the sale. ISSUE: Whether or not an easement had been extinguished by merger. HELD: No. The Court held that no genuine merger took place as a consequence of the sale in favor of the private respondent corporation. According to the Civil Code, a merger exists when ownership of the dominant and servient estates is consolidated in the same person. Merger then, as can be seen, requires full ownership of both estates. One thing ought to be noted here, however. The servitude in question is a personal servitude, that is to say, one constituted not in favor of a particular tenement but rather, 107

for the benefit of the general public as stated in Article 614 of the Civil Code. In personal servitude, there is therefore no “owner of a dominant tenement” to speak of, and the easement pertains to persons without a dominant estate, in this case, the public at large. Merger, as we said, presupposes the existence of a prior servient-dominant owner relationship, and the termination of that relation leaves the easement of no use. Unless the owner conveys the property in favor of the public, if that is possible, no genuine merger can take place that would terminate a personal easement.

Floro v. Llenado G.R. No. 75723, June 2, 1995, 244 SCRA 713 Romeo, J. FACTS: Petitioner Simeon Floro is the owner of Floro Park Subdivision who has its own egress and ingress to and from the Mac Arthur Highway by means of its Road Lot 4 and the PNR level crossing. On the other hand, Respondent Orlando Llenado, is the registered owner of Llenado Homes Subdivision, adjacent to Floro Park Subdivision. Prior to its purchase by Llenado, the land was known as the Emmanuel Homes Subdivision, a duly licensed and registered housing subdivision in the name of Soledad Ortega. Bounded on the South by the 5 to 6 meter-wide Palanas Creek, which separates it from the Floro Park Subdivision, and on the west by ricelands belonging to Marcial Ipapo. the Llenado Homes does not have any existing road or passage to the Mac Arthur Highway. However, a proposed access road traversing the idle riceland of Marcial Ipapo has been specifically provided in the subdivision plan of the Emmanuel Homes Subdivision which was duly approved by the defunct Human Settlement Regulatory Commission. Meanwhile, the Llenados sought, and were granted permission by the Floros to use Road Lots 4 and 5 of the Floro Park Subdivision as passageway to and from MacArthur Highway. However no contract of easement of right of way was ever perfected by both parties. Later, Floro barricaded Road Lot 5 with a pile of rocks, wooden posts and adobe stones, thereby preventing its use by the Llenados. Llenado instituted a complaint before the RTC of Malolos, Bulacan against Floro for easement of right of way. The RTC granted the prayer for the issuance of a writ of preliminary mandatory injunction and ordered Floro to open the road and pay damages. Thereafter, the trial court rendered another judgment dismissing the case and lifting the writ of preliminary mandatory injunction previously issued and ordered the plaintiff to pay defendant damages and costs. On appeal by Llenado on the CA, the judgment of the RTC was reversed ordering Floro to open roads 4 and 5 and remove all the objects that prevent passage on road 5 and to pay the plaintiff damages with costs and payment of indemnity for the easement of right of way. ISSUE: Whether or not Llenado is entitled to a compulsory easement of right of way. HELD: No. For the Llenados to be entitled to a compulsory servitude of right of way under the Civil Code, the preconditions provided under Articles 649 and 650 thereof must be established. These preconditions are: (1) that the dominant estate is 108

surrounded by other immovables and has no adequate outlet to a public highway (Art. 649, par. 1); (2) after payment of proper indemnity (Art. 649, par. 1); (3) that the isolation was not due to acts of the proprietor of the dominant estate (Art. 649, last par.); and, (4) that the right of way claimed is at the point least prejudicial to the servient estate; and insofar as consistent with this rule, where the distance from the dominant estate to a public highway may be the shortest (Art. 650). The burden of proving the existence of the prerequisites to validly claim a compulsory right of way lies on the owner of the dominant estate. On the past subdivision plans by Emmanuel Homes which is bought by Llenado, there is an indication of an access road through IPAPO’s property although it was not properly paved, a dirt road will suffice. Seeing this, Llenado has failed to comply with the first requirement. If the servitude requested by Llenado is allowed, other subdivision developers/owners would be encouraged to hastily prepare a subdivision plan with fictitious provisions for access roads merely for registration purposes. Furthermore, if such practice were tolerated, the very purpose for which Presidential Decree No. 957 was enacted, that is, to protect subdivision buyers from unscrupulous subdivision owners/developers who renege on their duties to develop their subdivisions in accordance with the duly approved subdivision plans, would be defeated. In order to justify the imposition of the servitude of right of way, there must be a real, not a fictitious or artificial necessity for it. Mere convenience for the dominant estate is not what is required by law as the basis for setting up a compulsory easement. Even in the face of a necessity, if it can be satisfied without imposing the servitude, the same should not be imposed. The complaint for easement of right of way filed by Llenado in the lower court did not contain a prayer for the fixing of the amount that he must pay Floro in the event that the easement of right of way is constituted. Thus, the existence of the second requisite has likewise not been established. Private respondent Llenado admitted that the Ipapo riceland was no longer being cultivated. Indications are that it has already been abandoned as a ricefield. There was no reason for private respondent's failure to develop the right of way except the inconvenience and expenses it would cost him. Hence, the third requisite has not been met. Failing to establish the existence of the prerequisites under Articles 649 and 650 of the Civil Code, private respondent Llenado's bid for a compulsory easement of right of way over Road Lots 4 and 5 of the Floro Park Subdivision must fail.

Quimen v. Court of Appeals G.R. No. 112331 May 29, 1996, 257 SCRA 163 Bellosillo, J. FACTS: Petitioner Anastacia Quimen together with her brothers Sotero, Sulpicio, Antonio and sister Rufina inherited a piece of property situated in Pandi, Bulacan. They 109

agreed to subdivide the property equally among themselves, as they did, with the shares of Anastacia, Sotero, Sulpicio and Rufina abutting the municipal road. Located directly behind the lots of Anastacia and Sotero is the share of their brother Antonio designated as Lot No. 1448-B-C which the latter divided into two (2) equal parts, now Lots Nos. 1448-B-6-A and 1448-B-6-B. The latter Lot is behind the property of Sotero, father of private respondent Yolanda Oliveros. Yolanda purchased Lot No. 1448-B-6-A from her uncle Antonio through her aunt Anastacia who was then acting as his administratrix. According to Yolanda, when petitioner offered her the property for sale she was hesitant to buy as it had no access to a public road. But Anastacia prevailed upon her to buy the lot with the assurance that she would give her a right of way on her adjoining property. Thereafter, Yolanda constructed a house on the lot she bought using as her passageway to the public highway a portion of Anastacia's property. But when Yolanda finally offered to pay for the use of the pathway Anastacia refused to accept the payment. In fact she was thereafter barred by Anastacia from passing through her property. Later, Yolanda purchased the other lot of Antonio Quimen, Lot No. 1448-B-6B, located directly behind the property of her parents who provided her a pathway between their house from the lot of Yolanda behind the sari sari store of Sotero, and Anastacia's perimeter fence. The store is made of strong materials and occupies the entire frontage of the lot measuring four (4) meters wide and nine meters (9) long. Although the pathway leads to the municipal road it is not adequate for ingress and egress. The municipal road cannot be reached with facility because the store itself obstructs the path so that one has to pass through the back entrance and the facade of the store to reach the road. Finally, Yolanda filed an action with the proper court praying for a right of way through Anastacia's property. The report was that the proposed right of way was at the extreme right of Anastacia's property facing the public highway, starting from the back of Sotero's sari-sari store and extending inward by one (1) meter to her property and turning left for about five (5) meters to avoid the store. However, the trial court dismissed her complaint. The Court of Appeals reversed the decision declaring that she was entitled to a right of way on petitioner’s property and that the way proposed by Yoland would cause the least damage and detriment to the servient estate. ISSUE: Whether or not passing through the property of Yolanda's parents is more accessible to the public road than to make a detour to her property and cut down the avocado tree standing thereon. HELD: Yes. The conditions sine quo non for a valid grant of an easement of right of way are: (a) the dominant estate is surrounded by other immovables without an adequate outlet to a public highway; (b) the dominant estate is willing to pay the proper indemnity; (c) the isolation was not due to the acts of the dominant estate; and, (d) the right of way being claimed is at a point least prejudicial to the servient estate. The criterion of least prejudice to the servient estate must prevail over the criterion of shortest distance although this is a matter of judicial appreciation. While shortest distance may ordinarily imply least prejudice, it is not always so as when there are permanent structures obstructing the shortest distance; while on the other hand, the longest distance may be free of obstructions and the easiest or most convenient to pass 110

through. In other words, where the easement may be established on any of several tenements surrounding the dominant estate, the one where the way is shortest and will cause the least damage should be chosen. However, as elsewhere stated, if these two (2) circumstances do not concur in a single tenement, the way which will cause the least damage should be used, even if it will not be the shortest. As between a right of way that would demolish a store of strong materials to provide egress to a public highway, and another right of way which although longer will only require an avocado tree to be cut down, the second alternative should be preferred.

De Jesus, et. al. v. Homart Corporation, et. al. G.R. No. 44191 – R, August 28, 1974, 19 CA Rep. 831 FACTS: Jesus and Luz Miranda de Jesus are owners of the building located in Tondo, Manila. They brought an action for damages against Homart Corporation and Howmill Manufacturing Corporation, owners of the land adjoining the plaintiff on the same street where a sixty storey concrete building was constructed. Plaintiffs allege that the defendants failed to observe the necessary care and precautions to protect the construction of the plaintiffs by depriving it of sufficient lateral or subjacent support, thereby causing it to sink in some parts; its walls, ceilings, and floorings to crack in some places; and by the careless manner of handling the cement used the roofing’s of the building of the plaintiff were damaged with the accumulated debris piled thereon. ISSUE: Whether or not proper precautions had been taken by the defendants in constructing the building in question so as to prevent causing damage to the building of the plaintiff. HELD: No. Article 684 of the New Civil Code provides “No property shall make such excavations upon his land as to deprive any adjacent land or building sufficient lateral or subjacent support”. A reading of Article 684 shows that the duty of an adjacent owner not to deprive any adjacent land or building of sufficient lateral or subjacent support is an absolute one. It does not depend on the degree of care and precaution made by the proprietor in making the excavation or building on his land. Plaintiffs’ house which adjoins the seven storey concrete building constructed by the defendants had sunk by about eight inches. The sinking of the left side of the house of the plaintiffs was due to the weakening of subjacent support and to the weight of the seven storey concrete building constructed by the defendant, as the excavation made necessarily disturbed the subjacent soil of the plaintiff’s land. Defendants having failed to provide the plaintiff’s land and house with sufficient lateral and subjacent support are liable for damages.

La Vista Association, Inc. v. Court of Appeals 111

G.R. No. 95252, September 5, 1997, 278 SCRA 498 Bellosillo, J. FACTS: The Tuasons owned a vast tract of land in Quezon City and Marikina, and when they sold to Philippine Building Corporation a portion of their landholdings, it was expressly provided in the Deed of Sale with Mortgage that the boundary line between the property sold and the adjoining property of the Tuasons shall be a road fifteen (15) meters wide, one-half of which shall be taken from the property sold to the Philippine Building Corporation and the other half from the portion adjoining belonging to the Tuasons. Philippine Building Corporation then sold and assigned with the consent of the Tuasons, the subject parcel of land to ATENEO which assumed the mortgage and the obligation in the seven and one-half roadway. On their part, the Tuasons developed a part of the estate adjoining the portion sold to Philippine Building Corporation into a residential village known as LA VISTA Subdivision. Thus the boundary between LA VISTA and the portion sold to ATENEO was the 15-meter wide roadway known as the Mangyan Road. The Tuasons developed its 7.5-meter share of the 15-meter wide boundary, while ATENEO deferred improvement on its share and erected instead an adobe wall on the entire length of the boundary. ATENEO subsequently sold to Solid Homes Inc. the land which the latter developed into a subdivision now known as LOYOLA Grand Villas. Solid Homes Inc. now claims to have an easement of right-of-way along Mangyan Road through which they could have access to Katipunan Avenue. LA VISTA however instructed its security guards to prohibit agents and assignees of Solid Homes, Inc., from traversing Mangyan Road, and even constructed concrete posts that prevented the residents of LOYOLA from passing through. Solid Homes, Inc., filed a case before the Regional Trial Court and prayed that LA VISTA been joined from preventing and obstructing the use and passage of LOYOLA residents through Mangyan Road. The lower court recognized the easement of right-ofway along Mangyan Road in favor of Solid Homes, Inc., and ordered LA VISTA to pay damages. On appeal by LA VISTA, the decision of the lower court was affirmed. ISSUE: Whether or not there is an easement of right-of-way over Mangyan Road. HELD: Yes. The predecessors-in-interest of both LA VISTA and Solid Homes, Inc., i.e., the Tuasons and the Philippine Building Corporation, respectively, clearly established a contractual easement of right-of-way over Mangyan Road. A voluntary easement is quite evidently manifested in the stipulation in the Deed of Sale with mortgage executed by them. When the easement was established by their contract, the parties unequivocally made provisions for its observance by all whom in the future might succeed them in dominion. It is thus very apparent that the parties and their respective

112

predecessors-in-interest intended to establish an easement of right-of-way over Mangyan Road for their mutual benefit, both as dominant and servient estates. With this, the free ingress and egress along Mangyan Road created by the voluntary agreement between Ateneo and Solid Homes, Inc., is thus legally demandable (Articles 619 and 625, New Civil Code) with the corresponding duty on the servient estate not to obstruct the same. LA VISTA contends that there are other routes to LOYOLA from Mangyan Road, however, this should not be taken into consideration since the opening of an adequate outlet to a highway can extinguish only legal or compulsory easements, not voluntary easements like in the case at bar. The fact that an easement by grant may have also qualified as an easement of necessity does not detract from its permanency as a property right, which survives the termination of the necessity.

Alcantara v. Reta, Jr. G.R. No. 136996, December 14, 2001, 372 SCRA 364 Pardo, J. FACTS: Alcantara and the other petitioners claim that they were tenants or lessees of the land owned by Reta. The land has been converted into a commercial center and Reta is threatening to eject them. They claim that since they are legitimate tenants or lessees of such land, they have the right of first refusal to purchase the land in accordance with Section 3(g) of Presidential Decree No. 1517, the Urban Land Reform Act. They also claimed that the amicable settlement executed between Reta and Ricardo Roble, one of the petitioners, was void ab initio for being violative of PD No. 1517. On the other hand, Reta claimed that the land is question is not within the scope of PD No. 1517 since it was not proclaimed as an Urban Land Reform Zone (ULRZ). Alcantara, among others, then filed complaint for the exercise of the right of first refusal under PD No. 1517 in the Regional Trial Court. However, such complaint was dismissed and such dismissal was affirmed by the Court of Appeals. Hence, this petition was filed. ISSUE: Whether the Alcantara and the other petitioners have the right of first refusal. HELD: No. The land involved has not been proclaimed an Urban Land Reform Zone (ULRZ). In fact, petitioners filed a petition with the National Housing Authority requesting that said land be declared as an ULRZ. Clearly, the request to have the land proclaimed as an ULRZ would not be necessary if the property was an ULRZ. PD No. 1517 pertains to areas proclaimed as ULRZ. Consequently, petitioners cannot claim any right under the said law since the land involved is not an ULRZ. To be able to qualify and avail of the rights and privileges granted by the said decree, one must be: (1) a legitimate tenant of the land for ten (10) years or more; (2) must have 113

built his home on the land by contract; and, (3) has resided continuously for the last ten (10) years. Those who do not fall within the said category cannot be considered "legitimate tenants" and, therefore, not entitled to the right of first refusal to purchase the property should the owner of the land decide to sell the same at a reasonable price within a reasonable time. Reta denies that he has lease agreements with Alcantara and Roble. Alcantara, on the other hand, failed to present evidence of a lease agreement other than his testimony in court. Reta allowed Roble to use sixty-two (62) coconut trees for P186 from where he gathered tuba. This arrangement would show that it is a usufruct and not a lease. Roble was also allowed to construct his house on the land because it would facilitate his gathering of tuba. This would be in the nature of a personal easement under Article 614 of the Civil Code. Whether the amicable settlement is valid or not, the conclusion would still be the same since the agreement was one of usufruct and not of lease. Thus, Roble is not a legitimate tenant as defined by PD No. 1517. With regard to the other petitioners, Reta admitted that he had verbal agreements with them. This notwithstanding, they are still not the legitimate tenants who can exercise the right of first refusal under PD No. 1517. From the moment Reta demanded that the petitioners vacate the premises, the verbal lease agreements, which were on a monthly basis since rentals were paid monthly, ceased to exist as there was termination of the lease. In conclusion, none of the petitioners is qualified to exercise the right of first refusal under PD No. 1517. There was also no intention on the part of Reta to sell the property. Hence, even if the petitioners had the right of first refusal, the situation which would allow the exercise of that right, that is, the sale or intended sale of the land has not happened. PD No. 1517 applies where the owner of the property intends to sell it to a third party.

Prosperity Credit Resources, Inc. v. Court of Appeals G.R. No. 114170, January 15, 1999, 301 SCRA 52 Mendoza, J. FACTS: Private respondent Metropolitan Fabrics, Inc. (MFI) and petitioner Prosperity Credit Resources, Inc. (PCRI) executed a Memorandum of Undertaking (MOU) wherein PCRI acceded to MFI’s request to redeem three of the seven lots foreclosed and won by the former in the ensuing public auction. The MOA was conditioned upon the agreement that the petitioner shall be given a right of way on the existing private road which forms part of the area to be redeemed by private respondents. Later, PCRI filed an injunctive suit against MFI alleging, inter alia, that the latter, in violation of the terms of the MOU, refused to allow PCRI to make excavations on one side of the access road for the installation of water. The trial court granted the petition for the issuance of the 114

writ of preliminary mandatory injunction. On appeal, the CA set aside the assailed order of the trial court; hence, this petition for review on certiorari. PCRI contends that it is entitled to the issuance of the writ of preliminary mandatory injunction as may be gleaned from the following provision in the MOU: The above cited lot, being an existing private road, will remain open to ingress and egress for whatever kind of passage in favor of PROSPERITY FINANCIAL RESOURCES, INC. or its successors=in-interest. ISSUE: Whether or not the RTC committed grave abuse of discretion in issuing a writ of preliminary mandatory injunction ordering private respondent to allow petitioner to undertake excavations along the access road for the purpose of installing water pipes. Held: Yes. There is no question as to the meaning of the terms “ingress” and “egress”. They give petitioner the right to use the private road as means of entry into and exit from its property on the northwestern side o f the compound. The question concerns the meaning of the phrase “for whatever kind of passage”. The trial court read this phrase to mean that petitioner had the right to make excavations on the side of the access road in order to install a network of pipes. The word “passage” does not, however; “clearly and unmistakably” convey a meaning that includes a right to install water pipes on the access road. The ordinary meaning of the word, as defined in Webster’s Dictionary, is that act or action of passing: movement or transference from one place or point to another.” this legal meaning is not different. It means, according to Black’s Law Dictionary, the act of passing; transit; transition.

Villanueva v. Velasco G.R. No. 130845, November 27, 2000, 346 SCRA 99 Quisumbing, J. FACTS: Petitioner Bryan Villanueva is the registered owner of the parcel of land covered by Transfer Certificate of Title No. 127862 of the Register of Deeds of Quezon City. He bought it from Pacific Banking Corporation, the mortgagee of said property. When petitioner bought the parcel of land there was a small house on its southeastern portion. It occupied one meter of the two-meter wide easement of right of way the Gabriel spouses granted to the Espinolas, predecessors-in-interest of private respondents, in a Contract of Easement of Right of Way. Unknown to petitioner, even before he bought the land, the Gabriels had constructed the aforementioned small house that encroached upon the two-meter easement. Petitioner was also unaware that private respondents, Julio Sebastian and Shirley Lorilla, had filed on May 8, 1991 for easement. As successors-in-interest, Sebastian and Lorilla wanted to enforce the contract of easement. On August 13, 1991, a writ of preliminary mandatory injunction was issued, ordering the Gabriels to provide the right of way and to demolish the small house encroaching on the easement. On January 5, 1995, Judge Tirso Velasco issued an Alias Writ of Demolition. Meanwhile, petitioner filed a Third Party Claim with Prayer to Quash Alias Writ of Demolition. He maintains that the writ of demolition could not apply to his property since he was not a party to the civil case. 115

ISSUE: Whether or not the easement on the property binds petitioner. HELD: Yes. Unlike other types of encumbrance of real property, a servitude like a right of way can exist even if they are not expressly stated or annotated as an encumbrance in a Torrens title because servitudes are inseparable from the estates to which they actively or passively belong. Moreover, Villanueva was bound by the contract of easement, not only as a voluntary easement but as a legal easement. A legal easement is mandated by law, and continues to exist unless its removal is provided for in a title of conveyance or the sign of the easement is removed before the execution of the conveyance conformably with Article 649 in accordance with Article 617 of the Civil Code.

National Irrigation Administration v. Court of Appeals G.R. No. 114348, September 20, 2000, 340 SCRA 661 Pardo, J. FACTS: A free patent over 3 hectares of land in Cagayan was issued and registered in the name of private respondent Dick Manglapus’ predecessor-in-interest, Vicente Manglapus. The land was granted to the latter subject to the provisions of sections 113, 121, 122 and 124 of Commonwealth Act No. 141 which provide that except in favor of the Government or any of its branches, units, or institutions, the land hereby acquired shall be inalienable and shall not be subject to encumbrance for a period of 5 years from the date of this patent and shall not be liable for the satisfaction of any debt contracted prior to the expiration of that period. Subsequently, private respondent Manglapus acquired the lot from Vicente Manglapus by absolute sale and was later registered 11 years later from the issuance of patent. Meanwhile, petitioner National Irrigation Administration entered into a contract with Villamar Development Construction. Under the contract, petitioner NIA was to construct canals in Cagayan. NIA then entered a portion of petitioner’s land and made diggings and fillings thereon. Private respondent then filed a complaint for damages alleging that petitioner’s diggings and fillings destroyed the agricultural use of his land and that no reasonable compensation was paid for its taking. ISSUE: Whether or not the petitioner NIA should pay Manglapus just compensation for the taking of a portion of his property for use as easement of a right of way. HELD: No. We find that NIA is under no obligation. We sustain the appeal. We agree with NIA that the Transfer Certificate of Title and the Original Certificate of Title covering the subject parcel of land contained a reservation granting the government a right of way over the land covered therein. Under the Original Certificate of Title, there was a reservation and condition that the land is subject to “to all conditions and public easements and servitudes recognized and 116

prescribed by law, especially thouse mentioned in Sections 109, 110, 111, 112, 113 and 114, Commonwealth Act No. 141, as amended.” This reservation, unlike the other provisos imposed on the grant, was not limited by any time period and thus is a subsisting condition. Section 112, Commonwealth Act No. 141, provides that lands granted by patent, “shall further be subject to a right of way not exceeding twenty meters in width for public highways, railrods, irrigation, ditches, aqueducts, telegraphs and telephone lines, and similar works as the Government or any public or quasi-public service or enterprises, including mining or forest concessionaires may reasonably require for carrying on their business, with damages for the improvements only. Article 619 of the Civil Code provides that “Easements are established either by law or by the will of the owners. The former are called legal and the latter voluntary easements.” In the present case, we find and declare that a legal easement of a rightof-way exists in favor of the government. The land was originally public land, and awarded to respondent Manglapus by free patent. The ruling would be otherwise if the land were originally private property, in which case, just compensation must be paid for the taking of a part thereof for public use as an easement of a right of way.

Remman Enterprises, Inc. v. Court of Appeals G.R. No. 125018, April 6, 2000, 330 SCRA 145 Bellosillo, J. FACTS: Petitioner Remman Enterprises, Inc. and private respondent Crispin Lat are adjoining landowners in Lipa City. The land of Lat is agricultural and planted mostly with fruit trees while Remman’s land is devoted to its piggery business. The latter’s land is 1 ½ meters higher in elevation than that of respondent Lat. Meanwhile, respondent noticed that petitioner’s waste disposal lagoon was already overflowing and inundating ¼ of Lat’s plantation. He made several representations with petitioner but they fell on deaf ears. Consequently, the trees growing on the flooded portion where it was inundated with water containing pig manure, started to wither and die. Private respondent then filed a complaint for damages alleging that the acidity of the soil in his plantation increased because of the overflow of the water heavy with pig manure from petitioner’s piggery farm. Petitioner denied the allegations and claimed that the construction of additional lagoons was already adopted to contain the waste water coming from its piggery to prevent any damage to the adjoining estate. Petitioner also argued that the damages, if any, were due to a fortuitous event. ISSUE: Whether or not the damages were due to a fortuitous event. HELD: No. We cannot agree with petitioner. We defer instead to the findings opions expressed by the lower courts: Even assuming that the heavy rains constituted an act of God; by reason of their negligence, the fortuitous event became humanized, rendering appellants liable for the ensuing damges. In National Power Corporation v. Court of Appeals, 233 SCRA 649 (1993), the Supreme Court held: “Accordingly, petitioners 117

cannot be heard to invoke the act of God or force majeure to escape liability for the loss or damages sustained by private respondents since they, the petitioners, were guilty of negligence. This event then was not occasioned exclusively by an act of God or force majeure; a human factor – negligence or imprudence – had intervened. The effect tehn of the force majeure in question may be deemed to have, even if only partly, resulted from the participation of man. Thus, the whole occurrence was thereby humanized, as it were, and removed from the rules applicable to acts of God.” As regards the alleged natural easement imposed upon the property of appelle, resort to pertinent provisions of applicable law is imperative. Under Article 637 of the Civil Code, it is provided that “lower estates are obliged to receive the waters which naturally and without the intervention of man descend from the higher estates, as well as the stones or earth which they carry with them. The owner of the lower estate cannot construct works which will impede this easement; neither can the owner of the higher estate make works which will increase the burden.” A similar provion is found under Article 50 of the Water Code of the Philippines (P.D. No. 1067), which provides that “lower estates are obliged to receive the water which naturally and without the intervention of man flow from the higher estates, as well as the stone or eath which they carry with them. The owner of the lower estate cannot construct works which will impede this natural flow, unless he provides an alternative method of drainage; neither can the owner of the higher estate make works which will increase this natural flow.” As worded, the two aforecited provisions impose a natural easement upon the lower estate to receive the waters which naturally and without the intervention of man descend from higher estates. However, where the waters which flow from a higher estate are those which are artificially collected in man-made lagoons, any damage occasioned thereby entitles the owner of the lower or servient estate to compensation.

Jesus is Lord Christian School Foundation, Inc. v. Municipality (now City) of Pasig, Metro Manila G.R. No. 152230, August 9, 2005, 466 SCRA 235 Callejo, Sr., J. FACTS: Respondent Municipality of Pasig needed an access road from E.R. Santos Street, a municipal road near the Pasig Public Market to Barangay Sto. Tomas Bukid, Pasig where 60 to 70 houses, mostly made of light materials, were located. The road has to be at least three meters in width, as required by the Fire Code, so that fire trucks could pass through in case of conflagration. Likewise, the residents in the area needed the road for water and electrical outlets. The municipality then decided to acquire 51 square meters out of the 1,791 square meter property of Lorenzo Ching Cuanco, Victor Ching Cuanco and Ernesto Ching Cuanco Kho, which is abutting E.R. Santos Street.

118

Meanwhile, the Sangguniang Bayan of Pasig approved an Ordinance authorizing the municipal mayor to initiate expropriation proceedings to acquire the said property and appropriate the fund therefore. The ordinance stated that the property owners were notified of the municipality’s intent to purchase the property for public use as an access road but they rejected the offer. The municipality then filed a complaint against the Cuancos for the expropriation of the property under Section 19 of the Republic Act No. 7160 or otherwise known as the Local Government Code. The Cuancos then contended that they had sold the said property to petitioner Jesus is the Lord Christian School Foundation, Inc. (JILCSFI) as evidenced by a deed of sale. When apprised about the complaint, petitioner JILCSFI filed a motion for leave to intervene as defendant-inintervention which was granted. The petitioner JILCSFI asserted that the respondent must comply with the requirements for the establishment of an easement of right-of-way, more specifically, the road must be constructed at the point lease prejudicial to the servient state, and that there must be no adequate outlet to a public highway. The petitioner also claimed that the portion of the lot sought to be expropriated is located at the middle protion of the petitioner’s entire parcel of land, thereby splitting the lot into two halves, and making it impossible for the petitioner to put up its school building and worship center. ISSUE: Whether or not the petitioner JILCSFI’s contentions are tenable. HELD: No. The subject property is expropriated for the purpose of constructing a road. The respondent is not mandated to comply with the essential requisites for an easement of right-of-way under the New Civil Code. Case law has it that in the absence of legislative restriction, the grantee of the power of eminent domain may determine the location and route of the land to be taken unless such determination is capricious and wantonly injurious. Expropriation is justified so long as it is for the public good and there is genuine necessity of public character. Governmentmay not capriciously choose what private property should be taken. The respondent has demonstrated the necessity for constructing a road from E.R. Santos Street to Sto. Tomas Bukid. The witnesses, who were residents of Sto. Tomas Bukid, testified that although there were other ways through which one can enter the vicinity, no vehicle, however, especially fire trucks, could enter the area except through the newly constructed Damayan Street. This is more than sufficient to establish that there is a genuine necessity for the construction of a road in the area. After all, absolute necessity is not required, only reasonable and practical necessity will suffice. Nonetheless, the respondent failed to show the necessity for constructing the road particularly in the petitioner’s property and not elsewhere. We note that the wheras clause of the ordinance states that the 51-square meter lot is the shortest and most suitable access road to connect Sto. Tomas Bukid to E.R. Santos Street. The respondent’s complaint also alleged that the said portion of the petitioner’s lot has been surveyed as the best possible ingress and egress. However, the respondent failed to adduce a preponderance of evidence.

119

Acap v. Court of Appeals G.R. No. 118114, December 7, 1995, 251 SCRA 30 Padilla, J. FACTS: The title to Lot 1130 of the Cadastral Survey of Hinigaran, Negros Occidental was evidenced by OCT R-12179. The lot has an area of 13,720 sq. m. The title was issued and is registered in the name of spouses Santiago Vasquez and Lorenza Oruma. After both spouses died, their only son Felixberto inherited the lot. In 1975, Felixberto executed a duly notarized document entitled “Declaration of Heirship and Deed of Absolute Sale” in favor of Cosme Pido. Since 1960, Teodoro Acap had been the tenant of a portion of the said land, covering an area of 9,500 sq. m. When ownership was transferred in 1975 by Felixberto to Cosme Pido, Acap continued to be the registered tenant thereof and religiously paid his leasehold rentals to Pido and thereafter, upon Pido’s death, to his widow Laurenciana. The controversy began when Pido died interstate and on 27 November 1981, his surviving heirs executed a notarized document denominated as “Declaration of Heirship and Waiver of Rights of Lot 1130 Hinigaran Cadastre,” wherein they declared to have adjudicated upon themselves the parcel of land in equal share, and that they waive, quitclaim all right, interests and participation over the parcel of land in favor of Edy de los Reyes. The document was signed by all of Pido’s heirs. Edy de los Reyes did not sign said document. It will be noted that at the time of Cosme Pido’s death, title to the property continued to be registered in the name of the Vasquez spouses. Upon obtaining the Declaration of Heirship with Waiver of Rights in his favor, de los Reyes filed the same with the Registry of Deeds as part of a notice of an adverse claim against the original certificate of title. Thereafter, delos Reyes sought for Acap to personally inform him that he had become the new owner of the land and that the lease rentals thereon should be paid to him. Delos Reyes alleged that he and Acap entered into an oral lease agreement wherein Acap agreed to pay 10 cavans of palay per annum as lease rental. In 1982, Acap allegedly complied with said obligation. In 1983, however, Acap refused to pay any further lease rentals on the land, prompting delos Reyes to seek the assistance of the then Ministry of Agrarian Reform (MAR) in Hinigaran, Negros Occidental. The MAR invited Acap, who sent his wife, to a conference scheduled on 13 October 1983. The wife stated that the she and her husband did not recognize delos Reyes’s claim of ownership over the land. On 28 April 1988, after the lapse of four (4) years, delos Reyes filed a complaint for recovery of possession and damages against Acap, alleging that as his leasehold tenant, Acap refused and failed to pay the agreed annual rental of 10 cavans of palay despite repeated demands. On 20 August 1991. ISSUE: Whether or not the subject declaration of heirship and waiver of rights is a recognized mode of acquiring ownership by private respondent over the lot in question. HELD: An asserted right or claim to ownership or a real right over a thing arising from a juridical act, however justified, is not per se sufficient to give rise to ownership over the res. That right or title must be completed by fulfilling certain conditions imposed by law. Hence, ownership and real rights are acquired only pursuant to a legal mode or

120

process. While title is the juridical justification, mode is the actual process of acquisition or transfer of ownership over a thing in question. Under Article 712 of the Civil Code, the modes of acquiring ownership are generally classified into two (2) classes, namely, the original mode (i.e., through occupation, acquisitive prescription, law or intellectual creation) and the derivative mode (i.e., through succession mortis causa or tradition as a result of certain contracts, such as sale, barter, donation, assignment or mutuum). In the case at bench, the trial court was obviously confused as to the nature and effect of the Declaration of Heirship and Waiver of Rights, equating the same with a contract (deed) of sale. They are not the same. In a Contract of Sale, one of the contracting parties obligates himself to transfer the ownership of and to deliver a determinate thing, and the other party to pay a price certain in money or its equivalent. Upon the other hand, a declaration of heirship and waiver of rights operates as a public instrument when filed with the Registry of Deeds whereby the intestate heirs adjudicate and divide the estate left by the decedent among themselves as they see fit. It is in effect an extrajudicial settlement between the heirs under Rule 74 of the Rules of Court. Hence, there is a marked difference between a sale of hereditary rights and a waiver of hereditary rights. The first presumes the existence of a contract or deed of sale between the parties. The second is, technically speaking, a mode of extinction of ownership where there is an abdication or intentional relinquishment of a known right with knowledge of its existence and intention to relinquish it, in favor of other persons who are co-heirs in the succession. Private respondent, being then a stranger to the succession of Cosme Pido, cannot conclusively claim ownership over the subject lot on the sole basis of the waiver document which neither recites the elements of either a sale, or a donation, or any other derivative mode of acquiring ownership.

De Luna v. Abrigo G.R. No. L-57455, January 18, 1990, 181 SCRA 150 Medialdea, J. FACTS: Prudencio de Luna donated a portion of a lot of the Cadastral Survey of Lucena to the Luzonian University Foundation. The donation was embodied in a Deed of Donation Intervivos and made subject to certain terms and conditions and provided for the automatic reversion to the donor of the donated property in case of violation or non-compliance. The foundation failed to comply with the conditions of the donation. De Luna "revived" the said donation in favor of the foundation, in a document entitled "Revival of Donation Intervivos" subject to terms and conditions which among others, required it to construct a chapel, a nursery and a kindergarten school in the donated property within five (5) years from execution. The automatic reversion to the donor of the donated area in case of violation of the conditions was also provided. The foundation, through its president, accepted the donation. A "Deed of Segregation" was 121

later executed by De Luna and the foundation whereby the area donated was adjudicated to the foundation. The heirs of de Luna later filed a complaint with the trial court alleging that the terms and conditions of the donation were not complied with by the foundation. Thus, it prayed for the cancellation of the donation and the reversion of the donated land to the heirs. The foundation invoked, among others, the defense of prescription of action. The court dismissed the complaint. It ruled that under Article 764 of the New Civil Code, actions to revoke a donation on the ground of non-compliance with any of the conditions of the donation shall prescribe in four years (4) counted from such non-compliance. In the instant case, the four-year period for filing the complaint for revocation commenced on April 9, 1976 and expired on April 9, 1980. Since the complaint was brought on September 23, 1980 or more than five (5) months beyond the prescriptive period, it was already barred by prescription. ISSUE: Whether or not the complaint is one for judicial decree of revocation of the donation in question as contemplated in Article 764 of the New Civil Code and which prescribes in four (4) years and not an action to enforce a written contract which prescribes in ten (10) years. HELD: The donation subject of this case is one with an onerous cause. It was made subject to the burden requiring the donee to construct a chapel, a nursery and a kindergarten school in the donated property within five years from execution of the deed of donation. It is true that under Article 764, actions for the revocation of a donation must be brought within for (4) years from the non-compliance of the conditions of the donation. However, the said article does not apply to onerous donations in view of the specific provision of Article 733 providing that onerous donations are governed by the rules on contracts. Therefore, the rules on contracts and the general rules on prescription and not the rules on donations are applicable in the case at bar. Furthermore, while the judicial action for the rescission of a contract is generally not necessary where the contract provides that it may be automatically revoked and cancelled for violation of any of its terms and conditions, however, where one of the parties contests or denies the rescission, judicial intervention is necessary not for purposes of obtaining a judicial declaration rescinding a contract already deemed rescinded by virtue of an agreement providing for rescission even without judicial intervention, but in order to determine whether or not the rescission was proper. Judicial action will be necessary as without it, the extrajudicial resolution will remain contestable and subject to judicial invalidation, unless attack thereon should become barred by acquiescence, estoppel or prescription. In the instant case, trial court was therefore not correct in holding that the complaint is barred by prescription under Article 764 because Article 764 does not apply to onerous donations. As provided in the donation executed on April 9, 1971, compliance with the terms and conditions of the contract of donation, shall be made within five (5) years from its execution. The complaint which was filed on September 23, 1980 was then well within the ten (10) year prescriptive period to enforce a written contract pursuant to Article 1144 par. 1, counted from April 9, 1976. 122

Reyes v. Mosqueda G.R. No. L-45262, July 23, 1990 Gutierrez, Jr., J. FACTS: On May 15, 1969, Dr. Emilio Pascual executed a Deed of Donation of real property located at 1109-1111 R. Papa St. Tondo, Manila in favor of Ofelia Parungao, a minor, with her mother, Rosario Duncil, accepting the gift and donation for and in her behalf. However, Ursula Pascual alleged that Dr. Pascual during his lifetime on November 2, 1966 executed a Donation mortis causa in her favor covering the said property. Parungao, upon reaching the age of majority was able to register the Deed of Donation with the Register of Deeds in Manila and was issued a TCT. On September 23, 1976, Ursula executed a deed of absolute sale over the Tondo property in favor of Benjamin, Oscar, Jose, and Emmanuel Reyes. Benjamin filed a complaint for the declaration of nullity of the TCT of Parungao and/or reconveyance of the deed of title. The CFI of Manila declared the TCT in the name of Parungao null and void and ordered the Register of Deeds to cancel the title. On appeal, the Court of Appeals ruled that the 1966 donation to Ursula was inter vivos, which meant that the property was already transferred to Ursula at that time. ISSUE: Whether or not the donation to Ursula was Inter Vivos or Mortis Causa. HELD: It was a Donation Inter Vivos. The title given by the donor in the deed of donation is not a determinative factor which makes the donation inter vivos or mortis causa. It is the body of the document of donation and the statements contained therein and not the title that should be considered in ascertaining the intent of the donor. In the case, the donor used the term donation Mortis Causa but from the stipulations of the deed, it can be clearly inferred that he was actually executing a donation Inter Vivos to Ursula. The transfer of ownership over the properties donated to Ursula was immediate and independent of the death of Dr. Pascual since it was a donation Inter Vivos. The provision as regards the reservation of properties for the donor's subsistence in relation to the other provisions of the deed of donation confirms the intention of the donor to give the naked ownership of the properties to Ursula immediately after the execution of the deed of donation. Hence, he could not have donated the property again in 1969 in favor of Parungao since the lot was already transferred to Ursula at that time.

Liguez v. Court of Appeals G.R. No. L-11240, December 18, 1957, 102 Phil. 577 Reyes, J.B.L., J. 123

FACTS: The case began upon complaint filed by petitioner-appellant against the widow and heirs of the late Salvador P. Lopez to recover a parcel of land in barrio Davao. Plaintiff averred to be its legal owner, pursuant to a deed of donation of said land, executed in her favor by the late owner, Salvador P. Lopez, on 18 May 1943. The defense interposed was that the donation was null and void for having an illicit causa or consideration, which was the plaintiff's entering into marital relations with Salvador P. Lopez, a married man; and that the property had been adjudicated to the appellees as heirs of Lopez by the court of First Instance, since 1949. It was ascertained by the Court of Appeals that the donated land originally belonged to the conjugal partnership of Salvador P. Lopez and his wife, Maria Ngo; that the latter had met and berated Conchita for living maritally with her husband, sometime during June of 1943; that the widow and children of Lopez were in possession of the land and made improvements thereon; that the land was assessed in the tax rolls first in the name of Lopez and later in that of his widow.; and that the deed of donation was never recorded. Upon these facts, the Court of Appeals held that the deed of donation was inoperative, and null and void (1) because the husband, Lopez, had no right to donate conjugal property to the plaintiff appellant; and (2) because the donation was tainted with illegal cause or consideration, of which donor and donee were participants. ISSUE: Whether or not the donation is valid. HELD: In the present case, it is scarcely disputable that Lopez would not have conveyed the property in question had he known that appellant would refuse to cohabit with him. The cohabitation was an implied condition to the donation, and being unlawful, necessarily tainted the donation itself. The rule that parties to an illegal contract, if equally guilty, will not be aided by the law but will both be left where it finds them, has been interpreted by this Court as barring the party from pleading the illegality of the bargain either as a cause of action or as a defense. Memo auditor propriam turpitudinem allegans. The appellant seeks recovery of the disputed land on the strength of a donation regular on its face. To defeat its effect, the appellees must plead and prove that the same is illegal. But such plea on the part of the Lopez heirs is not receivable, since Lopez, himself, if living, would be barred from setting up that plea; and his heirs, as his privies and successors in interest, can have no better rights than Lopez himself. Appellees, as successors of the late donor, being thus precluded from pleading the defense of immorality or illegal causa of the donation, the total or partial ineffectiveness of the same must be decided by different legal principles. In this regard, the Court of Appeals correctly held that Lopez could not donate the entirety of the property in litigation, to the prejudice of his wife Maria Ngo, because said property was conjugal in 124

character and the right of the husband to donate community property is strictly limited by law

Pershing Tan Queto v. Court of Appeals G.R. No. L-35648, March 27, 1987, 148 SCRA 54 Paras, J. FACTS: Herein private respondent Restituta Tacalinar Guangco de Pombuena received the questioned lot from her mother Basilides Tacalinar either as a purported donation or by way of purchase with P50 as the alleged consideration thereof. The donation or sale was consummated while Restituta was already married to her husband Juan Pombuena. Juan then filed for himself and his supposed co-owner Resitituta an application for a Torrens Title over the land which was later on granted pronouncing him (‘married to Resitiuta’) as the owner of the land. A contract of lease over the lot was entered into between petitioner, Pershing Tan Queto and Restituta with the consent of her husband for a period of 10 years. The lease of contract having expired, Restituta filed for unlawful detainer against Tan Queto. The unlawful detainer case was won by the spouses in the Municipal Court but on appeal in the CFI the entire case was dismissed because of a barter agreement whereby Tan Queto became the owner of the disputed lot and the spouses became the owners of a parcel of land with the house thereon previously owned before the barter by Tan Queto. After the barter agreement, Tan Queto constructed on the disputed land a concrete building without any objection from Restituta. Afterwards Restituta sued both Juan and Tan Queto for reconveyance of the title over the registered but disputed lot, for annulment of the barter, and for recovery of the land with damages. The respondent court’s decision which later on was affirmed by the Supreme court led to the reformation of the Contract of Sale of the disputed lot from Basilides to Restituta from a sale to a conveyance of the share of Restituta in the future hereditary estate of her parents. Hence, this petition for a motion for reconsideration. ISSUE: Whether or not the conveyance of the share of Restituta in the future hereditary estate of her parents was valid hence a paraphernal property. HELD: No. The court ruled that the land is conjugal, not paraphernal. The oral donation of the lot cannot be a valid donation intervivos because it was not executed in a public instrument (Art. 749, Civil Code), nor as a valid donation mortis causa for the formalities of a will were not complied with. The allegation that the transfer was a conveyance to RESTITUTA of her hereditary share in the estate of her mother (or parents) cannot be sustained for the contractual transmission of future inheritance is generally prohibited. The fact is ownership was acquired by both JUAN and RESTITUTA by tradition (delivery) as a consequence of the contract of sale (See Art. 712, Civil Code) with 125

P50.00 (then a considerable amount) as the cause or consideration of the transaction. The lot is therefore conjugal, having been acquired by the spouses thru onerous title (the money used being presumably conjugal there being no proof that RESTITUTA had paraphernal funds of her own).

Pajarillo vs. Intermediate Appellate Court G.R. No. 72908, August 11, 1989, 176 SCRA 340 Cruz, J. FACTS: Perfecta Balane de Cordero died intestate in 1945 and leaving a tract of 28 hectares of land with buildings and improvements in the Quezon Province. On May 20, 1946, perfecta’s siblings Juana and Felipe executed a public instrument entitled “ Extrajudicial settlement of the estate of the decease Perfecta Balane de Cordero.” In it they disposed that in according to Perfecta’s wishes and in consideration of love and affection, the said property be donated to private respondent Salud Suterio de Matias, Perfecta’s niece, who will assume the encumbrance/obligation to the Philippine National Bank in the amount of P 1,000. In the same document, the done accepted the donation in a public instrument. The instrument was never registered nor the title transferred to Salud’s name although she immediately took possession of the land. Sometime in 1951, Salud transferred the possession of the land to her mother Juana, who was then staying with her brother Claudio and his family. During the period they were occupying the land, Claudio paid realty taxes thereon. On May 25, 1956, Juana executed a deed of absolute sale conveying the land to Claudio. Two years later, Claudio had the land registered in his name. Claudio died in 1961 and his mother in 1963. On June 30, 1965, the private respondents Salud and Pedro Matias filed a complaint for the reconveyance of the property on the ground that the deed of sale in favour of Claudio was fictitious and the registration in his name was null and void. Salud claimed that no compensation was paid by Claudio and that the transaction was deliberately concealed from her by her brother and the defendants. ISSUE: Whether or not the extra-judicial settlement was a donation. HELD: Yes. Felipe and Juana had declared themselves the heirs of Perfecta and the owners of the property in question. As such, they were free to give the land to whomever they pleased and for whatever reason they saw fit. Hence, if they choose to respect Perfecta’s wishes and carry out her intentions by donating the land to Salud, there was no legal impediment to their doing so. There is no question that Felipe and Juana could have simply disregarded their sister’s sentiments and decided not to donate the property to Salud. The fact that they did no do this speaks well of their integrity and their loyalty to their deceased sister. The extra-judicial settlement also reflects their own affection for Salud which constituted the valid consideration for their own act of liberality.

126

Cruz v. Court of Appeals G.R. No. L-58671, November 22, 1985, 140 SCRA 245 Plana, J. FACTS: In 1973, Eduvigis Cruz, a childless widow, donated a 235.5 sq. m. residential lot in San Isidro, Taytay, Rizal together with the two-door apartment erected thereon to her grandnieces (private respondents Teresita, Lydia and Cecilia, all surnamed De Leon). The property was accordingly transferred to the names of private respondents. In 1974, Cruz judicially adopted Cresencia Ocreto, a minor, after which she extrajudicailly tried to revoke the donation, but the donee resisted, alleging that: (1) the property in question was co-owned by Eduvigis Cruz and her brother, the late Maximo Cruz, grandfather of the donees, hence the latter own 1/2 of the property by inheritance; and (2) Eduvigis owns another property, an agricultural land of more than two hectares situated in Barrio Dolores, Taytay, Rizal, hence the donation did not impair the presumptive legitime of the adoptive child. Petitioner filed a complaint against the donees for revocation of donation, invoking Article 760, par. 3 of the NCC. The trial court rendered a decision revoking the donation. On appal, The Court of Appeals reversed the trial court and dismissed the complaint. ISSUE: Whether or not the Court of Appeals correctly dismissed the complaint to annul the subject donation. HELD: Yes. In the case of the subsequent adoption of a minor by one who had previously donated some or all of his properties to another, the donor may sue for the annulment or reduction of the donation within 4 years from the date of adoption, if the donation impairs the legitime of the adopted, taking into account the whole estate of the donor at the time of the donation of the child (Articles 760, 761 and 763 of the NCC). Of course, the burden of proof is on the plaintiff-donor, who must allege and establish the requirements prescribed by law, on the basis of which annulment or reduction of the donation can be adjudged. Unfortunately, in the case at bar, the complaint for annulment does not allege that the subject donation impairs the legitime of the adopted child. Indeed, it contains no indication at all of the total assets of the donor. Nor is there proof of impairment of legitime. On the contrary, there is unrebutted evidence that the donor has another piece of land worth P273,420 in 1977. The legal situation of petitioner-donor is made worse by the factual finding of the Court of Appeals that the grandfather of the donees was the owner pro indiviso of one-half of the donated land, the effect of which is to reduce the value of the donation which can then more easily be taken from the portion of the estate within the free disposal of petitioner.

Roman Catholic Archbishop of Manila v. Court of Appeals 127

G.R. No. 77425, June 19, 1991, 198 SCRA 300 Regalado, J. FACTS: On August 23, 1930, the spouses Eusebio de Castro and Martina Rieta executed a deed of donation in favor of herein petitioner Roman Catholic Archbishop of Manila covering a parcel of land located at Cavite. The deed of donation provides that the donee shall not dispose or sell the property within a period of 100 years from the execution of the deed of donation, otherwise a violation of such condition would render ipso facto null and void the donation and the property would revert to the estate of the donors. However, on June 30, 1980 while within the prohibitive period to dispose, petitioner executed a deed of absolute sale of the property subject of the donation in favor of the petitioner-spouses Florencio and Soledad Ignao in consideration of the sum of P114,000.00. Hence, private respondents filed a complaint for the nullification of the deed of donation. In their answer, the petitioners filed a motion to dismiss based on the grounds that the action has been barred by prescription because the complaint was filed four years after the sale, and that the complaint states no cause of action. ISSUE: Whether or not the deed of donation in favor of the Roman Catholic Archbishop of Manila may be revoked. HELD: No. The complaint in the case at bar cannot be barred by prescription because the applicable prescriptive period is not the 4-year period provided in Article 764 of the New Civil Code, rather it is the 10-year period ordinary prescription shall apply because the deed of donation provides for the automatic reversion of the property to the original owner in case of violation of any condition. The Court in the previous case of De Luna v. Abrigo has already settled such prescriptive period. However, although the action cannot be dismissed on the ground of prescription, the same should be dismissed for lack of cause of action. The cause of action of the private respondents is based on the fact that the petitioner sold the lot during the 50th year of the prohibitive period of 100 years. Such prohibitive period imposed by the respondents was unreasonable because applying in analogy Articles 494 and 870 of the New Civil Code, the donor cannot order a prohibitive period of disposition exceeding 20 years. As such, the said condition regarding the prohibitive period being contrary to law shall be considered as null and void pursuant to Art. 727 of the New Civil Code but the donation shall remain valid and subsisting. Thus, respondents cannot anymore revoke the donation, and the sale of the property by the petitioner to the Ignao spouses shall be valid and with legal effects.

Eduarte v. Court of Appeals G.R. No. 105944, February 9, 1996, 253 SCRA 391 128

Francisco, J. FACTS: Pedro Calapine was the registered owner of a parcel of land with an area of 12,199 square meters. He executed a deed entitled “Donation InterVivos” ceding onehalf portion thereof to his niece Helen S. Doria. Eventually, the whole parcel of land was ceded to Doria by Calapine. Doria then donated a portion of 157 square meters to the Calauan Christian Reformed Church. He also sold, transferred and conveyed unto the spouses Eduarte the parcel of land, saving the 700 square meters on which Doria’s house was erected. However, Pedro Calapine filed a complaint against Doria, the Calauan Christian Reformed Church, Inc. and the spouses Eduarte claiming that his signature to the deed of donation was a forgery. He prays for the revocation of the donation made in favour of Doria, to declare null and void the deeds of donation and sale that she had executed in favor of the Calauan Christian Reformed Church, Inc. and the spouses Eduarte. ISSUE: Whether or not the petitioners are buyers in bad faith of the donated property. HELD: No. The rule is well-settled that mere possession cannot defeat the title of a holder of a registered torrens title to real property. When herein petitioners purchased the subject property from Helen Doria, the same was already covered by TCT No. T23205 under the latter's name. And although Helen Doria's title was fraudulently secured, such fact cannot prejudice the rights of herein petitioners absent any showing that they had any knowledge or participation in such irregularity. Thus, they cannot be obliged to look beyond the certificate of title which appeared to be valid on its fade and sans any annotation or notice of private respondents' adverse claim. Contrary therefore to the conclusion of respondent Court, petitioners are purchasers in good faith and for value as they bought the disputed property without notice that some other person has a right or interest in such property, and paid a full price for the same at the time of the purchase or before they had notice of the claim or interest of some other person in the property. And having established beyond doubt that Helen Doria fraudulently secured her title over the disputed property which she subsequently sold to petitioners, Helen Doria should instead be adjudged liable to private respondents, and not to petitioners as declared by the trial court and respondent Court of Appeals, for the resulting damages to the true owner and original plaintiff, Pedro Calapine. Petition granted.

Quilala v. Alcantara G.R. No. 132681, December 3, 2001, 371 SCRA 311 Ynares – Santiago, J. FACTS: On February 20, 1981, Catalina Quilala executed a "Donation of Real Property Inter Vivos" in favor of Violeta Quilala over a parcel of land. The "Donation of Real Property Inter Vivos" consists of two pages. The first page contains the deed of donation itself, and is signed on the bottom portion by Catalina Quilala as donor, Violeta 129

Quilala as donee, and two instrumental witnesses. The second page contains the Acknowledgment, which states merely that Catalina Quilala personally appeared before the notary public and acknowledged that the donation was her free and voluntary act and deed. There appear on the left-hand margin of the second page the signatures of Catalina Quilala and one of the witnesses, and on the right-hand margin the signatures of Violeta Quilala and the other witness The deed of donation was registered with the Register of Deeds and, in due course, TCT No. 17214 was cancelled and TCT No. 143015 was issued in the name of Violeta Quilala. On November 7, 1983, Catalina Quilala died. Violeta Quilala likewise died on May 22, 1984. Petitioner Ricky Quilala alleges that he is the surviving son of Violeta Quilala. Meanwhile, respondents Gliceria Alcantara, Leonora Alcantara, Ines Reyes and Juan Reyes, claiming to be Catalina's only surviving relatives within the fourth civil degree of consanguinity, executed a deed of extrajudicial settlement of estate, dividing and adjudicating unto themselves the above-described property. On September 13, 1984, respondents instituted against petitioner and Guillermo T. San Pedro, the Registrar of Deeds of Manila, an action for the declaration of nullity of the donation inter vivos. The trial court found that the deed of donation, although signed by both Catalina and Violeta, was acknowledged before a notary public only by the donor, Catalina. Consequently, there was no acceptance by Violeta of the donation in a public instrument, thus rendering the donation null and void. On appeal, the Court of Appeals rendered a decision affirming with modification the decision of the trial court by dismissing the complaint for lack of cause of action without prejudice to the filing of probate proceedings of Catalina's alleged last will and testament. ISSUE: Whether or not the deed of donation is void for lack of acceptance on the part of the donee Violeta Quilala. HELD: No. As stated above, the second page of the deed of donation, on which the Acknowledgment appears, was signed by the donor and one witness on the left-hand margin, and by the donee and the other witness on the right hand margin. Surely, the requirement that the contracting parties and their witnesses should sign on the left-hand margin of the instrument is not absolute. The intendment of the law merely is to ensure that each and every page of the instrument is authenticated by the parties. The requirement is designed to avoid the falsification of the contract after the same has already been duly executed by the parties. Hence, a contracting party affixes his signature on each page of the instrument to certify that he is agreeing to everything that is written thereon at the time of signing. Simply put, the specification of the location of the signature is merely directory. The fact that one of the parties signs on the wrong side of the page does not invalidate the document. The purpose of authenticating the page is served, and the requirement in the above-quoted provision is deemed substantially complied with.

130

In the same vein, the lack of an acknowledgment by the donee before the notary public does not also render the donation null and void. The instrument should be treated in its entirety. It cannot be considered a private document in part and a public document in another part. The fact that it was acknowledged before a notary public converts the deed of donation in its entirety a public instrument. The fact that the donee was not mentioned by the notary public in the acknowledgment is of no moment. To be sure, it is the conveyance that should be acknowledged as a free and voluntary act. In any event, the donee signed on the second page, which contains the Acknowledgment only. Her acceptance, which is explicitly set forth on the first page of the notarized deed of donation, was made in a public instrument.

Hemedes v. Court of Appeals G.R. No. 107132, October 8, 1999, 316 SCRA 347 Gonzaga – Reyes, J. FACTS: Jose Hemedes, father of Maxima Hemedes and Enrique D. Hemedes. Jose Hemedes executed a document entitled "Donation Inter Vivos With Resolutory Conditions" whereby he conveyed ownership over the subject land, together with all its improvements, in favor of his third wife, Justa Kausapin, subject to the following resolutory conditions that upon her death or marriage, the DONEE shall revert the said property to anyone of Jose Hemedes children. On September 27, 1960 a "Deed of Conveyance of Unregistered Real Property by Reversion" conveying to Maxima Hemedes. She had it titled and mortgage it to R & B Insurance with an annotation of USUFRUCT favor of her stepmother,Justa Kausapin. Unable to pay the mortgage, R & B Insurance extra-judicially foreclosed the property. However, Justa Kausapin executed another agreement or Kasunduan on May 27, 1971 to his stepson, Enrique D. Hemedes. He obtained tax declarations and pay realty taxes from thereon. The Ministry of Agrarian Reform Office conducted a cadastral survey and indicated Enrique Hemedes as the owner. Enrique Hemedes sold the property to Dominium Realty Const. Corp.(Dominium), a sister company of Asia Brewery. Asia Brewery started to introduce some improvements already when R & B insurance informed them that they are the owners of the property where these improvements are being built. ISSUE: Whether or not the kasunduan executed by Justa Kausapin in favor of Enrique D. Hemedes valid. HELD: The court dismissed the petition and affirmed the decision of the CA. It held that Maxima failed to comply with the requirements of Art. 1332 of the civil code and also failed to repudiate Justa Kausapin’s allegation that she did not execute such a deed and she never allowed to use the land as security for the loan. It was found that the deed of conveyance to Maxima was spurious and it follows that the original title she had for the 131

property was also null and void so as the mortgage to R & B Insurance. On the other hand, Kausapin executed an affidavit to affirm the authenticity of the kasundudan in favor of his stepson, Enrique Hemedes whom she is dependent from for her financial support. Siguan v. Lim G.R. No. 134685, November 19, 1999, 318 SCRA 725 Davide, Jr., C.J. FACTS: On 2 July 1991, a Deed of Donation conveying parcels of land and purportedly executed by LIM on 10 August 1989 in favor of her children, Linde, Ingrid and Neil was registered with the Office of the Register of Deeds of Cebu City. On 23 June 1993, petitioner filed an accion pauliana against LIM and her children to rescind the questioned Deed of Donation and to declare as null and void the new transfer certificates of title issued for the lots covered by the questioned Deed. Petitioner claimed therein that sometime in July 1991, LIM, through a Deed of Donation, fraudulently transferred all her real property to her children in bad faith and in fraud of creditors, including her; that LIM conspired and confederated with her children in antedating the questioned Deed of Donation, to petitioner's and other creditors' prejudice; and that LIM, at the time of the fraudulent conveyance, left no sufficient properties to pay her obligations. The RTC ruled in favor of Siguan and rescinded the Contract, but was reversed by the CA. ISSUE: Whether or not the Deed of Donation executed by respondent may be rescinded for being in fraud of her alleged creditor. HELD: We resolve these issues in the negative. Art. 1381 of the Civil Code enumerates the contracts which are rescissible, and among them are "those contracts undertaken in fraud of creditors when the latter cannot in any other manner collect the claims due them." The action to rescind contracts in fraud of creditors is known as accion pauliana. For this action to prosper, the following requisites must be present: (1) the plaintiff asking for rescission has a credit prior to the alienation, although demandable later; (2) the debtor has made a subsequent contract conveying a patrimonial benefit to a third person; (3) the creditor has no other legal remedy to satisfy his claim; (4) the act being impugned is fraudulent; (5) the third person who received the property conveyed, if it is by onerous title, has been an accomplice in the fraud. The general rule is that rescission requires the existence of creditors at the time of the alleged fraudulent alienation, and this must be proved as one of the bases of the judicial pronouncement setting aside the contract. Without any prior existing debt, there can neither be injury nor fraud. While it is necessary that the credit of the plaintiff in the accion pauliana must exist prior to the fraudulent alienation, the date of the judgment enforcing it is immaterial. Even if the judgment be subsequent to the alienation, it is merely declaratory, with retroactive effect to the date when the credit was constituted. In the instant case, the alleged debt of LIM 132

in favor of petitioner was incurred in August 1990, while the deed of donation was purportedly executed on 10 August 1989. Even assuming arguendo that petitioner became a creditor of LIM prior to the celebration of the contract of donation, still her action for rescission would not fare well because the third requisite was not met. Under Article 1381 of the Civil Code, contracts entered into in fraud of creditors may be rescinded only when the creditors cannot in any manner collect the claims due them. It is, therefore, "essential that the party asking for rescission prove that he has exhausted all other legal means to obtain satisfaction of his claim. 20 Petitioner neither alleged nor proved that she did so. On this score, her action for the rescission of the questioned deed is not maintainable even if the fraud charged actually did exist."

Noceda vs. Court of Appeals G.R. No. 119730, September 2, 1999, 313 SCRA 504 Gonzaga – Reyes, J. FACTS: Celestino Arbizo died in 1956 leaving behind a parcel of land having an area of 66,530 square meters. His heirs plaintiff Aurora Directo, defendant Rodolfo Noceda, and Maria Arbizo extrajudicially settled the partition of the land with Directo getting 11,426 square meters, Noceda got 13,294 square meters, and Arbizo got 41,810 square meters. Plaintiff Directo donated 625 square meters of her share to defendant Noceda, who is her nephew being the son of her deceased sister However another extrajudicial settlement-partition was executed. Three fifths of the said land went to Maria Arbizo while plaintiff Directo and defendant Noceda got only one-fifth each. Sometime in 1981, Noceda constructed his house on the land donated to him by Directo. Directo fenced the portion allotted to her in the extrajudicial settlement, excluding the donated portion, and constructed thereon three huts. But in 1985, Noceda removed the fence earlier constructed by Directo, occupied the three huts (3) and fenced the entire land of plaintiff Directo without her consent. Directo demanded from Noceda to vacate her land, but the latter refused. Hence, Directo filed a complaint for the recovery of possession and ownership and rescission/annulment of donation, against defendant Noceda ISSUE: Whether or not the acts of Noceda constitute ingratitude to warrant revocation of the donation. HELD: Yes. It was established that petitioner Noceda occupied not only the portion donated to him by private respondent Aurora Arbizo-Directo but he also fenced the whole area of Lot C which belongs to private respondent Directo, thus petitioner's act of occupying the portion pertaining to private respondent Directo without the latter's knowledge and consent is an act of usurpation which is an offense against the property of the donor and considered as an act of ingratitude of a donee against the donor. The law does not require conviction of the donee; it is enough that the offense be proved in the action for revocation. 133

The action to revoke by reason of ingratitude prescribes within one (1) year to be counted from the time (a) the donor had knowledge of the fact; (b) provided that it was possible for him to bring the action. It is incumbent upon petitioner to show proof of the concurrence of these two conditions in order that the one (1) year period for bringing the action be considered to have already prescribed. No competent proof was adduced by petitioner to prove his allegation.

Heirs of Cesario Velasquez v. Court of Appeals G.R. No. 126996, February 15, 2000, 325 SCRA 552 Gonzaga – Reyes, J. FACTS: Spouses Leoncia de Guzman and Cornelio Aquino died intestate sometime in 1945 and 1947, respectively and were childless, leaving 6 parcels of land situated in Pangasinan. Leoncia De Guzman was survived by her sisters Anatalia de Guzman and Tranquilina de Guzman. Sometime in 1989, the Meneses(heirs of Anatalia de Guzman) filed a complaint for annulment, partition and damages against the heirs of Cesario Velasquez(son of Tranquilina de Guzman) for the latters' refusal to partition the properties of the Spouses Aquino. The complaint alleged that before Leoncia’s death, she told that the documents of donation and partition which she and her husband earlier executed were not signed by them as it was not their intention to give away all the properties to Cesario Velasquez because Anatalia de Guzman who is one of her sisters had several children to support; and that Cesario Velasquez and his mother allegedly promised to divide the properties equally and to give the plaintiffs one-half thereof. Plaintiffs further claim that after the death of Leoncia, defendants forcibly took possession of all the properties and despite plaintiffs' repeated demands for partition, defendants refused. Plaintiffs prayed for the nullity of the documents covering the properties in question since they do not bear the genuine signatures of the Aquino spouses, to order the partition of the properties between plaintiffs and defendants in equal shares and to order the defendants to render an accounting of the produce of the land from the time defendants forcibly took possession until partition shall have been effected. Defendants filed their answer with counterclaim alleging that during the lifetime of spouses Aquino, they had already disposed of their properties in favor of petitioners' predecessors-in-interest, Cesario Velasquez and Camila de Guzman, and petitioners Anastacia and Jose Velasquez in the following manner: (1) The third and sixth parcels were conveyed to defendants' late parents Cesario Velasquez and Camila de Guzman, by virtue of a Escritura de Donation Propter Nuptias dated February 15, 1919; 134

(2) The second parcel was conveyed to defendants' late parents Cesario Velasquez and Camila de Guzman by virtue of a deed of conveyance dated July 14, 1939; (3) The first parcel was likewise conveyed to defendants Jose Velasquez and Anastacia Velasquez by virtue of a deed of conveyance (Donation Intervivos) dated April 10, 1939; (4) As to the fourth and fifth parcels, the same were owned and possessed by third parties. The trial court ruled in favor of the plaintiffs, giving credibility to Santiago Meneses’ testimony; declaring the Donation Intervivos, the Deed of Sale, the Deed of Donation, the Deed of Sale to third parties over the 4th and 5th parcels as null and void insofar as 1/2 of the 6 parcels are concerned legitimately belong to the plaintiffs; and ordering the defendants to pay damages. Defendants appealed the decision to respondent CA which affirmed the same. A motion for reconsideration was filed by the petitioners but the same was denied. ISSUE: Whether or not the petitioners have acquired absolute and exclusive ownership of the properties in question. HELD: Yes. Private respondent Santiago Meneses failed to prove the nullity of the Deeds of Conveyance executed by the Aquino spouses in favor of petitioners and their predecessors-in-interest Cesario Velasquez and Camila de Guzman since he failed to adduce any evidence to support his claim other than his bare allegations of its nullity. On the other hand, petitioners were able to show by documentary evidence that the Aquino spouses during their lifetime disposed of the four parcels of land subject of the complaint, to wit: (a) Escritura de donation propter nuptias, (b) Deed of donation inter vivos, (c) Escritura de Compreventa with a P500 consideration: (d) Deed of Conveyance with a consideration of P600 and confirming in the same Deed the Escritura de donation propter nuptias and Escritura de compraventa abovementioned. It was reversible error for the court to overlook the probative value of these notarized documents. A donation as a mode of acquiring ownership results in an effective transfer of title over the property from the donor to the donee and the donation is perfected from the moment the donor knows of the acceptance by the donee. Once a donation is accepted, the donee becomes the absolute owner of the property donated. The donation of the first parcel made by the Aquino spouses to petitioners Jose and Anastacia Velasquez who were then 19 and 10 years old respectively was accepted through their father Cesario Velasquez, and the acceptance was incorporated in the body of the same deed of donation and made part of it, and was signed by the donor and the acceptor. Legally speaking there was delivery and acceptance of the deed, and the donation existed perfectly and irrevocably. The donation inter vivos may be revoked only for the reasons provided in Articles 760, 764 and 765 of the Civil Code.

135

The donation propter nuptias in favor of Cesario Velasquez and Camila de Guzman over the third and sixth parcels including a portion of the second parcel became the properties of the spouses Velasquez since 1919. The deed of donation propter nuptias can be revoked by the non-performance of the marriage and the other causes mentioned in Article 86 of the Family Code. The alleged reason for the repudiation of the deed, i.e., that the Aquino spouses did not intend to give away all their properties since Anatalia had several children to support is not one of the grounds for revocation of donation either inter vivos or propter nuptias, although the donation might be inofficious. The Escritura compraventa over another portion of the second parcel and the Deed of conveyance dated July 14, 1939 in favor of Cesario and Camila Velasquez over the remaining portion of the second parcel is also valid. In fact in the deed of sale, the Aquino spouses ratified and confirmed the rights and interests of Cesario Velasquez and Camila de Guzman including the previous deeds of conveyance over the second parcel in the complaint and such deed of sale became the basis for the issuance of TCT in the names of Cesario Velasquez and Camila de Guzman. The best proof of the ownership of the land is the certificate of title and it requires more than a bare allegation to defeat the face value of TCT which enjoys a legal presumption of regularity of issuance. Notably, during the lifetime of Cesario Velasquez, he entered into contracts of mortgage and lease over the property as annotated at the back of the certificate of title which clearly established that he exercised full ownership and control over the property. Petitioners were able to establish that these four parcels of land were validly conveyed to them by the Aquino spouses, hence, they no longer formed part of the conjugal properties of the spouses at the time of their deaths. As regards the fourth and fifth parcels, petitioners alleged that these were also conveyed to third persons and they do not claim any right thereto. In view of the foregoing, the action of partition cannot be maintained. The properties sought to be partitioned by private respondents have already been delivered to petitioners and therefore no longer part of the hereditary estate which could be partitioned. No co-ownership exists between private respondents and petitioners.

Gonzales v. Court of Appeals G.R. No. 110335, June 18, 2001, 358 SCRA 598 Melo, J. FACTS: Deceased spouses Ignacio and Marina Gonzales were registered owners of two parcels of agricultural land. Petitioners are the successors-in-interest or the children and grandchildren of the deceased spouses. On the other hand, private respondents are the farmers and tenants of said spouses who have been cultivating the parcels of land even before World War II either personally or through their predecessors-ininterest. Marina Gonzales died intestate and appointed as administratix was petitioner 136

Lilia Gonzales. Prior to the partition of said estate, Ignacio Gonzales executed a Deed of Donation in favor of his grandchildren but was not registered. When Presidential Decree No. 27 took effect, the landholdings of the said spouses were placed under Operation Land Transfer. Private respondents were then issued the corresponding Certificates of Land Transfer. The administratix of the spouses’ estate, Lilia Gonzales filed an application for retention requesting that their property be excluded from the Operation Land Transfer. Initially, it was denied but was approved due to the deed of donation. ISSUE: Whether or not the property subject of the deed of donation which was not registered when P.D. No. 27 took effect, should be excluded from the Operation Land Transfer. HELD: No. Article 749 of the Civil Code provides inter alia that “in order that the donation of an immovable may be valid, it must be made in a public document, specifying therein the property donated and the value of the charges which the done must satisfy.” Corollarily, Article 709 of the same Code explicitly states that “the titles of ownership, or other rights over immovable property, which are not duly inscribed or annotated in the Registry of Property shall not prejudice third persons.” From the foregoing provisions, it may be inferred that as between the parties to a donation of immovable property, all that is required is for said donation to be contained in a public document. Registration is not necessary for it to be contained in a public document. It is not necessary for it to be considered valid and effective. However, in order to bind third persons, the donation must be registered in the Registry of Property. In the case at bar, the donation executed by Ignacio Gonzales in favor of his grand children, although in writing and duly notarized, has not been registered in accordance with law. For this reason, it shall not be binding upon private respondents who did not participate in said deed nor had no actual knowledge thereof.

Imperial v. Court of Appeals G.R. No. 112483, October 8, 1999, 316 SCRA 393 Gonzaga – Reyes, J. FACTS: Leoncio Imperial was the owner of a parcel of land with an area of 32,837 sq. m. and located in Albay. On July 7, 1951, Leoncio sold the lot for Php 1.00 to his acknowledged natural son, petitioner in this case. Petitioner and Victor Imperial, adopted son of Leoncio, agreed that despite the designation of the contract as Deed of Absolute Sale, the transaction is in fact a Donation. Two years after, Leoncio filed a complaint for the Annulment of Donation. It was however resolved through a compromise agreement under the following terms and conditions: (1) Leoncio recognized and agreed the legality and validity of the rights of petitioner; and (2) petitioner agreed to sell a designated 1,000 sq.m. portion of the donated land.

137

Leoncio died leaving only two heirs: petitioner and Victor Imperial. On March 8, 1962, Victor was substituted in the complaint for annulment. He moved for the execution of judgment and it was granted. After 15 years, Victor died and was survived only by his natural father, Ricardo Villalon. Ricardo Villalon is a lessee of the portion of the subject property. Villalon died leaving his heirs, Cesar and Teresa Villalon, respondents in this case. In 1986, respondents filed a complaint for the annulment of the donation. Allegedly, it impairs the legitime of Victor Imperial. ISSUES: 1.) Whether or not the respondents have the right to question the inofficious donation and seek its reduction. 2.) Whether or not the 30-year prescriptive period is applicable in the reduction of the inofficious donation. HELD: 1.) Yes. At the time of the substitution, the judgment approving the compromise agreement has already been rendered. Victor merely participated in the execution of the compromise judgment. He was not a party to the compromise agreement. When Victor substituted Leoncio, he was not deemed to have renounced his legitime. He was therefore not precluded or estopped from subsequently seeking the reduction. Nor are Victor’s heirs, upon his death, precluded from doing so. This is in accordance with Articles 772 and 1053 of the new Civil Code, to wit: Article 772. Only those who at the time of the donor’s death have a right to the legitime and their heirs and successors in interest may ask for the reduction of the inofficious donation xxx. and Article 1053. If the heir should die without having accepted or repudiated the inheritance, his rights shall be transmitted to his heirs. 2.) No. Under Article 1144 of the New Civil Code, actions upon an obligation created by law must be brought within ten years from the right of action accrues. Thus, the 10-year prescriptive period applies to the obligation to reduce inofficious donations required under Article 771 of the New Civil Code to the extent that they impair the legitime of compulsory heirs. The cause of action to enforce a legitime accrues upon the death of the donordecedent. Clearly so, since only then that the net estate may be ascertained and on which basis, the legitimes may be determined. It took 24 years since the death of Leoncio to initiate this case. Thus, the action has long prescribed. Not only has prescription set in, they are also guilty of estoppel and laches. Fifteen years after the death of Leoncio, Victor died. Ricardo Villalon, Victor’s sole heir, died four years later. While Victor was alive, he gave no indication of any interest to contest the donation of his deceased father.

138

Republic of the Philippines v. Silim G.R. No. 140487, April 2, 2001, 356 SCRA 1 Kapunan, J. FACTS: Respondent Spouses Silim and Mangubat donated a 5,600 square meter parcel of land in favor of the Bureau of Public Schools of the Municipality of Malangas, Zamboanga del Sur. In the Deed of Donation, the respondents imposed the condition that the said property should be “used exclusively and forever for school purposes only.” This donation was accepted by the District Supervisor of the Bureau, through an Affidavit of Acceptance and/or Confirmation of Donation. A school building was thereafter constructed on the donated land. However, another school building that was also supposed to be allocated for the donated parcel of land could not be released since the government required that it be built upon a 1 hectare parcel of land. By reason of this, the District Supervisor and the vice-mayor’s wife entered into a Deed of Exchange whereby the donated lot was exchanged with a bigger lot owned by the latter. Consequently, the school buildings were constructed on this new school site and the school building previously erected on the donated land was dismantled and transferred to the new location. One day, respondents were surprised when he saw the vice-mayor constructing a house on the donated land. ISSUES: 1.) Whether or not there was a valid donation despite non-notation of the acceptance in the Deed of Donation, as required in Article 749. 2.) Whether or not the condition on the donation was violated. HELD: 1.) Yes. The purpose of the formal requirement for acceptance of a donation is to ensure that such is duly communicated to the donor. In the case at bar, a school building was immediately constructed after the donation was executed. Respondents had knowledge of the existence of the school building put up on the donated lot. The actual knowledge by respondents of the construction and existence of the school building fulfilled the legal requirement that the acceptance of the donation by the donee be communicated to the donor. 2.) No. There was no violation even after the donated lot was exchanged for another one. The purpose of the donation remains the same, which is for the establishment of a school. The exclusivity of the purpose was not altered or affected. In fact, the exchange of the lot for a much bigger one was in furtherance and enhancement of the purpose of the donation. The acquisition of the bigger lot paved the way for the release of funds for the construction of Bagong Lipunan school building which could not be accommodated by the limited area of the donated lot.

139

Gestopa v. Court of Appeals G.R. No. 111904, October 5, 2000, 342 SCRA 105 Quisumbing, J. FACTS: Spouses Danlag were the owners of six parcels of unregistered lands. They executed three deeds of donation mortis causa, two of which were in favor of Mercedes Danlag-Pilapil. All deeds contained the reservation of the rights of the donors to amend, cancel or revoke the donation during their lifetime, and to sell, mortgage, or encumber the properties donated during the donors' lifetime, if deemed necessary. The spouses then executed another deed of donation inter vivos in favor of Mercedes which contained the condition that the donors, Danlag spouses, shall continue to enjoy the fruits of the land during their lifetime and that the donee enjoy the fruits of the land during their lifetime and that the donee cannot sell or dispose of the land during the lifetime of the donors without their prior consent and approval. Consequently, Mercedes caused the transfer of the parcels of land's tax declaration to her name and paid the taxes on them. Spouses Danlag sold two parcels of lots to Spouses Agripino and Isabel Gestopa and executed a deed of revocation recovering the six parcels of land subject to the deed of donation inter vivos. Mercedes Pilapil filed with the Regional Trial Court against the Spouses Danlag and Gestopa, for quieting of title over the parcels of land and alleged that the land was donated to her by Diego Danlag and that she accepted the donation openly and publicly exercised rights of ownership over the donated properties, and transferred the tax declarations to her name. She also alleged that the donation inter vivos was coupled with conditions and, according to Mercedes, since its perfection, she had complied with all of them; that she had not been guilty of any act of ingratitude; and that Diego Danlag had no legal basis to revoke the donation and then in selling the two parcels of land to the Gestopa spouses. In their opposition, the spouses Gestopa and the Danlag averred that the deed of donation was null and void because it was obtained by Mercedes through machinations and undue influence. Even assuming it was validly executed, the intention was for the donation to take effect upon the death of the donor and that the donation was void for it left the donor, Diego Danlag, without any property at all. ISSUE: Whether the donation is a donation inter vivos or a donation mortis causa. HELD: The Court Rules that it was a donation inter vivos. The Court affirmed the Court of Appeals' decision that the reservation by the donor of lifetime usufruct indicated that he transferred to Mercedes the ownership over the donated properties; that the right to sell belonged to the donee, and the donor's right referred to that of merely giving consent; that the donor changed his intention by donating inter vivos properties already donated mortis causa; that the transfer to Mercedes' name of the tax declarations pertaining to the donated properties implied that the donation was inter vivos; and that Mercedes did not purchase two of the six parcels of land donated to her.

140

In ascertaining the intention of the donor, all of the deed's provisions must be read together. The granting clause shows that Diego donated the properties out of love and affection for the donee. This is a mark of a donation inter vivos. Second, the reservation of lifetime usufruct indicates that the donor intended to transfer the naked ownership over the properties. Third, the donor reserved sufficient properties for his maintenance in accordance with his standing in society, indicating that the donor intended to part with the six parcels of land. Lastly, the donee accepted the donation. An acceptance clause is a mark that the donation is inter vivos. Acceptance is a requirement for donations inter vivos. Donations mortis causa, being in the form of a will, are not required to be accepted by the donees during the donors' lifetime. The right to dispose of the properties belonged to the donee. The donor's right to give consent was merely intended to protect his usufructuary interests. The limitation on the right to sell during the donors' lifetime implied that ownership had passed to the donees and donation was already effective during the donors' lifetime. Hence, the moment that it was accepted by Mercedes Danlag-Pilapil, ownership of the properties was transferred.

Sumipat v. Banga G.R. No. 155810, August 13, 2004 Tinga, J. FACTS: The spouses Placida Tabo-tabo and Lauro Sumipat acquired three parcels of land. The couple was childless. Lauro Sumipat, however, sired five illegitimate children. They are the petitioners herein. Lauro executed a document denominated “Deed of Absolute Transfer and/or Quit-Claim over Real Properties” in favor of the petitioners. On the document, it appears that the signature of his wife, Placida which indicates that she gave her marital consent. Moreover, it was alleged that Lauro executed it when he was already very sick and bedridden that upon petitioner Lydia’s request, their neighbor Benjamin Rivera lifted the body of Lauro whereupon Lydia guided his hand in affixing his signature on the document. Lydia left but later returned on the same day and requested Lauro’s unlettered wife, Placida to sign on the said document. After Lauro’s death, his wife, Placida and petitioners jointly administered the properties, 50% of the produce went to his wife. As wife’s share in the produce of the properties dwindled, she filed a complaint for declaration of partition disclaiming any partition in the execution of the subject document. ISSUE: Whether or not the questioned deed by its terms or under the surrounding circumstances has validly transferred title to the disputed properties to the petitioners. HELD: No. A perusal of the deed reveals that it is actually a gratuitous disposition of property — a donation — although Lauro Sumipat imposed upon the petitioners the condition that he and his wife, Placida, shall be entitled to one-half (1/2) of all the fruits or produce of the parcels of land for their subsistence and support. Where the deed of donation fails to show the acceptance, or where the formal notice of the acceptance, 141

made in a separate instrument, is either not given to the donor or else not noted in the deed of donation and in the separate acceptance, the donation is null and void. In this case, the donees’ acceptance of the donation is not manifested either in the deed itself or in a separate document. Hence, the deed as an instrument of donation is patently void. The Court declared that the deeds of sale questioned therein are not merely voidable but null and void ab initio as the supposed seller declared under oath that she signed the deeds without knowing what they were. The significant circumstance meant, the Court added, that her consent was not merely marred by vices of consent so as to make the contracts voidable, but that she had not given her consent at all.

142

View more...

Comments

Copyright ©2017 KUPDF Inc.
SUPPORT KUPDF